Anda di halaman 1dari 46

TOPNOTCH MEDICAL BOARD PREP PHYSIOLOGY GANONG SUPPLEMENT HANDOUT BY THE TOPNOTCH TEAM

For inquiries visit www.topnotchboardprep.com.ph or https://www.facebook.com/topnotchmedicalboardprep/

PHYSIOLOGY SUPPLEMENTAL
Instructions: Please Read Ganong’s Review of Medical Physiology 23rd or 25th Ed before using this.

SECTION 1: CELLULAR & MOLECULAR BASIS FOR MEDICAL PHYSIOLOGY


(CHAPTER 1) GENERAL PRINCIPLES & ENERGY PRODUCTION IN MEDICAL PHYSIOLOGY
ANSWER: B
• Equilibrium potential can be calculated for K+:
1. The membrane potential of a particular cell is at the EK= 61.5loG [Ko+]/[Ki+] at 37 °C
K+equilibrium. The intracellular concentration for K+ is at EK= 61.5loG [5.5]/[150]
150 mmol/L and the extracellular concentration for K+ is EK= -90 mV
at 5.5 mmol/L. What is the resting potential? Where
A. –70 mV EK = equilibrium potential for K+
B. –90 mV [Ko+] = K+ concentration outside the cell
C. +70 mV [Ki+] = K+ concentration inside the cell
D. +90 mV • Chloride has equilibrium potential of -70 mV, Sodium has equilibrium
potential of +60 mV.
Ganong. Review of Medical Physiology 23rd ed. Chapter 1, page .7
2. The difference in concentration of H+ in a solution of pH ANSWER: C
2.0 compared with one of pH 7.0 is: • “For each pH unit less than 7.0, the [H+] is increased tenfold (10 1); for
A. 5-fold each pH unit above 7.0, it is decreased tenfold (10-1).”
B. 1/5 as much • The difference in pH between 2.0 and 7.0 is 5, therefore an increased of
C. 105-fold 105 is expected.
D. 10–5 as much Ganong. Review of Medical Physiology 23rd ed. Chapter 1, page 4.
3. Transcription refers to: ANSWER: B
A. the process where an mRNA is used as a template for
protein production. • Transcription is the process where a DNA sequence is copied into RNA
B. the process where a DNA sequence is copied into RNA for the purpose of gene expression.
for the purpose of gene expression. • Translation is the process where an mRNA is used as a template for
C. the process where DNA wraps around histones to protein production.
form a nucleosome. • Synthesis is the process of replication of DNA prior to cell division.
Ganong. Review of Medical Physiology 23rd ed. Chapter 1, page 4.
D. the process of replication of DNA prior to cell division
ANSWER: C
4. The primary structure of a protein refers to:
A. the twist, folds, or twist and folds of the amino acid • Primary structure is the amino acid sequence of the protein.
sequence into stabilized structures within the protein • Secondary structure is the twist, folds, or twist and folds of the amino
(ie, α-helices and β-sheets). acid sequence into stabilized structures within the protein (ie, α-helices
B. the arrangement of subunits to form a functional and β-sheets).
structure. • Tertiary structure is the arrangement of twisted chains and folds within
C. the amino acid sequence of the protein. a protein into a stable structure.
D. the arrangement of twisted chains and folds within a • Quaternary structure is the arrangement of subunits to form a
protein into a stable structure. functional structure.
Ganong. Review of Medical Physiology 23rd ed. Chapter 1, page 17.
5. Fill in the blanks: Glycogen is a storage form of glucose. ANSWER: C
_______ refers to the process of making glycogen and _______
refers to the process of breakdown of glycogen. • “For each pH unit less than 7.0, the [H+] is increased tenfold (10 1); for
A. glycogenolysis, glycogenesis each pH unit above 7.0, it is decreased tenfold (10-1).”
B. glycolysis, glycogenolysis • The difference in pH between 2.0 and 7.0 is 5, therefore an increased of
C. glycogenesis, glycogenolysis 105 is expected.
Ganong. Review of Medical Physiology 23rd ed. Chapter 1, page 20
D. glycogenolysis, glycolysis
ANSWER:D
• “Low-density lipoproteins (LDL) is the major lipoprotein source of the
6. The major lipoprotein source of the cholesterol used in cholesterol used in cells. Chylomicrons is the transport system for
cells is: ingested exogenous lipids (exogenous pathway).
A. chylomicrons • Intermediate-density lipoproteins (IDL) give up phospholipids and,
B. intermediate-density lipoproteins (IDL) through the action of the plasma enzyme lecithin-cholesterol
C. albumin-bound free fatty acids acyltransferase (LCAT), pick up cholesteryl esters formed from
D. low-density lipoproteins (LDL) cholesterol in the HDL.
E. high-density lipoproteins (HDL) • High-density lipoproteins (HDL), also transports triglycerides and
cholesterol throughout the body.”
Ganong. Review of Medical Physiology 23rd ed. Chapter 1 , page 27 .
7. Which of the following produces the most high-energy ANSWER:
phosphate compounds?
A. Aerobic metabolism of 1 mol of glucose • “Catabolism of 1 mol of a six-carbon fatty acid through the citric acid
B. Anaerobic metabolism of 1 mol of glucose cycle to CO2 and H2O generates 44 mol of ATP, compared with the 38
C. Metabolism of 1 mol of galactose mol generated by catabolism of 1 mol of the six-carbon carbohydrate
D. Metabolism of 1 mol of amino acid glucose.”
Ganong. Review of Medical Physiology 23rd ed. Chapter 1 , page 23.
E. Metabolism of 1 mol of long-chain fatty acid
8. When LDL enters cells by receptor-mediated endocytosis, ANSWER: E
which of the following does not occur? • When LDL enters cells by receptor-mediated endocytosis, there will be:
A. in the formation of cholesterol from mevalonic acid 1. Decrease in the formation of cholesterol and in the rate of synthesis of
B. in the intracellular concentration of cholesteryl LDL receptors.
esters 2. Increase in the intracellular concentration of cholesteryl esters,
C. in the transfer of cholesterol from the cell to HDL transfer of cholesterol from the cell to HDL, and amount of
D.  in the rate of synthesis of LDL receptors cholesterol in endosomes
E.  in the cholesterol in endosomes Ganong. Review of Medical Physiology 23rd ed. Chapter 1 , page 27.
TOPNOTCH MEDICAL BOARD PREP MED PHYSIOLOGY GANONG SUPPLEMENT HANDOUT BY THE TOPNOTCH TEAM Page 1 of 46
For inquiries visit www.topnotchboardprep.com.ph or https://web.facebook.com/topnotchmedicalboardprep/
TOPNOTCH MEDICAL BOARD PREP PHYSIOLOGY GANONG SUPPLEMENT HANDOUT BY THE TOPNOTCH TEAM
For inquiries visit www.topnotchboardprep.com.ph or https://www.facebook.com/topnotchmedicalboardprep/
(CHAPTER 2) OVERVIEW OF CELLULAR PHYSIOLOGY IN MEDICAL PHYSIOLOGY
ANSWER: A
1. The electrogenic Na, K ATPase plays a critical role in
cellular physiology by
A. using the energy in ATP to extrude 3 Na+ out of the
cell in exchange for taking two K+ into the cell. • “Na, K ATPase catalyzes the hydrolysis of ATP to adenosine diphosphate
B. using the energy in ATP to extrude 3 K+ out of the cell (ADP) and uses the energy to extrude three Na+ from the cell and take
in exchange for taking two Na+ into the cell. two K+ into the cell for each molecule of ATP hydrolyzed.”
C. using the energy in moving Na+ into the cell or K+ • It is an electrogenic pump which does not produce new ATP.
outside the cell to make ATP.
Ganong. Review of Medical Physiology 23rd ed. Chapter 2, page 47.
D. using the energy in moving Na+ outside of the cell or
K+ inside the cell to make ATP.

2. Cell membranes: ANSWER: D


A. contain relatively few protein molecules.
B. contain many carbohydrate molecules. • “Cellular Membranes have variable protein and lipid contents
C. are freely permeable to electrolytes but not to depending on their location in the cell and is semipermeable, allowing
proteins. some substances to pass through it and excluding others. There were
D. have variable protein and lipid contents depending on many proteins embedded in the membrane with abundant
their location in the cell. phospholipids.”
E. have a stable composition throughout the life of the • Cell membrane have variable composition throughout life of the cell.
Ganong. Review of Medical Physiology 23rd ed. Chapter 2, page 32 .
cell.
3. Second messengers: ANSWER: D
A. are substances that interact with first messengers
outside cells.
B. are substances that bind to first messengers in the cell • “Second messengers bring about many short-term changes in cell
membrane. function by altering enzyme function, triggering exocytosis, and also can
C. are hormones secreted by cells in response to lead to the alteration of transcription of various genes.”
stimulation by another hormone.
D. mediate the intracellular responses to many different Ganong. Review of Medical Physiology 23rd ed. Chapter 2, page 51..
hormones and neurotransmitters.
E. are not formed in the brain.
ANSWER: B
4. The Golgi complex
• Golgi complex is an organelle that participates in posttranslational
A. is an organelle that participates in the breakdown of
processing of proteins.
proteins and lipids.
B. is an organelle that participates in posttranslational • Lysosome is an organelle that participates in the breakdown of proteins
processing of proteins. and lipids.
C. is an organelle that participates in energy production. • Mitochondrion is an organelle that participates in energy production.
D. is an organelle that participates in transcription and • Nucleus is an organelle that participates in transcription and
translation. translation.
E. is a subcellular compartment that stores proteins for • Ribosomal Endoplasmic Reticulum is a subcellular compartment that
trafficking to the nucleus. stores proteins for trafficking to the nucleus.
Ganong. Review of Medical Physiology 23rd ed. Chapter 2, page 40.
ANSWER: C
5. Endocytosis
A. includes phagocytosis and pinocytosis, but not
clathrin mediated or caveolae-dependent uptake of • “Endocytosis refers to the invagination of the plasma membrane to
extracellular contents. uptake extracellular contents into the cell. It includes phagocytosis,
B. refers to the merging of an intracellular vesicle with pinocytosis, clathrin mediated or caveolae dependent uptake of
the plasma membrane to deliver intracellular extracellular contents.”
contents to the extracellular milieu. • “Exocytosis refers to vesicular trafficking between Golgi stacks. The
C. refers to the invagination of the plasma membrane to process refers to the merging of an intracellular vesicle with the plasma
uptake extracellular contents into the cell. membrane to deliver intracellular contents to the extracellular milieu.”
Ganong. Review of Medical Physiology 23rd ed. Chapter 2, page 43 .
D. refers to vesicular trafficking between Golgi stacks.

6. G-protein–coupled receptors: ANSWER: C


A. are intracellular membrane proteins that help
regulate movement within the cell.
B. are plasma membrane proteins that couple the
extracellular binding of primary signaling molecules • “G-protein–coupled receptors are plasma membrane proteins that
to exocytosis. couple the extracellular binding of primary signaling molecules to the
C. are plasma membrane proteins that couple the activation of heterotrimeric G-proteins.”
extracellular binding of primary signaling molecules
Ganong. Review of Medical Physiology 23rd ed. Chapter 2, page 54..
to the activation of heterotrimeric G-proteins.
D. are intracellular proteins that couple the binding of
primary messenger molecules with transcription.
ANSWER: B
• “Tight junctions provide intercellular connections that link cells into a
7. Gap junctions are intercellular connections that
regulated tissue barrier, it also provides barrier to movement of
A. primarily serve to keep cells separated and allow for
proteins in the cell membrane and thus, are important to cellular
transport across a tissue barrier.
polarization.
B. serve as a regulated cytoplasmic bridge for sharing of
small molecules between cells. • Gap junctions provide contacts between cells that allow for direct
C. serve as a barrier to prevent protein movement passage of small molecules between two cells.
within the cellular membrane. • Desmosomes and adherens junctions are specialized structures that
D. are cellular components for constitutive exocytosis hold cells together.
that occurs between adjacent cells. • Hemidesmosomes and focal adhesions attach cells to their basal
lamina.”
Ganong. Review of Medical Physiology 23rd ed. Chapter 2 , page 60 ..
TOPNOTCH MEDICAL BOARD PREP MED PHYSIOLOGY GANONG SUPPLEMENT HANDOUT BY THE TOPNOTCH TEAM Page 2 of 46
For inquiries visit www.topnotchboardprep.com.ph or https://web.facebook.com/topnotchmedicalboardprep/
TOPNOTCH MEDICAL BOARD PREP PHYSIOLOGY GANONG SUPPLEMENT HANDOUT BY THE TOPNOTCH TEAM
For inquiries visit www.topnotchboardprep.com.ph or https://www.facebook.com/topnotchmedicalboardprep/
ANSWER: A
• “Filamentous (F) actin provides a structural component for cell
8. F-actin is a component of the cellular cytoskeleton that:
movement.
A. provides a structural component for cell movement.
• Globular (G) actin refers to the actin subunits that provide the
B. is defined as the “functional” form of actin in the cell.
molecular building blocks of the extended actin molecules found in the
C. refers to the actin subunits that provide the molecular
cell.
building blocks of the extended actin molecules found in
the cell. • Microtubules provides the molecular architecture for cell to cell
D. provides the molecular architecture for cell to cell communication.
communication. • Intermediate filaments form a flexible scaffolding for the cell and help
it resist external pressure.”
Ganong. Review of Medical Physiology 23rd ed. Chapter 2, pages 35,36 ..

(CHAPTER 3) IMMUNITY, INFECTION, & INFLAMMATION


1. In an experiment, a scientist treats a group of mice with ANSWER: B
an antiserum that substantially depletes the number of
circulating neutrophils. Compared with untreated control • “Neutrophils mount phagocytic responses that engulf and destroy
animals, the mice with reduced numbers of neutrophils bacteria. Reduced number of which, will lead to deficiency in oxidation
were found to be significantly more susceptible to death or release of reactive oxygen species.”
induced by bacterial inoculation. The increased mortality • Acquired immunity is the ability of lymphocytes to produce antibodies.
can be ascribed to a relative deficit in which of the • GM-CSF stimulates production of monocyte and granulocyte, reduced
following? neutrophil will not lead in deficiency of this factors.
A. Acquired immunity • Platelets are circulating cells that are important mediators of
B. Oxidants hemostasis.
C. Platelets • Integrins promotes cellular adhesion.
D. Granulocyte/macrophage colony stimulating factor
(GM-CSF) Ganong. Review of Medical Physiology 23rd ed. Chapter 3 , page 64 .
E. Integrins
ANSWER: C
2. A 20-year-old college student comes to the student health
center in April complaining of runny nose and congestion,
itchy eyes, and wheezing. She reports that similar
symptoms have occurred at the same time each year, and • IgE: releases histamine from basophils and mast cells which triggers the
that she obtains some relief from over-the-counter patient’s symptoms.
antihistamine drugs, although they make her too drowsy • IgG: Complement activation
to study. Her symptoms can most likely be attributed to • IgA: Localized protection in external
inappropriate synthesis of which of the following • secretions (tears, intestinal secretions,
antibodies specific for tree pollen? • IgM: Complement activation
A. IgA • IgD : Antigen recognition by B cells
B. IgD
C. IgE Ganong. Review of Medical Physiology 23rd ed. Chapter 3 , page 74 .
D. IgG
E. IgM

ANSWER: E
3. If a nasal biopsy were performed on the patient described
• Mast cell degranulation produces clinical manifestations of allergy up to
in Question 2 while symptomatic, histologic examination
and including anaphylaxis.
of the tissue would most likely reveal degranulation of
which of the following cell types? • Dendritic cells are antigen presenting cells.
A. Dendritic cells • Lymphocytes are key elements in the production of immunity.
B. Lymphocytes • Neutrophils enters the tissues, particularly if triggered to do so by an
C. Neutrophils infection or by inflammatory cytokines.
D. Monocytes • Monocytes are activated into macrophage and migrate in response to
E. Mast cells chemotactic stimuli and engulf and kill bacteria.
Ganong. Review of Medical Physiology 23rd ed. Chapter , pages 64, 65,71 .
ANSWER: B
4. A biotechnology company is working to design a new
therapeutic strategy for cancer that involves triggering an
enhanced immune response to cellular proteins that are • “The class I MHC proteins (MHC-I proteins) are coupled primarily to
mutated in the disease. Which of the following immune peptide fragments to which the host is not tolerant (eg, those from
cells or processes will most likely not be required for a mutant or viral proteins) are recognized by cytotoxic T cells.
successful therapy? • The digestion of these proteins occurs in proteasomes. The class II MHC
A. Cytotoxic T cells proteins (MHC-II proteins) are concerned primarily with peptide
B. Antigen presentation in the context of MHC-II products of extracellular antigens, such as bacteria, that enter the cell
C. Proteosomal degradation by endocytosis and are digested in the late endosomes.”
D. Gene rearrangements producing T cell receptors
Ganong. Review of Medical Physiology 23rd ed. Chapter 3 , page 71 ..
E. The immune synapse
ANSWER: B

5. The ability of the blood to phagocytose pathogens and • Granulocyte colony-stimulating factor (G-CSF): promotion of the
mount a respiratory burst is increased by: growth of granulocytes which is responsible for phagocytosis and
A. interleukin-2 (IL-2) respiratory burst.
B. granulocyte colony-stimulating factor (G-CSF) • Erythropoietin: stimulates production of RBC.
C. erythropoietin • Interleukin-4 (IL-4): plays a part in mast-cell sensitization and thus in
D. interleukin-4 (IL-4) allergy and in defense against nematode infections.
E. interleukin-5 (IL-5) • Interleukin-5 (IL-5): differentiation of eosinophils

Ganong. Review of Medical Physiology 23rd ed. Chapter 3, page 68.

TOPNOTCH MEDICAL BOARD PREP MED PHYSIOLOGY GANONG SUPPLEMENT HANDOUT BY THE TOPNOTCH TEAM Page 3 of 46
For inquiries visit www.topnotchboardprep.com.ph or https://web.facebook.com/topnotchmedicalboardprep/
TOPNOTCH MEDICAL BOARD PREP PHYSIOLOGY GANONG SUPPLEMENT HANDOUT BY THE TOPNOTCH TEAM
For inquiries visit www.topnotchboardprep.com.ph or https://www.facebook.com/topnotchmedicalboardprep/
ANSWER: C
6. Cells responsible for innate immunity are activated most • “The cells that mediate innate immunity include neutrophils,
commonly by: macrophages, and natural killer (NK) cells. All these cells respond to
A. glucocorticoids lipid and carbohydrate sequences unique to bacterial cell walls and
B. pollen to other substances characteristic of tumor and transplant cells.”
C. carbohydrate sequences in bacterial cell walls • Glucocorticoids, pollen, eosinophils, and thrombopoietin are not
D. eosinophils responsible for innate immunity.
E. thrombopoietin
Ganong. Review of Medical Physiology 23rd ed. hapter 3, page 69.
7. A patient suffering from an acute flare in his rheumatoid ANSWER: D
arthritis undergoes a procedure where fluid is removed
from his swollen and inflamed knee joint. Biochemical
analysis of the inflammatory cells recovered from the • “Rheumatoid arthritis is an inflammatory disease in which the nuclear
removed fluid would most likely reveal a decrease in factor-κB, plays a key role. NF-κB is a heterodimer that normally exists
which of the following proteins? in the cytoplasm of cells bound to IκBα, which renders it inactive.”
A. Interleukin-1 • Therefore, IκBα is decreased in rheumatoid arthritis.
B. Tumor necrosis factor-α
C. Nuclear factor-κB Ganong. Review of Medical Physiology 23rd ed. Chapter 3 , page 76.
D. IκBα
E. von Willbrand factor
8. In normal human blood: ANSWER: C
A. the eosinophil is the most common type of white
blood cell • In human blood, iron is mostly in hemoglobin, neutrophil is the most
B. there are more lymphocytes than neutrophils common type of white blood cell, and there are more red blood cells
C. the iron is mostly in hemoglobin than white blood cells and platelets.
D. there are more white cells than red cells
E. there are more platelets than red cells
ANSWER: E
9. Lymphocytes: • Lymphocytes are part of the body’s defense against cancer.
A. all originate from the bone marrow after birth. • “After birth, most of lymphocytes are formed in the lymph nodes,
B. are unaffected by hormones thymus, and spleen from precursor cells that originally came from the
C. convert to monocytes in response to antigens bone marrow.”
D. interact with eosinophils to produce platelets • Lymphocytes cannot be converted to monocytes in response to antigens
E. are part of the body’s defense against cancer and doesn’t interact with eosinophils to produce platelets.
Ganong. Review of Medical Physiology 23rd ed. Chapter 3, page 65.

(CHAPTER 4) EXCITABLE TISSUE: NERVE


1. Which of the following statements about glia is true? ANSWER: C
A. Microglia arise from macrophages outside of the
nervous system and are physiologically and • “Protoplasmic astrocytes produce substances that are tropic to
embryologically similar to other neural cell types. neurons to help maintain the appropriate concentration of ions and
B. Glia do not undergo proliferation. neurotransmitters by taking up K+ and the neurotransmitters glutamate
C. Protoplasmic astrocytes produce substances that are and GABA.”
tropic to neurons to help maintain the appropriate • “Microglia arise from macrophages outside of the nervous system and
concentration of ions and neurotransmitters by are physiologically and embryologically unrelated to other neural cell
taking up K+ and the neurotransmitters glutamate types; they are scavenger cells that resemble tissue macrophages and
and GABA. remove debris resulting from injury, infection, and disease.”
D. Oligodendrocytes and Schwann cells are involved in • “Oligodendrocytes and Schwann cells are involved in myelin
myelin formation around axons in the peripheral and formation around axons in the central and peripheral nervous
central nervous systems, respectively. systems, respectively. Glia undergo proliferation.”
E. Macroglia are scavenger cells that resemble tissue
macrophages and remove debris resulting from Ganong. Review of Medical Physiology 23rd ed. Chapter 4 , page 80 .
injury, infection, and disease.
2. Primary erythromelalgia, which may be due to a ANSWER: E
peripheral nerve sodium channelopathy, was diagnosed
in a 13-year-old girl who was experiencing frequent
episodes of red, painful, warm extremities. Which part of • “The number of Na+ channels per square micrometer of membrane in
a neuron has the highest concentration of Na+ channels myelinated mammalian neurons has been estimated to be 50–75 in the
per square micrometer of cell membrane? cell body, 350–500 in the initial segment, less than 25 on the surface
A. dendrites of the myelin, 2000–12,000 at the nodes of Ranvier, and 20–75 at the
B. cell body near dendrites axon terminals.”
C. initial segment
Ganong. Review of Medical Physiology 23rd ed. Chapter 4, page 85.
D. axonal membrane under myelin
E. node of Ranvier
3. A 45-year-old woman who works in an office had been ANSWER: E
experiencing tingling in her index and middle fingers and
thumb of her right hand. Recently, her wrist and hand had
become weak. Her physician ordered a nerve conduction • “In general, the greater the diameter of a given nerve fiber, the greater
test to evaluate her for carpal tunnel syndrome. Which one its speed of conduction.”
of the following nerves has the slowest conduction velocity? • In decreasing order of diameter and speed:
A. Aα fibers • Aα fibers > Aβ fibers > Aγ fibers > B fibers > C fibers
B. Aβ fibers
C. Aγ fibers Ganong. Review of Medical Physiology 23rd ed. Chapter 4, page 89.
D. B fibers
E. C fibers

TOPNOTCH MEDICAL BOARD PREP MED PHYSIOLOGY GANONG SUPPLEMENT HANDOUT BY THE TOPNOTCH TEAM Page 4 of 46
For inquiries visit www.topnotchboardprep.com.ph or https://web.facebook.com/topnotchmedicalboardprep/
TOPNOTCH MEDICAL BOARD PREP PHYSIOLOGY GANONG SUPPLEMENT HANDOUT BY THE TOPNOTCH TEAM
For inquiries visit www.topnotchboardprep.com.ph or https://www.facebook.com/topnotchmedicalboardprep/
ANSWER: A
4. Which of the following is not correctly paired? • “In the natural situation, impulses pass in one direction only, ie, from
A. Synaptic transmission: Antidromic conduction synaptic junctions or receptors along axons to their termination. Such
B. Molecular motors: Dynein and kinesin conduction is called orthodromic. Conduction in the opposite direction
C. Fast axonal transport: ~400 mm/day is called antidromic. Because synapses, unlike axons, permit
D. Slow axonal transport: 0.5–10 mm/day conduction in one direction only, an antidromic impulse will fail to pass
E. Nerve growth factor: Retrograde transport the first synapse they encounter and die out at that point.”
Ganong. Review of Medical Physiology 23rd ed. Chapter 4, page 88 .
5. Which of the following ionic changes is correctly matched ANSWER: C
with a component of the action potential?
• Opening of voltage-gated Na+ channels: Depolarization
A. Opening of voltage-gated K+ channels: After-
hyperpolarization • Opening of voltage-gated K+ channels: Repolarization
B. A decrease in extracellular Ca2+: Repolarization • A decrease in extracellular Ca2+: increased excitability  affects
C. Opening of voltage-gated Na+ channels: Resting Membrane Potential
Depolarization • Rapid closure of voltage-gated Na+ channels: Inactivated sate 
D. Rapid closure of voltage-gated Na+ channels: Resting Relative Refractory Period
membrane potential • Slow closure of voltage-gated K+ channels: After-hyperpolarization
E. Rapid closure of voltage-gated K+ channels: Relative
refractory period Ganong. Review of Medical Physiology 23rd ed. Chapter 4 , page 85

ANSWER: B
6. A man falls into a deep sleep with one arm under his head.
• “Conversely, pressure on a nerve can cause loss of conduction in large-
This arm is paralyzed when he awakens, but it tingles, and
diameter motor, touch, and pressure fibers while pain sensation
pain sensation in it is still intact. The reason for the loss of
remains relatively intact.”
motor function without loss of pain sensation is
A. A fibers are more susceptible to hypoxia than B
fibers.
B. A fibers are more sensitive to pressure than C fibers.
C. C fibers are more sensitive to pressure than A fibers.
D. Motor nerves are more affected by sleep than sensory
nerves.
E. Sensory nerves are nearer the bone than motor
nerves and hence are less affected by pressure.
Ganong. Review of Medical Physiology 23rd ed. Chapter 4, page 89, 90.
7. Which of the following statements about nerve growth ANSWER: B
factor is not true?
A. It is made up of three polypeptide subunits.
B. It is responsible for the growth and maintenance of • Nerve Growth Factor is made up of three polypeptide subunits. It can
adrenergic neurons in the basal forebrain and the be express both p75 NTR and Trk A receptors. It is necessary for the
striatum. growth and development of the sympathetic nervous system; and
C. It is necessary for the growth and development of the cholinergic neurons in the basal forebrain and the striatum. It is picked
sympathetic nervous system. up by nerves from the organs they innervate.
D. It is picked up by nerves from the organs they
Ganong. Review of Medical Physiology 23rd ed. Chapter 4, page 90
innervate.
E. It can express both p75 NTR and Trk A receptors.
ANSWER:
8. A 20-year-old female student awakens one morning with
severe pain and blurry vision in her left eye; the
symptoms abate over several days. About 6 months later, • “Typical physiological deficits of multiple sclerosis range from muscle
on a morning after playing volleyball with friends, she weakness, fatigue, diminished coordination, slurred speech, blurred or
notices weakness but not pain in her right leg; the hazy vision, bladder dysfunction, and sensory disturbances. Symptoms
symptoms intensify while taking a hot shower. Which of are often exasperated by increased body temperature or ambient
the following is most likely to be the case? temperature.”
A. The two episodes described are not likely to be • “In the most common form, transient episodes appear suddenly, last a
related. few weeks or months, and then gradually disappear. Subsequent
B. She may have primary-progressive multiple sclerosis. episodes can appear years later, and eventually full recovery does not
C. She may have relapsing-remitting multiple sclerosis. occur. Others have a progressive form of the disease in which there are
D. She may have a lumbar disk rupture. no periods of remission.”
Ganong. Review of Medical Physiology 23rd ed. Chapter 4, page 82.
E. She may have Guillain–Barre syndrome.

9. The distance from between one stimulating electrode to ANSWER: B


recording electrode is 4.5 cm. When the axon is
stimulated, the latent period is 1.5 ms. What is the • If the duration of the latent period and the distance between the
conduction velocity of the axon? stimulating and recording electrodes are known, axonal conduction
A. 15 m/s velocity can be calculated.
B. 30 m/s • Convert the units: 4.5cm = 0.045m ; 1.5ms = 0.0015s
C. 40 m/s • Conduction Velocity = Distance/Latent period
D. 67.5 m/s • Conduction Velocity = 0.045m/0.0015s = 30 m/s
E. This cannot be determined from the information given. Ganong. Review of Medical Physiology 23rd ed. Chapter , page .

ANSWER: C
10. Which part of a neuron has the highest concentration of
Na+ channels per square millimeter of cell membrane? • “The number of Na+ channels per square micrometer of membrane in
A. dendrites myelinated mammalian neurons has been estimated to be 50–75 in the
B. cell body near dendrites cell body, 350–500 in the initial segment, less than 25 on the surface
C. initial segment of the myelin, 2000–12,000 at the nodes of Ranvier, and 20–75 at the
D. axonal membrane under myelin axon terminals.”
E. none of the above Ganong. Review of Medical Physiology 23rd ed. Chapter 4, page 85.

TOPNOTCH MEDICAL BOARD PREP MED PHYSIOLOGY GANONG SUPPLEMENT HANDOUT BY THE TOPNOTCH TEAM Page 5 of 46
For inquiries visit www.topnotchboardprep.com.ph or https://web.facebook.com/topnotchmedicalboardprep/
TOPNOTCH MEDICAL BOARD PREP PHYSIOLOGY GANONG SUPPLEMENT HANDOUT BY THE TOPNOTCH TEAM
For inquiries visit www.topnotchboardprep.com.ph or https://www.facebook.com/topnotchmedicalboardprep/

11. Which of the following statements about nerve growth ANSWER: B


factor is NOT true?
A. It is made up of three polypeptide subunits • “Nerve Growth Factor is made up of three polypeptide subunits. It is
B. It facilitates the process of apoptosis necessary for the growth and development of the sympathetic nervous
C. It is necessary for the growth and development of the system; and cholinergic neurons in the basal forebrain and the
sympathetic nervous system striatum. There is evidence that the maintenance of neurons by NGF is
D. It is picked up by nerves from the organs they due to a reduction in apoptosis. It is picked up by nerves from the
innervate organs they innervate.”
Ganong. Review of Medical Physiology 23rd ed. Chapter 4, page 90 .
E. It is present in the brain

(CHAPTER 5) EXCITABLE TISSUE: MUSCLE


1. The action potential of skeletal muscle: ANSWER: B
A. has a prolonged plateau phase. • Action potential is essential for contraction which spreads inward to all
B. spreads inward to all parts of the muscle via the T parts of the muscle via the T- tubules; it triggers the release of Ca2+
tubules. from the terminal cisterns, the lateral sacs of the sarcoplasmic reticulum
C. causes the immediate uptake of Ca2+ into the lateral next to the T system; it is shorter than the action potential of cardiac
sacs of the sarcoplasmic reticulum. muscle (which has prolonged plateau phase).
D. is longer than the action potential of cardiac muscle.
E. is not essential for contraction. Ganong. Review of Medical Physiology 23rd ed. Chapter 5 , page 100 .
ANSWER: D
2. The functions of tropomyosin in skeletal muscle include:
A. sliding on actin to produce shortening. • Tropomyosin: acting as a “relaxing protein” at rest by covering up the
B. releasing Ca2+ after initiation of contraction. sites where myosin binds to actin.
C. binding to myosin during contraction. • Myosin: sliding on actin to produce shortening.
D. acting as a “relaxing protein” at rest by covering up • Sarcoplasmic reticulum: releasing Ca2+ after initiation of contraction.
the sites where myosin binds to actin. Actin: binding to myosin during contraction.
E. generating ATP, which it passes to the contractile • Phosphocreatine: generating ATP, which it passes to the contractile
mechanism. mechanism.
Ganong. Review of Medical Physiology 23rd ed. Chapter 5; pages 96, 102 .
ANSWER: B
• Tropomyosin: acting as a “relaxing protein” at rest by covering up the
3. The cross-bridges of the sarcomere in skeletal muscle are
sites where myosin binds to actin.
made up of
A. actin. • Myosin: sliding on actin to produce shortening/ cross-bridging.
B. myosin. • Actin: binding to myosin head during contraction.
C. troponin. • Troponin T binds the troponin components to tropomyosin;
D. tropomyosin. • Troponin I inhibits the interaction of myosin with actin; and
E. myelin. • Troponin C contains the binding sites for the Ca2+ that helps to initiate
contraction. Myelin covers the nerve fibers.
Ganong. Review of Medical Physiology 23rd ed. Chapter 2, page 51..
ANSWER: C
4. The contractile response in skeletal muscle • Muscle fiber does not have a refractory period, repeated stimulation
A. starts after the action potential is over. before relaxation produces additional activation of the contractile
B. does not last as long as the action potential. elements, or summation of contractions.
C. produces more tension when the muscle contracts • Note that because work is the product of force times distance, isotonic
isometrically than when the muscle contracts contractions do work, whereas isometric contractions do not.
isotonically. • When the muscle fiber contracts isometrically, the tension developed is
D. produces more work when the muscle contracts proportional to the number of cross-bridges between the actin and the
isometrically than when the muscle contracts myosin molecules
isotonically. • Contraction starts before the action potential is over and last longer as
E. decreases in magnitude with repeated stimulation. the action potential.
Ganong. Review of Medical Physiology 23rd ed. Chapter 5 , page 101 .
5. Gap junctions ANSWER: C
A. are absent in cardiac muscle.
B. are present but of little functional importance in cardiac • “Gap junctions are present in cardiac muscles and provide the pathway
muscle. for rapid spread of excitation from one cardiac muscle fiber to another.
C. are present and provide the pathway for rapid spread of It is also present in smooth muscle, which is responsible for its unitary
excitation from one cardiac muscle fiber to another. movement; on the other hand, it is absent in skeletal muscle cells.
D. are absent in smooth muscle.
E. connect the sarcotubular system to individual skeletal Ganong. Review of Medical Physiology 23rd ed. Chapter 5; pages 106,109 .
muscle cells

TOPNOTCH MEDICAL BOARD PREP MED PHYSIOLOGY GANONG SUPPLEMENT HANDOUT BY THE TOPNOTCH TEAM Page 6 of 46
For inquiries visit www.topnotchboardprep.com.ph or https://web.facebook.com/topnotchmedicalboardprep/
TOPNOTCH MEDICAL BOARD PREP PHYSIOLOGY GANONG SUPPLEMENT HANDOUT BY THE TOPNOTCH TEAM
For inquiries visit www.topnotchboardprep.com.ph or https://www.facebook.com/topnotchmedicalboardprep/
(CHAPTER 6) SYNAPTIC & JUNCTIONAL TRANSMISSION
1. Which of the following electrophysiologic events is correctly ANSWER: C
paired with the change in ionic currents causing the event?
• Endplate potential : increase in Na+ conductance .
A. Fast inhibitory postsynaptic potentials (IPSPs) and closing
of Cl– channels • Fast IPSPs (inhibitory postsynaptic potentials): opening of Cl–
B. Fast excitatory postsynaptic potentials (EPSPs) and an channels
increase in Ca2+ conductance • Fast EPSPs (excitatory postsynaptic potentials): an increase in
C. Endplate potential and an increase in Na+ conductance Na+ conductance
D. Presynaptic inhibition and closure of voltage-gated K+ • Presynaptic inhibition: opening of voltage-gated K+ channels
channels • Slow EPSPs: decrease in K+ conductance
E. Slow EPSPs and an increase in K+ conductance Ganong. Review of Medical Physiology 23rd ed. Chapter6; page 120, 128.

2. Which of the following physiologic processes is not correctly ANSWER: D


paired with a structure? • “The acetylcholine diffuses to the muscle-type nicotinic
A. Electrical transmission : gap junction acetylcholine receptors, which are concentrated at the tops of the
B. Negative feedback inhibition : Renshaw cell junctional folds of the membrane of the motor end plate. Binding of
C. Synaptic vesicle docking and fusion : presynaptic nerve acetylcholine to these receptors increases the Na+ and K+
terminal conductance of the membrane, and the resultant influx of Na+
D. Endplate potential : muscarinic cholinergic receptor produces a depolarizing potential, the end plate potential.”
E. Action potential generation : initial segment Ganong. Review of Medical Physiology 23rd ed. Chapter 6, page 124 ..
ANSWER: E
3. Initiation of an action potential in skeletal muscle • Temporal and spatial summation (not facilitation ) are factors
A. requires spatial facilitation. which causes excitatory post synaptic potential (EPSP) that is
B. requires temporal facilitation. different from action potential, because it is not an all-or-none
C. is inhibited by a high concentration of Ca2+ at the response. Initiation of action potential in skeletal muscle is caused
neuromuscular junction. by the release of acetylcholine (not norepinephrine) from the
D. requires the release of norepinephrine. presynaptic vesicles into the neuromuscular junction which
E. requires the release of acetylcholine. eventually binds with nicotinic cholinergic receptor.
Ganong. Review of Medical Physiology 23rd ed. Chapter 6, page 120, 124
4. A 35-year-old woman sees her physician to report muscle ANSWER: B
weakness in the extraocular eye muscles and muscles of the
extremities. She states that she feels fine when she gets up in • Myasthenia gravis is caused by the formation of circulating
the morning, but the weakness begins soon after she becomes antibodies to the muscle type of nicotinic acetylcholine receptors
active. The weakness is improved by rest. Sensation appears this leads to the major clinical feature of the disease–muscle
normal. The physician treats her with an anticholinesterase fatigue with sustained or repeated activity.
inhibitor, and she notes immediate return of muscle strength. • In contrast, Lambert–Eaton Syndrome which is a condition
Her physician diagnoses her with wherein muscle weakness is caused by an autoimmune attack
A. Lambert–Eaton syndrome. against one of the Ca2+ channels in the nerve at the neuromuscular
B. myasthenia gravis. junction, improves with repetitive stimulation.
C. multiple sclerosis. • Among the choices, only myasthenia will have clinical improvement
D. Parkinson disease. after administration of an anticholinesterase inhibitor.
Ganong. Review of Medical Physiology 23rd ed. Chapter 6, page 126.
E. muscular dystrophy.
5. A 55-year-old woman had an autonomic neuropathy that ANSWER: D
disrupted the sympathetic nerve supply to the pupillary dilator
muscle of her right eye. While having her eyes examined, the
ophthalmologist placed phenylephrine in her eyes. The right
eye became much more dilated than the left eye. This suggests
that • A good example of denervation supersensitivity is the response of
A. the sympathetic nerve to the right eye had regenerated. the denervated iris.
B. the parasympathetic nerve supply to the right eye • Hypersensitivity of the postsynaptic structure to the transmitter
remained intact and compensated for the loss of the previously secreted by the axon endings is a general phenomenon,
sympathetic nerve. largely due to the synthesis or activation of more receptors.
C. phenylephrine blocked the pupillary constrictor muscle of
Ganong. Review of Medical Physiology 23rd ed. Chapter 6, page 126.
the right eye.
D. denervation supersensitivity had developed.
E. the left eye also had nerve damage and so was not
responding as expected.
6. A 47-year-old woman was admitted to the hospital after ANSWER: E
experiencing nausea and vomiting for about 2 days followed
by severe muscle weakness and neurologic symptoms,
including ptosis and dysphagia. She indicated she had eaten at
a restaurant the evening before the symptoms began.
Laboratory tests were positive for Clostridium botulinum.
Neurotoxins
A. block the reuptake of neurotransmitters into presynaptic • Clinically, tetanus toxin causes spastic paralysis by blocking
terminals. presynaptic transmitter release in the CNS, and botulism causes
B. such as tetanus toxin bind reversibly to the presynaptic flaccid paralysis by blocking the release of acetylcholine at the
membrane at the neuromuscular junction. neuromuscular junction.
C. reach the cell body of the motor neuron by diffusion into
the spinal cord. Ganong. Review of Medical Physiology 23rd ed. Chapter 6, page 119 .
D. exert all of their adverse effects by acting centrally rather
than peripherally.
E. such as botulinum toxin prevent the release of
acetylcholine from motor neurons due to cleavage of either
synaptosome-associated proteins or vesicle-associated
membrane proteins.

TOPNOTCH MEDICAL BOARD PREP MED PHYSIOLOGY GANONG SUPPLEMENT HANDOUT BY THE TOPNOTCH TEAM Page 7 of 46
For inquiries visit www.topnotchboardprep.com.ph or https://web.facebook.com/topnotchmedicalboardprep/
TOPNOTCH MEDICAL BOARD PREP PHYSIOLOGY GANONG SUPPLEMENT HANDOUT BY THE TOPNOTCH TEAM
For inquiries visit www.topnotchboardprep.com.ph or https://www.facebook.com/topnotchmedicalboardprep/
ANSWER: E
7. Fast inhibitory postsynaptic potentials (IPSPs):
A. are a consequence of decreased Cl- conductance • Fast inhibitory postsynaptic potentials (IPSPs) can be produced
B. occur in skeletal muscle by a localized increase in Cl- transport and by opening of K+
C. can be produced by an increase in Na+ conductance channels, with movement of K+ out of the postsynaptic cell, or by
D. can be produced by a decrease in Ca2+ conductance closure of Na+ or Ca2+ channels. It occurs in the motor neuron
E. interact with other fast and slow potentials to move the presynaptic ending and interact with other fast and slow
membrane potential of the postsynaptic neuron toward or potentials to move the membrane potential of the postsynaptic
away from the firing level neuron toward or away from the firing level.
Ganong. Review of Medical Physiology 23rd ed. Chapter 6, page 119, 120.
ANSWER: C
8. Fast excitatory postsynaptic potentials (EPSPs):
A. are a consequence of decreased Cl- conductance • The EPSP is produced by depolarization of the postsynaptic cell
B. occur in skeletal muscle membrane immediately under the presynaptic ending. The
C. can be produced by an increase in Na+ conductance excitatory transmitter opens Na+ or Ca2+ ion channels in the
D. can be produced by a decrease in Ca2+ conductance postsynaptic membrane, producing an inward current.
E. All of the above Ganong. Review of Medical Physiology 23rd ed. Chapter 6, page 120.

(CHAPTER 7) NEUROTRANSMITTERS & NEUROMODULATORS


1. Which of the following statements about neurotransmitters is ANSWER: D
true?
A. All neurotransmitters are derived from amino acid • “Small-molecule transmitters include monoamines (eg,
precursors. acetylcholine, serotonin, histamine), catecholamines (dopamine,
B. Small-molecule neurotransmitters include dopamine, norepinephrine, and epinephrine), and amino acids (eg, glutamate,
histamine, ATP, glycine, enkephalin, and norepinephrine. GABA, glycine). Large-molecule transmitters include a large
C. Large-molecule transmitters include ATP, cannabinoids, number of peptides called neuropeptides including substance P,
substance P, and vasopressin. enkephalin, vasopressin.”
D. Norepinephrine can act as a neurotransmitter in the • Norepinephrine can act as a neurotransmitter in the periphery and
periphery and a neuromodulator in the CNS. a neuromodulator in the CNS.
Ganong. Review of Medical Physiology 23rd ed. Chapter 7, page 130 .
E. Nitrous oxide is a neurotransmitter in the CNS.
ANSWER: A
2. Which of the following statements is not true?
A. Neuronal glutamate is synthesized in glia by the enzymatic
conversion from glutamine and then diffuses into the
neuronal terminal where it is sequestered into vesicles
until released by an influx of Ca2+ into the cytoplasm after
an action potential reaches the nerve terminal.
B. After release of serotonin into the synaptic cleft, its actions
are terminated by reuptake into the presynaptic nerve • “Glutamate is formed by reductive amination of the Krebs cycle
terminal, an action that can be blocked by tricyclic intermediate a-ketoglutarate in the cytoplasm. Glutamate released
antidepressants. into the synaptic cleft is taken up by a Na+-dependent glutamate
C. Norepinephrine is the only small-molecule transmitter transporter, and in the astrocyte it is converted to glutamine. The
that is synthesized in synaptic vesicles instead of being glutamine enters the neuron and is converted to glutamate.”
transported into the vesicle after its synthesis.
Ganong. Review of Medical Physiology 23rd ed. Chapter 7, page 140 .
D. Each nicotinic cholinergic receptor is made up of five
subunits that form a central channel that, when the receptor
is activated, permits the passage of Na+ and other cations.
E. GABA transaminase converts glutamate to GABA; the
vesicular GABA transporter transports both GABA and
glycine into synaptic vesicles.
ANSWER: C
3. Which of the following receptors is correctly identified as an • GABA A receptor: ionotropic (chloride)
ionotropic or a G-protein–coupled receptor (GPCR)? • GABA B receptor: metabotropic/GPCR
A. Neurokinin receptor: ionotropic
• Neurokinin receptor: metabotropic/GPCR
B. Nicotinic receptor: GPCR
• Nicotinic receptor: ionotropic (sodium/potassium)
C. GABA A receptor: ionotropic
• NMDA receptor: ionotropic (calcium)
D. NMDA receptor: GPCR
E. Glycine: GPCR • Glycine: ionotropic (chloride)
Ganong. Review of Medical Physiology 23rd ed. Chap 2 , pg 54; chap 7, pg 141.
ANSWER: D
• Opioid receptors (δ, μ, κ) differ in physiologic effects, all three are G
4. A 27-year-old man was brought to the emergency department protein coupled receptors, and all inhibit adenylyl cyclase.
with symptoms of opioid intoxication. He was given an • Morphine is not an endogenous opioid.
intravenous dose of naloxone. Endogenous opioids
A. bind to both ionotropic receptors and GPCR.
B. include morphine, endorphins, and dynorphins.
C. show the following order of affinity for δ receptors:
dynorphins > > endorphins.
D. show the following order of affinity for μ receptors:
dynorphins > endorphins.
E. show the following order of affinity for κ receptors:
endorphins > > enkephalins.

Ganong. Review of Medical Physiology 23rd ed. Chapter 7 , page 144.

TOPNOTCH MEDICAL BOARD PREP MED PHYSIOLOGY GANONG SUPPLEMENT HANDOUT BY THE TOPNOTCH TEAM Page 8 of 46
For inquiries visit www.topnotchboardprep.com.ph or https://web.facebook.com/topnotchmedicalboardprep/
TOPNOTCH MEDICAL BOARD PREP PHYSIOLOGY GANONG SUPPLEMENT HANDOUT BY THE TOPNOTCH TEAM
For inquiries visit www.topnotchboardprep.com.ph or https://www.facebook.com/topnotchmedicalboardprep/

5. A 38-year-old woman was referred to a psychiatrist after ANSWER: B


telling her primary care physician that she had difficulty
sleeping (awakening at 4 AM frequently for the past few
months) and a lack of appetite causing a weight loss of over 20 • The case demonstrated a patient experiencing signs and symptoms
lb. She also said she no longer enjoyed going out with her of depression.
friends or doing volunteer service for underprivileged • “Focus in treating clinical depression has shifted from
children. What type of drug is her doctor most likely to suggest norepinephrine to serotonin. It is known that the primary serotonin
as an initial step in her therapy? metabolite 5-HIAA is low in CSF of depressed individuals. Drugs
A. A serotonergic receptor antagonist such as fluoxetine (Prozac), which inhibit serotonin reuptake
B. An inhibitor of neuronal uptake of serotonin without affecting norepinephrine reuptake, are effective as
C. An inhibitor of monoamine oxidase antidepressants.”
D. An amphetamine-like drug
Ganong. Review of Medical Physiology 23rd ed. Chapter 7, page 137.
E. A drug that causes an increase in both serotonin and
dopamine
6. A 55-year-old woman had been receiving long-term treatment ANSWER: E
with phenelzine for her depression. After she consumed
Chianti wine, aged cheddar cheese, processed meats, and dried
fruits one night at a party, the following symptoms developed: • Patients taking MAO inhibitors should avoid tyramine-containing
a severe headache, chest pain, rapid heartbeat, enlarged foods (aged cheese, cured meats, and pickled food). Tyramine’s
pupils, increased sensitivity to light, and nausea. What is the spectrum of action is similar to that of norepinephrine. In patients
most likely cause of these symptoms? treated with MAO inhibitors—particularly inhibitors of the MAO-A
A. The foods were contaminated with botulinum toxin. isoform—this effect of tyramine may be greatly intensified, leading
B. She had a myocardial infarction. to marked increases in blood pressure. This occurs because of
C. She experienced a migraine headache. increased bioavailability of tyramine and increased neuronal stores
D. She had an adverse reaction to the mixture of alcohol with of catecholamines.
her antidepressant.
E. She had a hypertensive crisis from eating foods high in Katzung. Basic and Clinical Pharmaclogy. 13th edition.page 199
tyramine while taking a monoamine oxidase inhibitor for
her depression.
7. Which of the following is a ligand-gated ion channel? ANSWER: C
A. VIP receptor • GABA A receptor: ionotropic (chloride)
B. norepinephrine receptor • GABA B, VIP, norepinephrine, metamorphic glutamate
C. GABA(A) receptor receptor: metabotropic/GPCR
D. GABA(B) receptor
E. Metamorphic glutamate receptor Ganong. Review of Medical Physiology 23rd ed. Chapter 7 , page 139, 145, 141.
ANSWER: D
8. Which of the following synaptic transmitters is NOT a peptide,
• “Small-molecule transmitters include monoamines (eg,
polypeptide, or protein?
acetylcholine, serotonin, histamine), catecholamines (dopamine,
A. substance P
norepinephrine, and epinephrine), and amino acids (eg, glutamate,
B. met-enkephalin
GABA, glycine). Large-molecule transmitters include a large
C. Beta-endorphin
number of peptides called neuropeptides including substance P,
D. serotonin
enkephalin, vasopressin.”
E. dynorphin
Ganong. Review of Medical Physiology 23rd ed. Chapter 7, page 130 .
9. Activation of which of the following receptors would be ANSWER: C
expected to decrease anxiety? • The increase in Cl– conductance produced by GABA A receptors is
A. nicotinic cholinergic receptors potentiated by benzodiazepines, drugs that have marked anti-
B. glutamate receptors
anxiety activity and are also effective muscle relaxants,
C. GABA (A) receptors
anticonvulsants, and sedatives.
D. glucocorticoid receptors
E. alpha-1 adrenergic receptors Ganong. Review of Medical Physiology 23rd ed. Chapter 7, page 142.
10. Which of the following receptors is coupled to a ANSWER: B
heterotrimeric G protein? • GABA B receptor: metabotropic/GPCR
A. glycine receptor • GABA A receptor: ionotropic (chloride)
B. GABA (B) receptor • Nicotinic receptor: ionotropic (sodium/potassium)
C. nicotinic acetylcholine receptor at myoneural junction • Glycine receptor: ionotropic (chloride)
D. 5-HT3 receptor • ANP and 5HT3 receptors: ionotropic
E. ANP receptor Ganong. Review of Medical Physiology 23rd ed. Chap2 , pg 54; chap7, pg 141.
ANSWER: A
11. Which of the following would NOT be expected:
A. A drug that increases the entry of arginine into neurons • The following factors increases adrenergic transmission: increase in
B. A drug that enhances tyrosine hydroxylase activity. the entry of tyrosine (not arginine) into neurons; enhancement of
C. A drug that enhances dopamine beta-hydroxylase activity tyrosine hydroxylase activity, dopamine beta-hydroxylase
D. A drug that inhibits monoamine oxidase activity; inhibition of monoamine oxidase and norepinephrine
E. A drug that inhibits norepinephrine reuptake. reuptake.
Ganong. Review of Medical Physiology 23rd ed. Chapter 7 , page 138.

TOPNOTCH MEDICAL BOARD PREP MED PHYSIOLOGY GANONG SUPPLEMENT HANDOUT BY THE TOPNOTCH TEAM Page 9 of 46
For inquiries visit www.topnotchboardprep.com.ph or https://web.facebook.com/topnotchmedicalboardprep/
TOPNOTCH MEDICAL BOARD PREP PHYSIOLOGY GANONG SUPPLEMENT HANDOUT BY THE TOPNOTCH TEAM
For inquiries visit www.topnotchboardprep.com.ph or https://www.facebook.com/topnotchmedicalboardprep/

SECTION 2: CENTRAL AND PERIPHERAL NEUROPHYSIOLOGY


(CHAPTER 8) SOMATOSENSORY NEUROTRANSMISSION: TOUCH, PAIN, & TEMPERATURE
ANSWER: D
1. A 28-year-old man was seen by a neurologist because he had • “Meissner corpuscles are dendrites encapsulated in connective
experienced prolonged episodes of tingling and numbness in tissue and respond to changes in texture and slow vibrations.”
his right arm. He underwent a neurologic exam to evaluate his • “Merkel cells are expanded dendritic endings, and they respond to
sensory nervous system. Which of the following receptors is sustained pressure and touch.”
correctly paired with the type of stimulus to which it is most • “Ruffini corpuscles are enlarged dendritic endings with elongated
apt to respond? capsules, and they respond to sustained pressure.”
A. Pacinian corpuscle and motion. • “Pacinian corpuscles consist of unmyelinated dendritic endings of
B. Meissner corpuscle and deep pressure. a sensory nerve fiber, encapsulated by concentric lamellae of
C. Merkel cells and warmth. connective tissue that give the organ the appearance of a cocktail
D. Ruffini corpuscles and sustained pressure. onion. Theses receptors respond to deep pressure and fast
E. Muscle spindle and tension. vibration.”
Ganong. Review of Medical Physiology 23rd ed. Chapter 8, page 150 .
ANSWER: A
2. Nociceptors • Nociceptors are activated by strong pressure, severe cold, severe
A. are activated by strong pressure, severe cold, severe heat, heat, and chemicals. Visceral organs are also innervated by
and chemicals. nociceptor and is triggred by organ distention. They are not
B. are absent in visceral organs. specialized structure, they are unmyelinated dendrites of sensory
C. are specialized structures located in the skin and joints. neurons located around hair follicles throughout the glabrous and
D. are innervated by group II afferents. hairy skin as well as deep tissue. They are innervated by Aδ fibers
E. are involved in acute but not chronic pain. and C fibers and are involved in acute and chronic pain.
Ganong. Review of Medical Physiology 23rd ed. Chapter 10 , page 167.
3. A generator potential ANSWER: B
A. always leads to an action potential.
B. increases in amplitude as a more intense stimulus is • Generator or receptor potential resembles EPSP which increases
applied. in amplitude as a more intense stimulus is applied. It is not an all-
C. is an all-or-none phenomenon. or-none phenomenon and does not always leads to an action
D. is unchanged when a given stimulus is applied repeatedly potential.
over time.
Ganong. Review of Medical Physiology 23rd ed. Chapter 8 , page 151 .
E. All of the above.
ANSWER: A
4. Sensory systems code for the following attributes of a • “ All sensory systems code for four elementary attributes of a
stimulus: stimulus: modality, location, intensity, and duration.”
A. modality, location, intensity, and duration. • “Modality is the type of energy transmitted by the stimulus.
B. threshold, receptive field, adaptation, and discrimination. Location is the site on the body or space where the stimulus
C. touch, taste, hearing, and smell. originated. Intensity is signaled by the response amplitude or
D. threshold, laterality, sensation, and duration. frequency of action potential generation. Duration refers to the
E. sensitization, discrimination, energy, and projection. time from start to end of a response in the receptor.”
Ganong. Review of Medical Physiology 23rd ed. Chapter 8 , page 152 .
ANSWER: D
5. Which of the following are correctly paired? • Nociceptive pain is caused by nere damage.
A. Neuropathic pain and withdrawal reflex • “Painful stimulus causes a “bright,” sharp, localized sensation (fast
B. First pain and dull, intense, diffuse, and unpleasant feeling pain) followed by a dull, intense, diffuse, and unpleasant feeling
C. Physiologic pain and allodynia (slow pain). Pain is frequently classified as physiologic or acute
D. Second pain and C fibers pain an pathologic or chronic pain, which includes inflammatory
E. Nociceptive pain and nerve damage pain and neuropathic pain.”
Ganong. Review of Medical Physiology 23rd ed. Chapter 10 , page 168 .
6. A 32-year-old woman experienced the sudden onset of a ANSWER: C
severe cramping pain in the abdominal region. She also
became nauseated. Visceral pain
A. shows relatively rapid adaptation. • Visceral pain is is poorly localized, and often referred, causing
B. is mediated by B fibers in the dorsal roots of the spinal contraction or spasm of nearby skeletal muscles. It shows slow
nerves. adaptation mediated by C fibers in the dorsal roots of the spinal
C. is poorly localized. nerves.
D. resembles “fast pain” produced by noxious stimulation of
Ganong. Review of Medical Physiology 23rd ed. Chapter 10 , page 169,170 .
the skin.
E. causes relaxation of nearby skeletal muscles.
7. A ventrolateral cordotomy is performed that produces relief of ANSWER: B
pain in the right leg. It is effective because it interrupts the • The axons from the neurons responsible for nociception crosses the
A. left dorsal column. midline and ascend in the ventrolateral quadrant of the spinal cord,
B. left ventrolateral spinothalamic tract. where they form the ventrolateral spinothalamic tract; therefore,
C. right ventrolateral spinothalamic tract. ventrolateral cordotomy will result in loss of pain sensation in the
D. right medial lemniscal pathway. contralateral side.
E. a direct projection to the primary somatosensory cortex. Ganong. Review of Medical Physiology 23rd ed. Chapter 11 , page 175.
ANSWER: B
8. Which of the following CNS regions is not correctly paired with
• Nucleus raphe magnus and serotonin
a neurotransmitter or a chemical involved in pain modulation?
A. Periaqueductal gray matter and morphine • Locus Coeruleus and norepinephrine
B. Nucleus raphe magnus and norepinephrine • Periaqueductal gray matter and morphine
C. Spinal dorsal horn and enkephalin • Spinal dorsal horn and enkephalin
D. Dorsal root ganglion and opioids • Dorsal root ganglion and opioids
E. Spinal dorsal horn and serotonin • Spinal dorsal horn and serotonin
Ganong. Review of Medical Physiology 23rd ed. Chap11 , pg177; Chap15, pg 238

TOPNOTCH MEDICAL BOARD PREP MED PHYSIOLOGY GANONG SUPPLEMENT HANDOUT BY THE TOPNOTCH TEAM Page 10 of 46
For inquiries visit www.topnotchboardprep.com.ph or https://web.facebook.com/topnotchmedicalboardprep/
TOPNOTCH MEDICAL BOARD PREP PHYSIOLOGY GANONG SUPPLEMENT HANDOUT BY THE TOPNOTCH TEAM
For inquiries visit www.topnotchboardprep.com.ph or https://www.facebook.com/topnotchmedicalboardprep/

9. A 47-year-old woman experienced migraine headaches that were ANSWER: A


not relieved by her current pain medications. Her doctor
prescribed one of the newer analgesic agents that exert their
effects by targeting synaptic transmission in nociception and • Topiramate and voltage-gated Na+ channels
peripheral sensory transduction. Which of the following drugs is • Ziconotide and voltage-gated N-type calcium channels
correctly paired with the type of receptor it acts on to exert its • Naloxone and opioid receptors (partial antagonist)
antinociceptive effects? • Lidocaine and sodium channels
A. Topiramate and Na+ channel • Gabapentin and VG-Ca2+ channels
B. Ziconotide and NMDA receptors
C. Naloxone and opioid receptors Katzung. Basic and Clinical Pharmaclogy. 13th edition. pgs 329,540,576,730,
741,
D. Lidocaine and TRPVI channels
E. Gabapentin and Nav1.8
10. A 40-year-old man loses his right hand in a farm accident. Four ANSWER: E
years later, he has episodes of severe pain in the missing hand
(phantom limb pain). A detailed PET scan study of his cerebral
cortex might be expected to show
A. expansion of the right hand area in his right primary • “Numerous animal studies point to dramatic reorganization of
somatosensory cortex. cortical structures. If a digit is amputated in a monkey, the
B. expansion of the right hand area in his left primary cortical representation of the neighboring digits spreads into
somatosensory cortex. the cortical area that was formerly occupied by the
C. a metabolically inactive spot where his hand area in his left representationof the amputated digit.”
primary somatosensory cortex would normally be.
D. projection of fibers from neighboring sensory areas into the Ganong. Review of Medical Physiology 23rd ed. Chapter 11 page 176.
right hand area of his right primary somatosensory cortex.
E. projection of fibers from neighboring sensory areas into the
right hand area of his left primary somatosensory cortex
ANSWER: D
• “The dorsal column–medial lemniscal system carries signals
upward to the medulla of the brain and cross to the opposite
side in the medulla, they continue through the brain stem to the
thalamus by way of the medial lemniscus.”
• “Conversely, signals in the anterolateral system, immediately
11. A 50-year-old woman undergoes a neurologic exam that after entering the spinal cord, synapse in the dorsal horns of the
indicates loss of pain and temperature sensitivity, vibratory sense, spinal gray matter, then cross to the opposite side of the cord
and proprioception in the left leg. These symptoms could be and ascend through the anterior and lateral white columns of
explained by the cord. They terminate at all levels of the lower brain stem
A. a tumor on the right medial lemniscal pathway in the sacral and in the thalamus.”
spinal cord. • “Thalamic sensory relay area, the third-order nerve fibers
B. a peripheral neuropathy. project mainly to the postcentral gyrus of the cerebral cortex,
C. a tumor on the left medial lemniscal pathway in the sacral which is called somatic sensory area I. “
spinal cord.
D. a tumor affecting the right posterior paracentral gyrus.
E. a large tumor in the right lumbar ventrolateral spinal cord.

Guyton.Textbook of Medical Physiology 11th ed.Chapter 47 , page 587, 588 .


ANSWER: C
12. The distance by which two touch stimuli must be separated to be • “Two point tactile discrimination on the tips of the fingers, a
perceived as two separate stimuli is greatest on the person can distinguish two separate points even when the
A. lips needles are as close together as 1 to 2 millimeters. However,
B. palm of the hand on the person’s back, the needles must usually be as far apart as
C. back of the scapula 30 to 70 millimeters before two separate points can be
D. dorsum of the hand detected. The reason for this difference is the different numbers
E. tips of the fingers of specialized tactile receptors in the two areas.”
Guyton.Textbook of Medical Physiology 11th ed.Chapter 47 , page 592, 593.
13. Visceral pain ANSWER: C
A. shows relatively rapid adaptation
B. is mediated by B fibers in the dorsal roots of the spinal nerves • Visceral pain is is poorly localized, and often referred, causing
C. is poorly localized contraction or spasm of nearby skeletal muscles. It shows slow
D. resembles “fast pain” produced by noxious stimulation of the adaptation mediated by C fibers in the dorsal roots of the spinal
skin nerves.
Ganong. Review of Medical Physiology 23rd ed. Chapter 10 , page 169,170 .
E. causes relaxation of nearby skeletal muscles
ANSWER: D
• Nociceptors and thermoreceptors are free nerve endings on
14. Thermoreceptors unmyelinated or lightly myelinated fibers in hairy and
A. are activated only by severe cold or severe heat glaborous skin and deep tissues.
B. are located on superficial layers of the skin • They are innervated by Aδ fibers and C fibers.
C. are a subtype of nociceptors • Thermoreceptor is not a subtype of nociceptor and unlike
D. are on dendritic endings of Aδ fibers and C fibers nociceptor which is activated only in severe temperature
E. All of the above condition, thermoreceptor threshold for hot and cold
temperature is 30 and 24 degrees celsius respectively
Ganong. Review of Medical Physiology 23rd ed. Chapter 10 , page 167, 168 .

TOPNOTCH MEDICAL BOARD PREP MED PHYSIOLOGY GANONG SUPPLEMENT HANDOUT BY THE TOPNOTCH TEAM Page 11 of 46
For inquiries visit www.topnotchboardprep.com.ph or https://web.facebook.com/topnotchmedicalboardprep/
TOPNOTCH MEDICAL BOARD PREP PHYSIOLOGY GANONG SUPPLEMENT HANDOUT BY THE TOPNOTCH TEAM
For inquiries visit www.topnotchboardprep.com.ph or https://www.facebook.com/topnotchmedicalboardprep/
(CHAPTER 9) VISION
ANSWER: D
1. A visual exam in an 80-year-old man shows he has a • Central scotoma: blind spots in the visual field
reduced ability to see objects in the upper and lower • Heteronymous hemianopia with macular sparing: reduced ability to
quadrants of the left visual fields of both eyes but some see objects in the upper and lower quadrants of opposite sides of the
vision remains in the central regions of the visual field. visual fields but some vision remains in the central regions of the visual
The diagnosis is field
A. central scotoma. • Lesion of the optic chiasm: bitemporal visual loss.
B. heteronymous hemianopia with macular sparing. • Homonymous hemianopia with macular sparing: reduced ability to
C. lesion of the optic chiasm. see objects in the upper and lower quadrants of the left visual fields of
D. homonymous hemianopia with macular sparing. both eyes but some vision remains in the central regions of the visual field
E. retinopathy. • Retinopathy: scarring of the retina that can lead to total loss of vision.
Ganong. Review of Medical Physiology 23rd ed. Chapter 12 , page 198 .
ANSWER: D
2. A 45-year-old woman who had never needed to wear
glasses experienced difficulty reading a menu in a dimly-
lit restaurant. She then recalled that as of late she needed
to have the newspaper closer to her eyes in order to read • Accommodation is when the gaze is directed at a near object, the ciliary
it. A friend recommended she purchase reading glasses. muscle contracts. This decreases the distance between the edges of the
Visual accommodation involves ciliary body and relaxes the lens ligaments, so that the lens springs into a
A. increased tension on the lens ligaments. more convex shape.
B. a decrease in the curvature of the lens.
C. relaxation of the sphincter muscle of the iris. Ganong. Review of Medical Physiology 23rd ed. Chapter 12, page 189
D. contraction of the ciliary muscle.
E. increased intraocular pressure.
ANSWER: C
3. A 28-year-old man with severe myopia made an
appointment to his ophthalmologist when he began to
notice flashing lights and floaters in his visual field. He • “The retina extends anteriorly almost to the ciliary body. It is organized in
was diagnosed with a retinal detachment. The retina 10 layers and contains the rods and cones, which are the visual
A. is epithelial tissue that contains photoreceptors. receptors, plus four types of neurons: bipolar cells, ganglion cells,
B. lines the anterior one-third of the choroid. horizontal cells, and amacrine cells.”
C. has an inner nuclear layer that contains bipolar cells, • “It contains ganglion cells whose axons form the optic nerve. The area of
horizontal cells, andsee amacrine cells. the optic disk contains no visual receptors over the disk, and
D. contains ganglion cells whose axons form the consequently this spot is blind (the blind spot).”
oculomotor nerve.
Ganong. Review of Medical Physiology 23rd ed. Chapter 12 , page 182..
E. contains an optic disk where visual acuity is greatest
ANSWER: B
4. A 62-year-old white woman experienced a rapid onset of
blurry vision along with loss of central vision. A • “Near the posterior pole of the eye is a yellowish pigmented spot, the
comprehensive eye exam showed that she had wet age- macula lutea. This marks the location of the fovea centralis, a thinned-
related macular degeneration. The fovea of the eye out, rod-free portion of the retina that is present in humans and other
A. has the lowest light threshold. primates. In it, the cones are densely packed, and each synapses to a
B. is the region of highest visual acuity. single bipolar cell, which, in turn, synapses on a single ganglion cell,
C. contains only red and green cones. providing a direct pathway to the brain. Consequently, the fovea is the
D. contains only rods. point where visual acuity is greatest.”
E. is situated over the head of the optic nerve. Ganong. Review of Medical Physiology 23rd ed. Chapter 12 , page 183,184.

ANSWER: E
• In the extrafoveal portions of the retina, rods predominate (Figure 12-7),
and there is a good deal of convergence.

5. Which of the following parts of the eye has the greatest


concentration of rods?
A. Ciliary body
B. Iris
C. Optic disk
D. Fovea
E. Parafoveal region

Ganong. Review of Medical Physiology 23rd ed. Chapter 12 , page 185, 186 .

6. Which of the following is not correctly paired? ANSWER: C


A. Rhodopsin: retinal and opsin
B. Obstruction of the canal of Schlemm: elevated • “Myopia is a defect that can be corrected by glasses with biconcave
intraocular pressure lenses, which make parallel light rays diverge slightly before they strike
C. Myopia: convex lenses the eye.”
D. Astigmatism: nonuniform curvature of the cornea
E. Inner segments of rods and cones: synthesis of the Ganong. Review of Medical Physiology 23rd ed. Chapter 12, page 188.
photosensitive compounds

TOPNOTCH MEDICAL BOARD PREP MED PHYSIOLOGY GANONG SUPPLEMENT HANDOUT BY THE TOPNOTCH TEAM Page 12 of 46
For inquiries visit www.topnotchboardprep.com.ph or https://web.facebook.com/topnotchmedicalboardprep/
TOPNOTCH MEDICAL BOARD PREP PHYSIOLOGY GANONG SUPPLEMENT HANDOUT BY THE TOPNOTCH TEAM
For inquiries visit www.topnotchboardprep.com.ph or https://www.facebook.com/topnotchmedicalboardprep/
ANSWER: D
7. The correct sequence of events involved in
phototransduction in rods and cones in response to light is:
A. activation of transducin, decreased release of
glutamate, structural changes in rhodopsin, closure of
Na+ channels, and decrease in intracellular cGMP.
B. decreased release of glutamate, activation of
transducin, closure of Na+ channels, decrease in
intracellular cGMP, and structural changes in
rhodopsin.
C. structural changes in rhodopsin, decrease in
intracellular cGMP, decreased release of glutamate,
closure of Na+ channels, and activation of transducin.
D. structural changes in rhodopsin, activation of
transducin, decrease in intracellular cGMP, closure of
Na+ channels, and decreased release of glutamate.
E. activation of transducin, structural changes in
rhodopsin, closure of Na+ channels, decrease in
intracellular cGMP, and decreased release of glutamate.
Ganong. Review of Medical Physiology 23rd ed. Chapter 12 , page 192 .
8. A 25-year-old medical student spent a summer volunteering ANSWER: B
in the sub-Saharan region of Africa. There he noted a high
incidence of people reporting difficulty with night vision due
to a lack of vitamin A in their diet. Vitamin A is a precursor • “In view of the importance of vitamin A in the synthesis of
for the synthesis of retinene1/retinal, it is not surprising that a deficiency in this vitamin
A. rods and cones. produces visual abnormalities. Among these, one of the earliest to
B. retinal. appear is night blindness (nyctalopia).”
C. rod transducin.
Ganong. Review of Medical Physiology 23rd ed. Chapter 12 , page 191.
D. opsin.
E. cone transducin.
9. An 11-year-old boy was having difficulty reading the graphs ANSWER: D
that his teacher was showing at the front of classroom. His
teacher recommended he be seen by an ophthalmologist.
Not only was he asked to look at a Snellen letter chart for
visual acuity but he was also asked to identify numbers in
an Ishihara chart. He responded that he merely saw a bunch • “Color blindness are inherited as recessive and X-linked
of dots. Abnormal color vision is 20 times more common in characteristics. Color blindness is present in males if the X
males than females because most cases are caused by an chromosome has the abnormal gene. Females show a defect only
abnormal when both X chromosomes contain the abnormal gene.”
A. dominant gene on the Y chromosome.
Ganong. Review of Medical Physiology 23rd ed. Chapter 12 , page 196..
B. recessive gene on the Y chromosome.
C. dominant gene on the X chromosome.
D. recessive gene on the X chromosome.
E. recessive gene on chromosome 22.
ANSWER: D
• Area V3A of visual cortex is responsible for motion and not for color
vision. Table: Functions of Visual Projection Areas in the Human Brain
10. Which of the following is not involved in color vision?
A. Activation of a pathway that signals differences
between S cone responses and the sum of L and M cone
responses
B. Geniculate layers 3–6
C. P pathway
D. Area V3A of visual cortex
E. Area V8 of visual cortex

Ganong. Review of Medical Physiology 23rd ed. Chapter 12, page 196 ..
11. A tumor was diagnosed near the base of the skull in a 56- ANSWER: B
year-old woman, impinging on her optic tract. Which of the
following statements about the central visual pathway is
correct?
A. The fibers from each temporal hemiretina decussate in
the optic chiasm, so that the fibers in the optic tracts are
those from the temporal half of one retina and the nasal
half of the other. • “ The visual pathway is from the rods and cones to bipolar cells to
B. In the geniculate body, the fibers from the nasal half of ganglion cells then via the optic tract to the thalamic lateral geniculate
one retina and the temporal half of the other synapse on body to the occipital lobe of the cerebral cortex. The fibers from each
the cells whose axons form the geniculocalcarine tract nasal hemiretina decussate in the optic chiasm; the fibers from the
C. Layers 2 and 3 of the visual cortex contain clusters of nasal half of one retina and the temporal half of the other synapse on
cells called globs that contain a high concentration of the cells whose axons form the geniculocalcarine tract.”
cytochrome oxidase.
Ganong. Review of Medical Physiology 23rd ed. Chapter , page .
D. Complex cells have a preferred orientation of a linear
stimulus and, compared to simple cells, are more
dependent on the location of the stimulus within the
visual field.
E. The visual cortex is arranged in horizontal columns that
are concerned with orientation.
TOPNOTCH MEDICAL BOARD PREP MED PHYSIOLOGY GANONG SUPPLEMENT HANDOUT BY THE TOPNOTCH TEAM Page 13 of 46
For inquiries visit www.topnotchboardprep.com.ph or https://web.facebook.com/topnotchmedicalboardprep/
TOPNOTCH MEDICAL BOARD PREP PHYSIOLOGY GANONG SUPPLEMENT HANDOUT BY THE TOPNOTCH TEAM
For inquiries visit www.topnotchboardprep.com.ph or https://www.facebook.com/topnotchmedicalboardprep/

12. Visual accommodation involves ANSWER: D


A. Increased tension on the lens ligaments • Accommodation is when the gaze is directed at a near object, the
B. a decrease in the curvature of the lens ciliary muscle contracts. This decreases the distance between the
C. relaxation of the sphincter muscle of the iris edges of the ciliary body and relaxes the lens ligaments, so that the
D. contraction of the ciliary muscle lens springs into a more convex shape.
E. increased intraocular pressure Ganong. Review of Medical Physiology 23rd ed. Chapter 12, page 189 .
ANSWER: B
13. The fovea of the eye: • “Near the posterior pole of the eye is a yellowish pigmented spot, the
A. has the lowest light threshold macula lutea. This marks the location of the fovea centralis, a
B. is the region of highest visual acuity thinned-out, rod-free portion of the retina that is present in humans
C. contains only red and green cones and other primates. In it, the cones are densely packed, and each
D. contains only rods synapses to a single bipolar cell, which, in turn, synapses on a single
E. is situated over the head of the optic nerve ganglion cell, providing a direct pathway to the brain. Consequently,
the fovea is the point where visual acuity is greatest.”
Ganong. Review of Medical Physiology 23rd ed. Chapter 12 , page 183,184 .
14. Vitamin A is a precursor for the synthesis of: ANSWER: B
A. Somatostatin • “In view of the importance of vitamin A in the synthesis of
B. Retinene1 retinene1/retinal, it is not surprising that a deficiency in this vitamin
C. The pigment of the iris produces visual abnormalities. Among these, one of the earliest to
D. Scotopsin appear is night blindness (nyctalopia).”
E. Aqueous humor Ganong. Review of Medical Physiology 23rd ed. Chapter 12 , page 191.
ANSWER: D
• Area V3A of visual cortex is responsible for motion and not for color
vision. Table: Functions of Visual Projection Areas in the Human Brain
15. Which of the following is not involved in color vision?
A. Activation of a pathway that signals differences
between S cone responses and the sum of L and M cone
responses
B. Geniculate layers 3–6
C. P pathway
D. Area V3A of visual cortex
E. Area V8 of visual cortex

Ganong. Review of Medical Physiology 23rd ed. Chapter 12, page 196 ..

(CHAPTER 10) HEARING & EQUILIBRIUM


1. A 45-year-old woman visited her clinician after experiencing ANSWER: A
sudden onset of vertigo, tinnitus & hearing loss in her left
ear, nausea, and vomiting. This was the second episode in
the past few months. She was referred to an otolaryngologist
to rule out Ménière disease. Which of the following • Sound waves are funneled through the external ear to the external
statements correctly describe the functions of the external, auditory meatus and then they pass inward to the tympanic
middle, or inner ear? membrane.
A. Sound waves are funneled through the external ear to • The cochlea of the inner ear contains receptors for hearing, otolith
the external auditory meatus and then they pass inward organs contain receptors that respond to head tilt, and the
to the tympanic membrane. semicircular canals contain receptors that respond to rotation.
B. The cochlea of the inner ear contains receptors for • Contraction of the tensor tympani and stapedius muscles of the
hearing, semicircular canals contain receptors that middle ear cause the manubrium of the malleus to be pulled inward
respond to head tilt, and the otolith organs contain and the footplate of the stapes to be pulled outward.
receptors that respond to rotation. • Sound waves are transformed by the eardrum and auditory ossicles
C. Contraction of the tensor tympani and stapedius into movements of the footplate of the stapes.
muscles of the middle ear cause the manubrium of the • E. The semicircular canals, the utricle, and the saccule of the inner
malleus to be pulled outward and the footplate of the ear are concerned with equilibrium.
stapes to be pulled inward.
D. Sound waves are transformed by the eardrum & auditory Ganong. Review of Medical Physiology 23rd ed. Chapter 13. page 211-214
ossicles into movements of the footplate of the malleus.
E. The semicircular canals, the utricle, and the saccule of
the middle ear are concerned with equilibrium.
ANSWER: E
2. A 45-year-old man with testicular cancer underwent
chemotherapy treatment with cisplatin. He reported several
adverse side effects including changes in taste, numbness
and tingling in his fingertips, and reduced sound clarity. • “The outer hair cells respond to sound, like the inner hair cells, but
When the damage to the outer hair cells is greater than the depolarization makes them shorten and hyperpolarization makes
damage to the inner hair cells, them lengthen. This action somehow increases the amplitude and
A. perception of vertical acceleration is disrupted. clarity of sounds.”
B. K+ concentration in endolymph is decreased.
C. K+ concentration in perilymph is decreased. Ganong. Review of Medical Physiology 23rd ed. Chapter 13, page 211
D. there is severe hearing loss.
E. affected hair cells fail to shorten when exposed to sound

TOPNOTCH MEDICAL BOARD PREP MED PHYSIOLOGY GANONG SUPPLEMENT HANDOUT BY THE TOPNOTCH TEAM Page 14 of 46
For inquiries visit www.topnotchboardprep.com.ph or https://web.facebook.com/topnotchmedicalboardprep/
TOPNOTCH MEDICAL BOARD PREP PHYSIOLOGY GANONG SUPPLEMENT HANDOUT BY THE TOPNOTCH TEAM
For inquiries visit www.topnotchboardprep.com.ph or https://www.facebook.com/topnotchmedicalboardprep/

3. Which of the following statements is correct? ANSWER: E


A. The motor protein for inner hair cells is prestin.
• High-pitched sounds generate waves that reach maximum height
B. The auditory ossicles function as a lever system to
near the base of the cochlea; low-pitched sounds generate waves that
convert the resonant vibrations of the tympanic
peak near the apex.
membrane into movements of the stapes against the
endolymph-filled scala tympani. • The motor protein for outer hair cells is prestin.
C. The loudness of a sound is directly correlated with the • The auditory ossicles function as a lever system to convert the
amplitude of a sound wave, and pitch is inversely resonant vibrations of the tympanic membrane into movements of
correlated with the frequency of the sound wave. the stapes against the endolymph-filled scala media.
D. Conduction of sound waves to the fluid of the inner ear • The loudness of a sound is directly correlated with the amplitude of
via the tympanic membrane and the auditory ossicles is a sound wave, and pitch is directly correlated with the frequency of
called bone conduction. the sound wave.
E. High-pitched sounds generate waves that reach • Bone conduction, is the transmission of vibrations of the bones of
maximum height near the base of the cochlea; low- the skull to the fluid of the inner ear
pitched sounds generate waves that peak near the apex. Ganong. Review of Medical Physiology 23rd ed. Chapter 13 , page 210, 211.

4. A 40-year-old man, employed as a road construction worker ANSWER: E


for nearly 20 years, went to his clinician to report that he
recently began to notice difficulty hearing during normal
conversations. A Weber test showed that sound from a
vibrating tuning fork was localized to the right ear. A
Schwabach test showed that bone conduction was below
normal. A Rinne test showed that both air and bone
conduction were abnormal, but air conduction lasted longer
than bone conduction. The diagnosis was
A. sensorial deafness in both ears.
B. conduction deafness in the right ear.
C. sensorial deafness in the right ear.
D. conduction deafness in the left ear.
E. sensorineural deafness in the left ear. Ganong. Review of Medical Physiology 23rd ed. Chapter 13, page 214.
ANSWER: B

5. What would the diagnosis be if a patient had the following


test results? Weber test showed that sound from a vibrating
tuning fork was louder than normal; Schwabach test showed
that bone conduction was better than normal; and Rinne test
showed that air conduction did not outlast bone conduction.
A. Sensorial deafness in both ears
B. Conduction deafness in both ears
C. Normal hearing
D. Both sensorial and conduction deafness
E. A possible tumor on the eighth cranial nerve

Ganong. Review of Medical Physiology 23rd ed. Chapter 13, page 214.

6. The auditory pathway ANSWER: D


A. and vestibular pathway contain a synapse in the
cerebellum.
B. and vestibular pathway project to the same regions of
the cerebral cortex.
C. is composed of afferent fibers of the eighth cranial • “The activity within the auditory pathway passes from the eighth
nerve, the dorsal and ventral cochlear nuclei, the cranial nerve afferent fibers to the dorsal and ventral cochlear nuclei
superior colliculi, the lateral geniculate body, and the to the inferior colliculi to the thalamic medial geniculate body and
auditory cortex. then to the auditory cortex.”
D. is composed of afferent fibers of the eighth cranial
Ganong. Review of Medical Physiology 23rd ed. Chapter 13, page 216
nerve, the dorsal and ventral cochlear nuclei, the
inferior colliculi, the medial geniculate body, and the
auditory cortex.
E. is not subject to plasticity like the visual pathways.
ANSWER: D
7. A healthy male medical student volunteered to undergo
evaluation of the function of his vestibular system for a class
demonstration. The direction of his nystagmus is expected to • Nystagmus is frequently horizontal (ie, the eyes move in the
be vertical when he is rotated horizontal plane), but it can also be vertical (when the head is tipped
A. after warm water is put in one of his ears. sidewise during rotation) or rotatory (when the head is tipped
B. with his head tipped backward. forward).
C. after cold water is put in both of his ears.
D. with his head tipped sideways. Ganong. Review of Medical Physiology 23rd ed. Chapter 13, page 215.
E. with his head tipped forward.

TOPNOTCH MEDICAL BOARD PREP MED PHYSIOLOGY GANONG SUPPLEMENT HANDOUT BY THE TOPNOTCH TEAM Page 15 of 46
For inquiries visit www.topnotchboardprep.com.ph or https://web.facebook.com/topnotchmedicalboardprep/
TOPNOTCH MEDICAL BOARD PREP PHYSIOLOGY GANONG SUPPLEMENT HANDOUT BY THE TOPNOTCH TEAM
For inquiries visit www.topnotchboardprep.com.ph or https://www.facebook.com/topnotchmedicalboardprep/
ANSWER: E

8. In the utricle, tip links in hair cells are involved in


A. formation of perilymph.
B. depolarization of the stria vascularis.
C. movements of the basement membrane.
D. perception of sound.
E. regulation of distortion-activated ion channels. Schematic representation of the role of tip links in the responses of hair
cells. When a stereocilium is pushed toward a taller stereocilium, the tip line is
stretched and opens an ion channel in its taller neighbour. The channel next is
presumably moved down the taller stereocilium by a molecular motor, so the
tension on the tip link is released. When the hairs return to the resting position,
the motor moves back up the stereocilium.
Ganong. Review of Medical Physiology 23rd ed. Chapter 13, page 208
9. Postrotatory nystagmus is caused by continued movement ANSWER: C
of
A. aqueous humor over the ciliary body in the eye.
B. cerebrospinal fluid over the parts of the brainstem that • “The direction of the quick component during rotation is the same as
contain the vestibular nuclei. that of the rotation, but the postrotatory nystagmus that occurs
C. endolymph in the semicircular canals, with consequent owing to displacement of the cupula when rotation is stopped is in
bending of the cupula and stimulation of hair cells. the opposite direction.”
D. endolymph toward the helicotrema.
Ganong. Review of Medical Physiology 23rd ed. Chapter 13, page 215.
E. perilymph over hair cells that have their processes
embedded in the tectorial membrane
10. A patient enters the hospital for evaluation of deafness. He ANSWER: A
is found to also have an elevated plasma renin, although his
blood pressure is 118/75 mm Hg. Mutation of what single • “Bartter syndrome is due to defective transport in the thick ascending
gene may explain these findings? limb. It is characterized by chronic Na+ loss in the urine, with resultant
A. The gene for barttin hypovolemia causing stimulation of rennin and aldosterone secretion.
B. The gene for Na+ channel The condition can be caused by loss-of-function mutations in the gene
C. The gene for renin for any of four key proteins: the Na–K–2Cl cotransporter, the ROMK K+
D. The gene for cystic fibrosis transmembrane channel,the ClC–Kb Cl– channel, or barttin.”
conductance regulator
Ganong. Review of Medical Physiology 23rd ed. Chap 13, pg 214 ;Chap 38, pg 654.
E. The gene for tyrosine hydroxylase

(CHAPTER 11) SMELL & TASTE


1. A young boy was diagnosed with congenital anosmia, a ANSWER: D
rare disorder in which an individual is born without the
ability to smell. Odorant receptors are
A. located in the olfactory bulb. • “Olfactory sensory neuron expresses only one of the 1000 different
B. located on dendrites of mitral and tufted cells. odorant receptors. Each neuron projects to one or two glomeruli which
C. located on neurons that project directly to the provides a distinct two-dimensional map in the olfactory bulb that is
olfactory cortex. unique to the odorant. The mitral cells with their glomeruli project to
D. located on neurons in the olfactory epithelium that different parts of the olfactory cortex.”
project to mitral cells and from there directly to the
olfactory cortex. Ganong. Review of Medical Physiology 23rd ed. Chapter 14, page 222..
E. located on sustentacular cells that project to the
olfactory bulb.
ANSWER: D
2. A 37-year-old female was diagnosed with multiple
sclerosis. One of the potential consequences of this • Taste receptors , not synonymous with taste buds, are a type of
disorder is diminished taste sensitivity. Taste receptors chemoreceptor.
A. for sweet, sour, bitter, salt, and umami are spatially • It used to be thought that the surface of the tongue had special areas, but
separated on the surface of the tongue. it is now clear that all tastants are sensed from all parts of the tongue
B. are synonymous with taste buds. and adjacent structures.
C. are a type of chemoreceptor. • The sensory nerve fibers from the anterior two-thirds of the tongue
D. are innervated by afferents in the facial, trigeminal, (facial nerve), and those from the posterior third of the
and glossopharyngeal nerves. (glossopharyngeal nerve). The fibers from areas other than the tongue
E. All of the above. (vagus nerve)
Ganong. Review of Medical Physiology 23rd ed. Chapter 14, page 223-225 .
ANSWER: D
3. Which of the following does not increase the ability to • “Termination of the activity of transducin in photoreception is also
discriminate many different odors? accelerated by its binding of β-arrestin.” therefore β-arrestin is for vision
A. Many different receptors and not for olfaction.
B. Pattern of olfactory receptors activated by a given • The portion of the nasal cavity containing the olfactory receptors is
odorant poorly ventilated in humans, amount of air reaching this region is greatly
C. Projection of different mitral cell axons to different increased by sniffing.
parts of the brain • Increased in number of different receptors, pattern of olfactory receptors
D. High β-arrestin content in olfactory neurons activated by a given odorant, and projection of different mitral cell axons
E. Sniffing to different parts of the brain increases the ability to discriminate many
different odors
Ganong. Review of Medical Physiology 23rd ed. Chap 12, page 191; Chap 14, page 223.

TOPNOTCH MEDICAL BOARD PREP MED PHYSIOLOGY GANONG SUPPLEMENT HANDOUT BY THE TOPNOTCH TEAM Page 16 of 46
For inquiries visit www.topnotchboardprep.com.ph or https://web.facebook.com/topnotchmedicalboardprep/
TOPNOTCH MEDICAL BOARD PREP PHYSIOLOGY GANONG SUPPLEMENT HANDOUT BY THE TOPNOTCH TEAM
For inquiries visit www.topnotchboardprep.com.ph or https://www.facebook.com/topnotchmedicalboardprep/

4. As a result of an automobile accident, a 10-year-old boy suffered ANSWER: D


damage to the brain including the periamygdaloid, piriform, and
entorhinal cortices. Which of the following sensory deficits is he most • “The pathway to the amygdala is probably involved with the
likely to experience? emotional responses to olfactory stimuli, and the pathway to
A. Visual disturbance the entorhinal cortex is concerned with olfactory
B. Hyperosmia memories.” Damage along this pathway would result to odor
C. Auditory problems and taste abnormalities.
D. Taste and odor abnormalities
Ganong. Review of Medical Physiology 23rd ed. Chapter 14, page 221.
E. No major sensory deficits
5. A 9-year-old boy had frequent episodes of uncontrollable nose bleeds. ANSWER: C
At the advice of his clinician, he underwent surgery to correct a • Conscious discrimination of odor depends on the neocortex
problem in his nasal septum. A few days after the surgery, he told his (orbitofrontal and frontal cortices).
mother he could not smell the cinnamon rolls she was baking in the • Although there are millions of olfactory sensory neurons,
oven. Which of the following is true about olfactory transmission? each expresses only one of the 1000 different odorant
A. An olfactory sensory neuron expresses a wide range of odorant receptors.
receptors. • The olfactory glomeruli demonstrate lateral inhibition
B. Lateral inhibition within the olfactory glomeruli reduces the mediated by periglomerular cells and granule cells. This
ability to distinguish between different types of odorant sharpens and focuses olfactory signals.
receptors. • Olfaction is physiologically related to gustation because of
C. Conscious discrimination of odors is dependent on the pathway its use of chemoreceptors , and thru chemoreceptive
to the orbitofrontal cortex. sensory interaction, they are able to perceive complex flavors.
D. Olfaction is closely related to gustation because odorant and
gustatory receptors use the same central pathways. Ganong. Review of Medical Physiology 23rd ed. Chapter 14, pages
E. All of the above. 219,221,222.
6. A 31-year-old female is a smoker who has had poor oral hygiene for ANSWER: D
most of her life. In the past few years she has noticed a reduced
sensitivity to the flavors in various foods which she used to enjoy
eating. Which of the following is not true about gustatory sensation?
A. The sensory nerve fibers from the taste buds on the anterior
two-thirds of the tongue travel in the chorda tympani branch of
the facial nerve. • The human olfactory epithelium contains 10 to 20 million
B. The sensory nerve fibers from the taste buds on the posterior bipolar olfactory sensory neurons interspersed with glia-like
third of the tongue travel in the petrosal branch of the supporting (sustentacular cells) and basal stem cells
glossopharyngeal nerve. which permit growth of new taste buds.
C. The pathway from taste buds on the left side of the tongue is
transmitted ipsilaterally to the cerebral cortex. Ganong. Review of Medical Physiology 23rd ed. Chapter 14, page 219.
D. Sustentacular cells in the taste buds serve as stem cells to permit
growth of new taste buds.
E. The pathway from taste receptors includes synapses in the
nucleus of the tractus solitarius in the brainstem and ventral
posterior medial nucleus in the thalamus.
7. A 20-year-old woman was diagnosed with Bell palsy (damage to facial ANSWER: E
nerve). Which of the following symptoms is she likely to exhibit? • “The onset of Bell’s palsy is fairly abrupt, with maximal
A. Loss of sense of taste weakness being attained by 48 h as a general rule. Pain
B. Facial twitching behind the ear may precede the paralysis for a day or two.
C. Droopy eyelid Taste sensationmay be l ost unilaterally, and hyperacusis
D. Ipsilateral facial paralysis may be present.”
E. All of the above Harrison’s Principles of Internal Medicine 19th ed. Chap 445 , page 2648.

(CHAPTER 12) REFLEX & VOLUNTARY CONTROL OF POSTURE & MOVEMENT


1. When dynamic γ-motor neurons are activated at the same time as α- ANSWER: E
motor neurons to muscle,
A. prompt inhibition of discharge in spindle Ia afferents takes place. • “Activation of dynamic γ-motor neurons increases the
B. clonus is likely to occur. dynamic sensitivity of the group Ia endings.”
C. the muscle will not contract. • “Because of this “α–γ coactivation,” intrafusal and extrafusal
D. the number of impulses in spindle Ia afferents is smaller than fibers shorten together, and spindle afferent activity can
when α discharge alone is increased. occur throughout the period of muscle contraction.”
E. the number of impulses in spindle Ia afferents is greater than
Ganong. Review of Medical Physiology 23rd ed. Chapt 9, page 158, 160.
when α discharge alone is increased.
ANSWER: C

2. The inverse stretch reflex


A. occurs when Ia spindle afferents are inhibited.
B. is a monosynaptic reflex initiated by activation of the Golgi
tendon organ.
C. is a disynaptic reflex with a single interneuron inserted between
the afferent and efferent limbs.
D. is a polysynaptic reflex with many interneurons inserted
between the afferent and efferent limbs.
E. uses type II afferent fibers from the Golgi tendon organ.

Ganong. Review of Medical Physiology 23rd ed. Chap , page .


TOPNOTCH MEDICAL BOARD PREP MED PHYSIOLOGY GANONG SUPPLEMENT HANDOUT BY THE TOPNOTCH TEAM Page 17 of 46
For inquiries visit www.topnotchboardprep.com.ph or https://web.facebook.com/topnotchmedicalboardprep/
TOPNOTCH MEDICAL BOARD PREP PHYSIOLOGY GANONG SUPPLEMENT HANDOUT BY THE TOPNOTCH TEAM
For inquiries visit www.topnotchboardprep.com.ph or https://www.facebook.com/topnotchmedicalboardprep/

ANSWER: E
• “Withdrawal reflexes are prepotent, they preempt the spinal
pathways from any other reflex activity taking place at the moment.”
3. Withdrawal reflexes are not • “The withdrawal reflex is a typical polysynaptic reflex that occurs in
A. initiated by nociceptive stimuli. response to a usually painful stimulation.”
B. prepotent. • “A weak stimulus causes one quick flexion movement; a strong
C. prolonged if the stimulus is strong. stimulus causes prolonged flexion and sometimes a series of flexion
D. an example of a flexor reflex. movements.”
E. accompanied by the same response on both sides of the • “When a strong stimulus is applied to a limb, the response includes
body. not only flexion and withdrawal of that limb but also extension of
the opposite limb. This crossed extensor response is properly part
of the withdrawal reflex.”
Ganong. Review of Medical Physiology 23rd ed. Chapter 9, page 163-164.
4. While exercising, a 42-year-old woman developed sudden ANSWER: C
onset of tingling in her right leg and an inability to control
movement in that limb. A neurologic exam showed a
hyperactive knee jerk reflex and a positive Babinski sign. • “When the spinal cord is suddenly transected all cord functions,
Which of the following is not characteristic of a reflex? including the cord reflexes, immediately become depressed, a
A. Reflexes can be modified by impulses from various parts reaction called spinal shock.”
of the CNS • “After a few hours to a few weeks, the spinal neurons gradually
B. Reflexes may involve simultaneous contraction of some regain their excitability, they increase their own natural degree of
muscles and relaxation of others excitability to make up at least partially for the loss.”
C. Reflexes are chronically suppressed after spinal cord
transection Textbook of Medical Physiology 11th ed. Guyton Chapter 54, page 684 .
D. Reflexes involve transmission across at least one synapse
E. Reflexes often occur without conscious perception
ANSWER: E
• “Commands for voluntary movement originate in cortical
association areas. The cortex, basal ganglia, and cerebellum work
cooperatively to plan movements. Movement executed by the cortex
is relayed via the corticospinal tracts and corticobulbar tracts to
5. Increased neural activity before a skilled voluntary movement
motor neurons. The cerebellum provides feedback to adjust and
is first seen in the
smooth movement.”
A. spinal motor neurons.
B. precentral motor cortex.
C. midbrain.
D. cerebellum.
E. cortical association areas.

Ganong. Review of Medical Physiology 23rd ed. Chapter 16, page 242
6. A 58-year-old woman was brought to the emergency ANSWER: B
department of her local hospital because of a sudden change of
consciousness. All four limbs were extended, suggestive of
decerebrate rigidity. A brain CT showed a rostral pontine
hemorrhage. Which of the following describes components of
the central pathway responsible for control of posture?
A. The tectospinal pathway terminates on neurons in the
dorsolateral area of the spinal ventral horn that innervate • The medial brain stem pathways, which work in concert with the
limb muscles. ventral corticospinal tract, are the pontine and medullary
B. The medullary reticulospinal pathway terminates on reticulospinal, vestibulospinal, and tectospinal tracts. These
neurons in the ventromedial area of the spinal ventral pathways descend in the ipsilateral ventral columns of the spinal
horn that innervate axial and proximal muscles. cord and terminate predominantly on interneurons and long
C. The pontine reticulospinal pathway terminates on propriospinal neurons in the ventromedial part of the ventral horn
neurons in the dorsomedial area of the spinal ventral to control axial and proximal muscles.
horn that innervate limb muscles.
Ganong. Review of Medical Physiology 23rd ed. Chapter 16, page 246.
D. The medial vestibular pathway terminates on neurons in
the dorsomedial area of the spinal ventral horn that
innervate axial and proximal muscles.
E. The lateral vestibular pathway terminates on neurons in
the dorsolateral area of the spinal ventral horn that
innervate axial and proximal muscles.
7. A 38-year-old woman with a metastatic brain tumor was ANSWER: C
brought to the emergency department of her local hospital
because of irregular breathing and progressive loss of
consciousness. She also showed signs of decerebrate posturing.
Which of the following is not true about decerebrate rigidity? • “In midcollicular decerebrate cats, section of dorsal roots to a limb
A. It involves hyperactivity in extensor muscles of all 4 limbs immediately eliminates the hyperactivity of extensor muscles. This
B. The excitatory input from the reticulospinal pathway suggests That decerebrate rigidity is spasticity due to facilitation
activates γ-motor neurons, which indirectly activate α- of the myotatic stretch reflex.”
motor neurons.
C. It is actually a type of spasticity due to inhibition of the Ganong. Review of Medical Physiology 23rd ed. Chapter 16, page 247 .
stretch reflex.
D. It resembles what ensues after uncal herniation.
E. Lower extremities are extended with toes pointed inward

TOPNOTCH MEDICAL BOARD PREP MED PHYSIOLOGY GANONG SUPPLEMENT HANDOUT BY THE TOPNOTCH TEAM Page 18 of 46
For inquiries visit www.topnotchboardprep.com.ph or https://web.facebook.com/topnotchmedicalboardprep/
TOPNOTCH MEDICAL BOARD PREP PHYSIOLOGY GANONG SUPPLEMENT HANDOUT BY THE TOPNOTCH TEAM
For inquiries visit www.topnotchboardprep.com.ph or https://www.facebook.com/topnotchmedicalboardprep/
ANSWER: A
• The subthalamic nucleus releases glutamate to excite the
globus pallidus, internal segment.
• The substantia nigra pars reticulata releases GABA to inhibit
8. Which of the following describes a connection between the thalamus.
components of the basal ganglia?
• The substantia nigra pars compacta releases dopamine to
A. The subthalamic nucleus releases glutamate to excite the
inhibit the striatum.
globus pallidus, internal segment.
• The striatum releases acetylcholine to inhibit the substantia
B. The substantia nigra pars reticulata releases dopamine to
nigra pars reticulata.
inhibit the striatum.
• E. The globus pallidus, external segment releases GABA to
C. The substantia nigra pars compacta releases dopamine to
inhibit the subthalamic nucleus .
excite the globus pallidus, external segment.
D. The striatum releases acetylcholine to excite the substantia
nigra pars reticulata.
E. The globus pallidus, external segment releases glutamate to
excite the striatum.

Ganong. Review of Medical Physiology 23rd ed. Chapter 16 , page 251 , 257
9. A 60-year-old man with Parkinson disease, which was ANSWER: E
diagnosed 15 years ago, has been taking carbidopa and
levodopa (Sinemet); until recently, he has been able to continue
to work and help with routine jobs around the house. Now his
tremor and rigidity interfere with these activities. His clinician
has suggested that he undergo deep brain stimulation therapy. • Administration of L-dopa (levodopa), unlike dopamine, crosses
The therapeutic effect of L-dopa in patients with Parkinson the blood– brain barrier and helps repair the dopamine
disease eventually wears off because deficiency. However, the degeneration of these neurons
A. antibodies to dopamine receptors develop. continues and in 5 to 7 years the beneficial effects of L-dopa
B. inhibitory pathways grow into the basal ganglia from the disappear.
frontal lobe.
Ganong. Review of Medical Physiology 23rd ed. Chapter 16, page 254.
C. there is an increase in circulating α-synuclein.
D. the normal action of nerve growth factor (NGF) is disrupted
E. the dopaminergic neurons in the substantia nigra continue
to degenerate.
ANSWER: C
• Granule cells release glutamate to excite basket cells and
stellate cells.
• Basket cells release GABA to inhibit Purkinje cells.
• Climbing and mossy fiber inputs exert a strong excitatory effect
on Purkinje cells.
10. An 8-year-old girl was brought to her pediatrician because her • The axons of Purkinje cells are the sole output of the cerebellar
parents noted frequent episodes of gait unsteadiness and speech cortex, and they release GABA to inhibit the deep cerebellar
difficulties. Her mother was concerned because of a family nuclei.
history of Friedreich ataxia. Which of the following is a correct • Golgi cells are excited by mossy fiber collaterals.
description of connections involving cerebellar neurons?
A. Basket cells release glutamate to activate Purkinje cells.
B. Climbing fiber inputs exert a strong excitatory effect on
Purkinje cells, and mossy fiber inputs exert a strong
inhibitory effect on Purkinje cells.
C. Granule cells release glutamate to excite basket cells and
stellate cells.
D. The axons of Purkinje cells are the sole output of the
cerebellar cortex, and they release glutamate to excite the
deep cerebellar nuclei.
E. Golgi cells are inhibited by mossy fiber collaterals.

Ganong. Review of Medical Physiology 23rd ed. Chapter 16 , page 256-257 .


ANSWER: D
11. After falling down a flight of stairs, a young woman is found to
have partial loss of voluntary movement on the right side of her
body and loss of pain and temperature sensation on the left side
below the midthoracic region. It is probable that she has a lesion • There is transection of the right half of the spinal cord in the
A. transecting the left half of the spinal cord in the lumbar upper thoracic region resulting in Brown–Séquard Syndrome,
region. described as “the loss of the spinothalamic tract leads to
B. transecting the left half of the spinal cord in the upper contralateral loss of pain and temperature sensation beginning
thoracic region. one or two segments below the lesion. Damage to the
C. transecting sensory and motor pathways on the right side of corticospinal tract produces weakness and spasticity in
the pons. certain muscle groups on the same side of the body.”
D. transecting the right half of the spinal cord in the upper
thoracic region. Ganong. Review of Medical Physiology 23rd ed. Chapter 11 , page 177 .
E. transecting the dorsal half of the spinal cord in the upper
thoracic region.

TOPNOTCH MEDICAL BOARD PREP MED PHYSIOLOGY GANONG SUPPLEMENT HANDOUT BY THE TOPNOTCH TEAM Page 19 of 46
For inquiries visit www.topnotchboardprep.com.ph or https://web.facebook.com/topnotchmedicalboardprep/
TOPNOTCH MEDICAL BOARD PREP PHYSIOLOGY GANONG SUPPLEMENT HANDOUT BY THE TOPNOTCH TEAM
For inquiries visit www.topnotchboardprep.com.ph or https://www.facebook.com/topnotchmedicalboardprep/

12. At the age of 30, a male postal worker reported weakness in his ANSWER: E
right leg. Within a year the weakness had spread to his entire • Patient’s neurologic examination decribes a lower motor neuron
right side. A neurologic examination revealed flaccid paralysis, problem; therefore the cerebral cortex, basal ganglia, and
muscular atrophy, fasciculations, hypotonia, and hyporeflexia of vestibulocerebellar can be eliminated because they are within
muscles in the right arm and leg. Sensory and cognitive function the CNS (upper motor neurons).
tests were normal. Which of the following diagnosis is likely? • Lower motor neurons (i.e. α-motor neurons.) are those whose
A. A large tumor in the left primary motor cortex axons terminate on skeletal muscles. An example of a disease that
B. A cerebral infarct in the region of the corona radiate leads to lower motor neuron damage is amyotrophic lateral
C. A vestibulocerebellar tumor sclerosis (ALS) which is a selective, progressive degeneration of
D. Damage to the basal ganglia α -motor neurons.
E. Amyotrophic lateral sclerosis Ganong. Review of Medical Physiology 23rd ed. Chapter 16 , page 244.
ANSWER: C
• Basal ganglia is involved in the planning and programming of
13. A primary function of the basal ganglia is: movement or, more broadly, in the processes by which an
A. sensory integration abstract thought is converted into voluntary action.
B. short-term memory • Thalamus: sensory integration
C. planning voluntary movement • Ventral hippocampus: short-term memory
D. neuroendocrine control • Precentral motor cortex: planning voluntary movement
E. slow-wave sleep • Hypothalamus: neuroendocrine control
• Pontine Reticular Formation: slow-wave sleep
Ganong. Review of Medical Physiology 23rd ed. Chapter 16 , page 250.
ANSWER: B
14. Patients with transected spinal cords frequently have a negative
nitrogen balance because:
A. they develop hypercalcemia, and this causes dissolution of
the protein in bone
B. they are paralyzed below the level of the transection • “Due to immobilization, SCI patients develop a negative nitrogen
C. they lack afferent input that normally maintains growth balance and catabolize large amounts of body protein.”
hormone secretion
Ganong. Review of Medical Physiology 23rd ed. Chapter 16 , page 250.
D. they have difficulty voiding, and this causes nitrogen to
accumulate in the urine in the bladder
E. their corticotropin response to stress is reduced

15. Which of the following diseases is NOT known to be caused by ANSWER: A


overexpression of trinucleotide repeat?
A. Alzheimer disease • Alzheimer’s disease appears to be a metabolic
B. Fragile X syndrome neurodegenerative disease and not caused by trinucleotide
C. Spinocerebellar ataxia, type 3 repeats.
D. Huntington disease
Guyton.Textbook of Medical Physiology 11th ed.Chapter 59 , page 746.
E. Friedreich ataxia

(CHAPTER 13) AUTONOMIC NERVOUS SYSTEM


ANSWER: A
1. Hypertension developed in a 26-year-old man after he began • Phenylephrine: Alpha1-selective agonist which causes
taking amphetamine to boost his energy and to suppress his increased vascular resistance secondary to its action in the
appetite. Which of the following drugs would be expected to vascular smooth muscle
mimic the effects of increased sympathetic discharge on blood • Trimethaphan: ganglion blocker, treatment for malignant HPN
vessels? • Atropine: blocks muscarinic receptors and inhibits
A. Phenylephrine parasympathomimetic effects.
B. Trimethaphan • Reserpine: Drugs that deplete the adrenergic nerve terminal of its
C. Atropine norepinephrine stores
D. Reserpine • Albuterol: Beta-adrenoceptor agonists, caused bronchodilation.
E. Albuterol Katzung. Pharmacology Examination & Board Review 10th Edition.Chapter 9,
page 74,75,79; Chapter 11 page 102; Chapter 20, page 184
ANSWER: D
2. A 35-year-old woman in whom multiple system atrophy was
• Acetylcholine is released from all sympathetic preganglionic
diagnosed had symptoms indicative of failure of sympathetic
nerve terminals.
nerve activity. Which of the following statements about the
sympathetic nervous system is correct? • Not all postganglionic sympathetic nerves releases
A. All postganglionic sympathetic nerves release norepinephrine; exemptions are neurons that innervate sweat
norepinephrine from their terminals. glands and blood vessels in some skeletal muscles, both of
B. Cell bodies of preganglionic sympathetic neurons are which releases acetylcholine.
located in the intermediolateral column of the thoracic and • Cell bodies of preganglionic sympathetic neurons are located in
sacral spinal cord. the intermediolateral column of the thoracic and lumbar spinal
C. The sympathetic nervous system is required for survival. cord.
D. Acetylcholine is released from all sympathetic • Although survival is possible without an ANS, the ability to adapt
preganglionic nerve terminals. to environmental stressors is severely compromised
E. The sympathetic nervous system adjusts pupillary • The sympathetic nervous system adjusts pupillary diameter by
diameter by relaxing the pupillary constrictor muscle. contracting the pupillary constrictor muscle.
Ganong. Review of Medical Physiology 23rd ed. Chapter 17 , page 266 .

TOPNOTCH MEDICAL BOARD PREP MED PHYSIOLOGY GANONG SUPPLEMENT HANDOUT BY THE TOPNOTCH TEAM Page 20 of 46
For inquiries visit www.topnotchboardprep.com.ph or https://web.facebook.com/topnotchmedicalboardprep/
TOPNOTCH MEDICAL BOARD PREP PHYSIOLOGY GANONG SUPPLEMENT HANDOUT BY THE TOPNOTCH TEAM
For inquiries visit www.topnotchboardprep.com.ph or https://www.facebook.com/topnotchmedicalboardprep/
3. A 45-year-old man had a meal containing wild mushrooms that he ANSWER: C
picked in a field earlier in the day. Within a few hours after eating,
nausea, vomiting, diarrhea, urinary urgency, vasodilation, sweating,
and salivation developed. Which of the following statements about the
parasympathetic nervous system is correct?
A. Postganglionic parasympathetic nerves release acetylcholine to
activate muscarinic receptors on sweat glands • “In contrast, release of acetylcholine onto smooth muscle of
B. Parasympathetic nerve activity affects only smooth muscles and some organs leads to contraction (eg, walls of the
glands. gastrointestinal tract) while release onto other organs leads
C. Parasympathetic nerve activity causes contraction of smooth to relaxation (eg, sphincters in the gastrointestinal tract).”
muscles of the gastrointestinal wall and relaxation of the
Ganong. Review of Medical Physiology 23rd ed. Chapter 17, page 266 .
gastrointestinal sphincter.
D. Parasympathetic nerve activity causes contraction of the radial
muscle of the eye to allow accommodation for near vision.
E. An increase in parasympathetic activity causes an increase in
heart rate.
ANSWER: D
4. Which of the following is correctly paired? • Vasculature of some skeletal muscles: Muscarinic
A. Sinoatrial node: Nicotinic cholinergic receptors cholinergic receptors
B. Autonomic ganglia: Muscarinic cholinergic receptors • Sinoatrial node: β1-adrenergic receptors
C. Pilomotor smooth muscle: β2-adrenergic receptors • Autonomic ganglia: Nicotinic cholinergic receptors
D. Vasculature of some skeletal muscles: Muscarinic cholinergic • Pilomotor smooth muscle: α1-adrenergic receptors
receptors • Sweat glands: α1-adrenergic receptors
E. Sweat glands: α2-adrenergic receptors Ganong. Review of Medical Physiology 23rd ed. Chapter 7, page 135 .
Chapter 17, page 270..
ANSWER: C
• Pathways that control autonomic responses. Direct
projections (solid lines) to autonomic preganglionic
neurons include the hypothalamic paraventricular nucleus,
5. A 57-year-old man had severe hypertension that was found to result parabrachial nucleus, nucleus of the solitary tract,
from a tumor compressing on the surface of the medulla. Which one of ventrolateral medulla, and medullary raphe (not shown).
the following statements about pathways involved in the control of Indirect projections (dashed lines) include the cerebral
sympathetic nerve activity is correct? cortex, amygdala, and periaqueductal grey matter.
A. Preganglionic sympathetic nerves receive inhibitory input from
the rostral ventrolateral medulla.
B. The major source of excitatory input to preganglionic sympathetic
nerves is the paraventricular nucleus of the hypothalamus.
C. The activity of sympathetic preganglionic neurons can be affected
by the activity of neurons in the amygdala.
D. Unlike the activity in δ-motor neurons, sympathetic preganglionic
neurons are not under any significant reflex control.
E. Under resting conditions, the sympathetic nervous system is not
active; it is active only during stress giving rise to the term “flight
or fight” response.

Ganong. Review of Medical Physiology 23rd ed. Chapter 17, page 271
6. Diabetic autonomic neuropathy was diagnosed a few years ago in a ANSWER: C/E
53-year-old woman with diabetes. She recently noted abdominal
distension and a feeling of being full after eating only a small portion
of food, suggesting the neuropathy had extended to her enteric • The enteric nervous system is not a subdivision of the
nervous system to cause gastroparesis. Which of the following parasympathetic nervous system; in fact, it can function
statements about the enteric nervous system is correct? independent of the autonomic innervation to the
A. The enteric nervous system is a subdivision of the parasympa- gastrointestinal tract.
thetic nervous system for control of gastrointestinal function. • the myenteric plexus (Auerbach’s plexus), between the
B. The myenteric plexus is a group of motor neurons located within outer longitudinal and middle circular muscle layers, and
circular layer of muscle in a portion of the gastrointestinal tract. the submucous plexus (Meissner’s plexus), between the
C. The submucosal plexus is a group of sensory neurons located middle circular layer and the mucosa.
between the circular muscle and the luminal mucosa of the • Neurons comprising the enteric nervous system are
gastrointestinal tract. located only in the intestine.
D. Neurons comprising the enteric nervous system are located only
in the stomach and intestine. Ganong. Review of Medical Physiology 23rd ed. Chapter 26, page 448 .
E. The enteric nervous system can function independent of the
autonomic innervation to the gastrointestinal tract.
ANSWER: A
7. Which of the following drugs would NOT be expected to increase
sympathetic discharge or mimic the effects of increased sympathetic
discharge? • Prazosin is an alpha1-selective blocker prototype used in
A. Prazosin hypertension and benign prostatic hyperplasia .
B. Neostigmine • All other drugs are symphatomimetics.
C. Amphetamine
D. Isoproterenol Katzung. Pharmacology Examination & Board Review 10th Edition.
Chapter 11 page 102
E. Methoxamine

TOPNOTCH MEDICAL BOARD PREP MED PHYSIOLOGY GANONG SUPPLEMENT HANDOUT BY THE TOPNOTCH TEAM Page 21 of 46
For inquiries visit www.topnotchboardprep.com.ph or https://web.facebook.com/topnotchmedicalboardprep/
TOPNOTCH MEDICAL BOARD PREP PHYSIOLOGY GANONG SUPPLEMENT HANDOUT BY THE TOPNOTCH TEAM
For inquiries visit www.topnotchboardprep.com.ph or https://www.facebook.com/topnotchmedicalboardprep/

8. Sympathetic nerve activity: ANSWER: A


A. is essential for survival
B. causes contraction of some smooth muscles and relaxation of • Sympathetic nerve activity causes contraction of some
others smooth muscles (i.e. sphincters) and relaxation of others
C. causes relaxation of the radial muscles of the eye to dilate the pupil (i.e. walls GI tract)
D. relaxes smooth muscle of the gastrointestinal wall and
gastrointestinal sphincter Ganong. Review of Medical Physiology 23rd ed. Chapter 17 , page 268.
E. all of the above
9. Parasympathetic nerve activity: ANSWER: D
A. is essential for survival
B. affects only smooth muscles and glands • “Parasympathetic action favors digestion and
C. causes contraction of the radial muscle of the eye to allow absorption of food by increasing the activity of the
accommodation for near vision intestinal musculature, increasing gastric secretion, and
D. contracts smooth muscle of the gastrointestinal wall and relaxing the pyloric sphincter.”
relaxation of gastrointestinal sphincter
Ganong. Review of Medical Physiology 23rd ed. Chapter 17, page 266.
E. all of the above

(CHAPTER 14) ELECTRICAL ACTIVITY OF THE BRAIN, SLEEP–WAKE STATES, & CIRCADIAN RHYTHMS
ANSWER: C
1. In a healthy, alert adult sitting with their eyes closed, the dominant • alpha (8–13 Hz): awake but at rest with the mind
EEG rhythm observed with electrodes over the occipital lobes is wandering and the eyes closed.
A. delta (0.5–4 Hz). • delta (0.5–4 Hz) : stage 3 NREM sleep
B. theta (4–7 Hz). • theta (4–7 Hz) : early stage of slowwave sleep
C. alpha (8–13 Hz). • beta (18–30 Hz) : aroused, alert state
D. beta (18–30 Hz). • fast, irregular low-voltage activity : motor activity in
E. fast, irregular low-voltage activity. response to the stimulus
Ganong. Review of Medical Physiology 23rd ed. Chap 15, pg 233, 234.
ANSWER: D

2. A 35-year-old man spent the evening in a sleep clinic to determine


whether he had obstructive sleep apnea. The tests showed that NREM
sleep accounted for over 30% of his total sleep time. Which of the
following pattern of changes in central neurotransmitters or
neuromodulators are associated with the transition from NREM to
wakefulness?
A. Decrease in norepinephrine, increase in serotonin, increase in
acetylcholine, decrease in histamine, and decrease in GABA.
B. Decrease in norepinephrine, increase in serotonin, increase in
acetylcholine, decrease in histamine, and increase in GABA.
C. Decrease in norepinephrine, decrease in serotonin, increase in
acetylcholine, increase in histamine, and increase in GABA.
D. Increase in norepinephrine, increase in serotonin, decrease in
acetylcholine, increase in histamine, and decrease in GABA.
E. Increase in norepinephrine, decrease in serotonin, decrease in
acetylcholine, increase in histamine, and decrease in GABA.

Ganong. Review of Medical Physiology 23rd ed. Chapter 15, page 239
3. A gamma rhythm (30–80 Hz) ANSWER: C
A. is characteristic of seizure activity.
B. is seen in an individual who is awake but not focused. • “The gamma rhythm has been suggested as a mechanism
C. may be a mechanism to bind together sensory information into a to “bind” together diverse sensory information into a
single percept and action. single percept and action .”
D. is independent of thalamocortical loops.
Ganong. Review of Medical Physiology 23rd ed. Chapter 15, page 236 .
E. is generated in the hippocampus.
ANSWER: D
4. For the past several months, a 67-year-old woman experienced
difficulty initiating and/or maintaining sleep several times a week. A
friend suggested that she take melatonin to regulate her sleep–wake
cycle. Melatonin secretion would probably not be increased by • Direct light stimulation of the retina would send signals to
A. stimulation of the superior cervical ganglia. the optic nerve that would lead to decrease activity of the
B. intravenous infusion of tryptophan. pineal gland and decrease in secretion of melatonin.
C. intravenous infusion of epinephrine.
Ganong. Review of Medical Physiology 23rd ed. Chapter , page .
D. stimulation of the optic nerve.
E. induction of pineal hydroxyindole-O-methyltransferase.

TOPNOTCH MEDICAL BOARD PREP MED PHYSIOLOGY GANONG SUPPLEMENT HANDOUT BY THE TOPNOTCH TEAM Page 22 of 46
For inquiries visit www.topnotchboardprep.com.ph or https://web.facebook.com/topnotchmedicalboardprep/
TOPNOTCH MEDICAL BOARD PREP PHYSIOLOGY GANONG SUPPLEMENT HANDOUT BY THE TOPNOTCH TEAM
For inquiries visit www.topnotchboardprep.com.ph or https://www.facebook.com/topnotchmedicalboardprep/
5. Childhood absence epilepsy was diagnosed in a 10-year-old boy. His ANSWER: A
EEG showed a bilateral synchronous, symmetric 3-Hz spike-and-wave
discharge. Absence seizures
A. are a form of nonconvulsive generalized seizures accompanied by
momentary loss of consciousness. • “Absence seizures (formerly called petit mal seizures) are
B. are a form of complex partial seizures accompanied by momentary one of the forms of nonconvulsive generalized seizures
loss of consciousness. characterized by a momentary loss of consciousness.
C. are a form of nonconvulsive generalized seizures without a loss of They are associated with 3/s doublets, each consisting of a
consciousness. typical spike and rounded wave, and lasting about 10 s.”
D. are a form of simple partial seizures without a loss of
Ganong. Review of Medical Physiology 23rd ed. Chap 15, page 233..
consciousness.
E. are a form of convulsive generalized seizures accompanied by
momentary loss of consciousness.
6. A 57-year-old professor at a medical school experienced numerous ANSWER:B
episodes of a sudden loss of muscle tone and an irresistible urge to • “Narcolepsy is characterized by a sudden onset of REM
sleep in the middle of the afternoon. The diagnosis was narcolepsy, sleep, unlike normal sleep which begins with NREM, slow-
which: wave sleep.”
A. is characterized by a sudden onset of NREM sleep. • “Narcolepsy has a familial incidence strongly associated
B. has a familial incidence associated with a class II antigen of the with a class II antigen of the major histocompatibility
major histocompatibility complex. complex on chromosome 6.”
C. may be due to the presence of an excessive number of orexin- • “Brains from humans with narcolepsy often contain fewer
producing neurons in the hypothalamus. hypocretin (orexin)-producing neurons in the
D. is often effectively treated with dopamine receptor agonists. hypothalamus.”
E. is the most common cause of daytime sleepiness. Ganong. Review of Medical Physiology 23rd ed. Chap15 , page 237.
7. Narcolepsy is triggered by abnormalities in the: ANSWER: C
A. skeletal muscle • “Brains from humans with narcolepsy often contain fewer
B. medulla oblongata hypocretin (orexin)-producing neurons in the
C. hypothalamus hypothalamus.”
D. olfactory bulb
E. neocortex Ganong. Review of Medical Physiology 23rd ed. Chap15 , page 237.

(CHAPTER 15) LEARNING, MEMORY, LANGUAGE, & SPEECH


ANSWER: C

1. A 27-year-old man suffered a traumatic brain injury as a result of a


motorcycle accident. He was unconscious and was rushed to the
emergency department of the local hospital. A CT scan was
performed and appropriate interventions were taken. About 6
months later he still had memory deficits. Which of the following is
correctly paired to show the relationship between a brain area and
a type of memory?
A. Hippocampus and implicit memory
B. Neocortex and associative learning
C. Medial temporal lobe and declarative memory
D. Angular gyrus and procedural memory
E. Striatum and priming

Ganong. Review of Medical Physiology 23rd ed. Chap 19, page 290.
2. The optic chiasm and corpus callosum are sectioned in a dog, and ANSWER: E
with the right eye covered, the animal is trained to bark when it
sees a red square. The right eye is then uncovered and the left eye • This is an example of Intercortical Transfer of Memory “If a
covered. The animal will now cat or monkey is conditioned to respond to a visual stimulus
A. fail to respond to the red square because the square does not with one eye covered and then tested with the blindfold
produce impulses that reach the right occipital cortex. transferred to the other eye, it performs the conditioned
B. fail to respond to the red square because the animal has response.”
bitemporal hemianopia. • “This is true even if the optic chiasm has been cut, making the
C. fail to respond to the red square if the posterior commissure is visual input from each eye go only to the ipsilateral cortex.”
also sectioned. • “Partial callosal section experiments indicate that the memory
D. respond to the red square only after retraining. transfer occurs in the anterior commissure of the corpus
E. respond promptly to the red square in spite of the lack of input callosum.”
Ganong. Review of Medical Physiology 23rd ed. Chapter 15, page 239
to the left occipital cortex.

3. A 32-year-old man had medial temporal lobe epilepsy for over 10 ANSWER: C
years. This caused bilateral loss of hippocampal function. As a
• Hippocampus: ability to encode events of the recent past into
result, this individual might be expected to experience a
long-term memory.
A. disappearance of remote memories.
B. loss of working memory. • Neocortical areas: remote memories.
C. loss of the ability to encode events of the recent past into long- • Prefrontal cortex: working memory.
term memory. • Inferior temporal lobe: ability to recall faces and forms but
D. loss of the ability to recall faces and forms but not the ability to not the ability to recall printed or spoken words.
recall printed or spoken words. • Amygdala: production of inappropriate emotional responses
E. production of inappropriate emotional responses when when recalling events of the recent past.
recalling events of the recent past. Ganong. Review of Medical Physiology 23rd ed. Chap19, pg 294, 298, 398 .

TOPNOTCH MEDICAL BOARD PREP MED PHYSIOLOGY GANONG SUPPLEMENT HANDOUT BY THE TOPNOTCH TEAM Page 23 of 46
For inquiries visit www.topnotchboardprep.com.ph or https://web.facebook.com/topnotchmedicalboardprep/
TOPNOTCH MEDICAL BOARD PREP PHYSIOLOGY GANONG SUPPLEMENT HANDOUT BY THE TOPNOTCH TEAM
For inquiries visit www.topnotchboardprep.com.ph or https://www.facebook.com/topnotchmedicalboardprep/

4. A 70-year-old woman fell down a flight of stairs, hitting her head ANSWER: A/ D
on the concrete sidewalk. The trauma caused a severe intracranial
hemorrhage. The symptoms she might experience are dependent • Damage to the angular gyrus in the categorical hemisphere :
on the area of the brain most affected. Which of the following is Anomic aphasia
incorrectly paired? • Damage to Broca area in the categorical hemisphere : Slow
A. Damage to the parietal lobe of the representational speech or Nonfluent Aphasia.
hemisphere : Unilateral inattention and neglect • Inferior parietal lobule: unilateral inattention and neglect
B. Loss of cholinergic neurons in the nucleus basalis of Meynert • Damage to the parietal lobe of the representational
and related areas of the forebrain : Loss of recent memory hemisphere : Asteriognosis and Agnosia.
C. Damage to the mammillary bodies : Loss of recent memory • Loss of cholinergic neurons in the nucleus basalis of Meynert
D. Damage to the angular gyrus in the categorical hemisphere : and related areas of the forebrain : Loss of recent memory
Nonfluent aphasia • Damage to the mammillary bodies : Loss of recent memory
E. Damage to Broca area in the categorical hemisphere : Slow Ganong. Review of Medical Physiology 23rd ed. Chapter , pg 293, 296, 298
speech
ANSWER: B
5. The representational hemisphere is better than the categorical
hemisphere at • “Lesions in the categorical hemisphere produce language
A. language functions. disorders, whereas extensive lesions in the representational
B. recognition of objects by their form. hemisphere do not. Instead, lesions in the representational
C. understanding printed words. hemisphere produce astereognosis—the inability to identify
D. understanding spoken words. objects by feeling them.”
E. mathematical calculations. Ganong. Review of Medical Physiology 23rd ed. Chapter 19, page 298.

6. A 67-year-old woman suffered a stroke that damaged the ANSWER: D


posterior end of the superior temporal gyrus. A lesion of Wernicke
area in the categorical hemisphere causes her to:
A. lose her short-term memory. • Lesion in Wernicke’s area: the speech itself is normal and
B. experience nonfluent aphasia in which she speaks in a slow, sometimes the patients talk excessively. However, what they
halting voice. say is full of jargon and neologisms that make little sense.
C. experience déjà vu.
D. talk rapidly but make little sense, which is characteristic of Ganong. Review of Medical Physiology 23rd ed. Chapter 19, page 297 ..
fluent aphasia.
E. lose the ability to recognize faces, w/c is called prosopagnosia
7. Which of the following is most likely involved in production of LTP? ANSWER: D
A. NO release, activation of NMDA receptors, and membrane
hyperpolarization
B. Decreased Ca2+ in presynaptic or postsynaptic neurons, • “Long-term potentiation (LTP) is a rapidly developing
activation of NMDA receptors, and membrane depolarization persistent enhancement of the postsynaptic potential
C. Activation of NMDA receptors, NO-induced reduction in response to presynaptic stimulation after a brief period of
glutamate release in a presynaptic neuron, and membrane rapidly repeated stimulation of the presynaptic neuron.”
depolarization • Increased Ca2+ in presynaptic or postsynaptic neurons,
D. Increased Ca2+ in presynaptic or postsynaptic neurons, activation of NMDA receptors, and membrane
activation of NMDA receptors, and membrane depolarization depolarization.
E. NO-induced increase in glutamate release in a presynaptic
Ganong. Review of Medical Physiology 23rd ed. Chapter 19 , page 291.
neuron, activation of non-NMDA receptors, membrane
hyperpolarization
8. A 79-year-old woman has been experiencing difficulty finding her ANSWER: B
way back home after her morning walks. Her husband has also
noted that she takes much longer to do routine chores around the
home and often appears to be confused. He is hoping that this is
just due to “old age” but fears it may be a sign of Alzheimer • The cytopathologic hallmarks of Alzheimer disease are
disease. Which of the following is the definitive sign of this dse? intracellular neurofibrillary tangles, made up in part of
A. Loss of short-term memory hyperphosphorylated form of the tau protein that normally
B. The presence of intracellular neurofibrillary tangles and binds to microtubules, and extracellular senile plaques,
extracellular neuritic plaques with a core of β-amyloid peptides which have a core of β-amyloid peptides (AΒ ) surrounded
C. A mutation in genes for amyloid precursor protein (APP) on by altered nerve fibers and reactive glial cells.
chromosome 21
Ganong. Review of Medical Physiology 23rd ed. Chapter 19 , page 294 .
D. Rapid reversal of symptoms with the use of
acetylcholinesterase inhibitors
E. A loss of cholinergic neurons in the nucleus basalis of Meynert
9. The representational hemisphere: ANSWER: A
A. is the right cerebral hemisphere in most right-handed
individuals • In 96% of right-handed individuals, who constitute 91% of
B. is the left cerebral hemisphere in most left-handed individuals the human population, the left hemisphere is the dominant
C. includes the part of the brain concerned with language or categorical hemisphere; and the right hemisphere is the
functions non-dominant or the representational hemisphere,
D. is the site of lesions in most patients with aphasia
E. is morphologically identical to the opposite Ganong. Review of Medical Physiology 23rd ed. Chapter 19 , page 295.
nonrepresentational hemisphere

TOPNOTCH MEDICAL BOARD PREP MED PHYSIOLOGY GANONG SUPPLEMENT HANDOUT BY THE TOPNOTCH TEAM Page 24 of 46
For inquiries visit www.topnotchboardprep.com.ph or https://web.facebook.com/topnotchmedicalboardprep/
TOPNOTCH MEDICAL BOARD PREP PHYSIOLOGY GANONG SUPPLEMENT HANDOUT BY THE TOPNOTCH TEAM
For inquiries visit www.topnotchboardprep.com.ph or https://www.facebook.com/topnotchmedicalboardprep/

SECTION 3: ENDOCRINE AND REPRODUCTIVE PHYSIOLOGY


(CHAPTER 17) HYPOTHALAMIC REGULATION OF HORMONAL FUNCTIONS
ANSWER: B
• When the effective osmotic pressure of the plasma rises,
vasopressin secretion is increased and the thirst
mechanism is stimulated; water is retained in the body,
diluting the hypertonic plasma; and water intake is
1. Thirst is stimulated by increased
A. increases in plasma osmolality and volume.
B. an increase in plasma osmolality and a decrease in plasma volume.
C. a decrease in plasma osmolality and an increase in plasma volume.
D. decreases in plasma osmolality and volume.
E. injection of vasopressin into the hypothalamus.

Ganong. Review of Medical Physiology 23rd ed. Chapter 39 , page 665


2. When an individual is naked in a room in which the air temperature is ANSWER: E
21°C (69.8°F) and the humidity 80%, the greatest amount of heat is
lost from the body by • When an individual is in a cold environment, heat is lost by
A. elevated metabolism. conduction to the surrounding air and by radiation to
B. respiration. cool objects in the vicinity.
C. urination.
D. vaporization of sweat. Ganong. Review of Medical Physiology 23rd ed. Chapter 18, page 284
E. radiation and conduction.
In questions 3–8, select A if the item is associated with (a) below, B if the ANSWER: 3. B, 4. A, 5. A, 6. B, 7. D, 8. D
item is associated with (b) below, C if the item is associated with both (a) • V1A and V1B receptors act through phosphatidylinositol
and (b), and D if the item is associated with neither (a) nor (b). hydrolysis to increase the intracellular Ca2+ concentration.
(a) V1A vasopressin receptors The V2 receptors act through Gs to increase cyclic
(b) V2 vasopressin receptors adenosine 3',5'-monophosphate (cAMP) levels.
3. Activation of Gs • V2 receptors and involves the insertion of proteins called
4. Vasoconstriction water channels/aquaporins, into the apical (luminal)
5. Increase in intracellular inositol triphosphate membranes of the principal cells of the collecting ducts.
6. Movement of aquaporin • V1A receptors mediate the vasoconstrictor effect of
7. Proteinuria vasopressin,
8. Milk ejection Ganong. Review of Medical Physiology 23rd ed. Chap39 , page 666..

(CHAPTER 18) THE PITUITARY GLAND


ANSWER: E
• The hormones of the posterior lobe (PL) are released into
the general circulation from the endings of supraoptic
and paraventricular neurons, whereas hypophysiotropic
hormones of the anterior pituitary are secreted into the
1. A neuroscientist is studying communication between the portal hypophysial circulation , within the median
hypothalamus and pituitary in a rat model. She interrupts blood flow eminence from the endings of arcuate and other
through the median eminence and then measures circulating levels of hypothalamic neurons.
pituitary hormones following appropriate physiologic stimulation.
Secretion of which of the following hormones will be unaffected by the
experimental manipulation?
A. Growth hormone
B. Prolactin
C. Thyroid-stimulating hormone
D. Follicle-stimulating hormone
E. Vasopressin

Ganong. Review of Medical Physiology 23rd ed. Chap 18 , page 274.


2. Which of the following pituitary hormones is an opioid peptide? ANSWER: E
A. α-Melanocyte-stimulating hormone (α-MSH) • Proopiomelanocortin (POMC) is hydrolyzed to
B. β-MSH corticotropin-like intermediate-lobe peptide (CLIP), γ-
C. ACTH LPH, and appreciable quantities of β –endorphin, which is
D. Growth hormone an opioid peptide
E. β-Endorphin Ganong. Review of Medical Physiology 23rd ed. Chap22 , page 352.
3. During childbirth, a woman suffers a serious hemorrhage and goes into ANSWER: E
shock. After she recovers, she displays symptoms of hypopituitarism.
• Thus, patient with hypopituitarism is that of a lethargic
Which of the following will not be expected in this patient?
person (from lack of thyroid hormones) who is gaining
A. Cachexia
weight (not cachexia) (because of lack of fat mobilization
B. Infertility
by growth, adrenocorticotropic, adrenocortical, and
C. Pallor
thyroid hormones) and has lost all sexual functions.
D. Low basal metabolic rate
E. Intolerance to stress Ganong. Review of Medical Physiology 23rd ed. Chapter 75 , page 926.

TOPNOTCH MEDICAL BOARD PREP MED PHYSIOLOGY GANONG SUPPLEMENT HANDOUT BY THE TOPNOTCH TEAM Page 25 of 46
For inquiries visit www.topnotchboardprep.com.ph or https://web.facebook.com/topnotchmedicalboardprep/
TOPNOTCH MEDICAL BOARD PREP PHYSIOLOGY GANONG SUPPLEMENT HANDOUT BY THE TOPNOTCH TEAM
For inquiries visit www.topnotchboardprep.com.ph or https://www.facebook.com/topnotchmedicalboardprep/

4. A scientist finds that infusion of growth hormone into the median ANSWER: C
eminence of the hypothalamus in experimental animals inhibits the
secretion of growth hormone and concludes that this proves that • Growth hormone secretion is under feedback control,
growth hormone feeds back to inhibit GHRH secretion. Do you accept like the secretion of other anterior pituitary hormones. It
this conclusion? acts on the hypothalamus to antagonize GHRH release.
A. No, because growth hormone does not cross the blood-brain Growth hormone also increases circulating IGF-I, and IGF-I
barrier in turn exerts a direct inhibitory action on growth
B. No, because the infused growth hormone could be stimulating hormone secretion from the pituitary. It also stimulates
dopamine secretion. somatostatin secretion.
C. No, because substances placed in the median eminence could be • GH in systemic circulation is the one responsible for the
transported to the anterior pituitary. feedback, and not the one in the portal system.
D. Yes, because systemically administered growth hormone inhibits
growth hormone secretion. Ganong. Review of Medical Physiology 23rd ed. Chapter 24, page 384
E. Yes, because growth hormone binds GHRH, inactivating it.
ANSWER: B
5. The growth hormone receptor • Growth hormone receptor:
A. activates Gs. o Activates PLC
B. requires dimerization to exert its effects. o Requires dimerization to exert its effects.
C. must be internalized to exert its effects. o Is not internalized to exert its effects, only the JAK
D. resembles the IGF-I receptor. STAT migrates in the nucleus.
E. resembles the ACTH receptor. o resembles the prolactin receptor.
Ganong. Review of Medical Physiology 23rd ed. Chapter 24, page 381,
382; chapter 25, page 401
ANSWER: E
6. Which of the following hormones exerts the least effect on growth?
A. growth hormone • Growth overall is a complex phenomenon that is affected
B. testosterone not only by growth hormone and somatomedins, but
C. T4 also, as would be predicted by the previous discussion, by
D. insulin thyroid hormones, androgens, estrogens,
E. Vasopressin glucocorticoids, and insulin .
Ganong. Review of Medical Physiology 23rd ed. Chapter24 , page 384.

(CHAPTER 19) THE THYROID GLAND


1. A 40-year-old woman comes to her primary care clinician complaining ANSWER: C
of nervousness and an unexplained weight loss of 20 pounds over the
past 3 months despite her impression that she is eating all the time.
On physical examination, her eyes are found to be protruding, her skin • “Thyroid gland is increased to two to three times normal
is moist and warm, and her fingers have a slight tremor. Compared to size, with tremendous hyperplasia and infolding of the
a healthy individual, a biopsy of her thyroid gland would most likely follicular cell lining into the follicles, so that the number of
reveal which of the following: cells is increased greatly. Also, each cell increases its rate
A. Decreased numbers of reabsorption lacunae of secretion severalfold.” This leads to a decrease in the
B. Decreased evidence of endocytosis cross-sectional area occupied by colloid
C. A decrease in the cross-sectional area occupied by colloid
Guyton.Textbook of Medical Physiology 11th ed. Chapter76, page 940.
D. Increased levels of NIS in the basolateral membrane of thyrocytes
E. Decreased evidence of lysosomal activity
ANSWER: B
2. Which of the following is not essential for normal biosynthesis of • Iodide (not ferritin) undergoes a process referred to as
thyroid hormones? organification. First, it is oxidized to iodine, and then
A. Iodine incorporated into the carbon 3 position of tyrosine
B. Ferritin residues that are part of the thyroglobulin, which is a
C. Thyroglobulin glycoprotein.
D. Protein synthesis • TSH, there are increases in iodide binding; synthesis of T3,
E. TSH T4, and iodotyrosines; secretion of thyroglobulin into the
colloid.
Ganong. Review of Medical Physiology 23rd ed. Chap20, page 304.
ANSWER: E
3. Increasing intracellular I– due to the action of NIS is an example of
A. Endocytosis • The basolateral membranes of thyrocytes facing the
B. Passive diffusion capillaries contain a Na+/I–symporter that transports
C. Na+ and K+ cotransport two Na+ ions and one I ion into the cell with each cycle,
D. Primary active transport against the The process involved is secondary active
E. Secondary active transport transport electrochemical gradient for I –.
Ganong. Review of Medical Physiology 23rd ed. Chapter , page .
4. The metabolic rate is least affected by an increase in the plasma level of ANSWER: C
A. TSH
B. TRH • Patients with elevated or decreased concentrations of
C. TBG binding proteins, particularly TBG, are typically neither
D. Free T4 hyper-nor hypothyroid; that is, they are euthyroid.
Ganong. Review of Medical Physiology 23rd ed. Chapter 20, page 306
E. Free T3
5. In which of the following conditions is it most likely that the TSH ANSWER: C
response to TRH will be reduced? • TRH directly affects the anterior pituitary gland cells to
A. Hypothyroidism due to tissue resistance to thyroid hormone increase their output of TSH. However, high level of
B. Hypothyroidism due to disease destroying the thyroid gland thyroid hormone secretion caused by thyroid-
C. Hyperthyroidism due to circulating antithyroid antibodies with stimulating immunoglobulin in turn suppresses anterior
TSH activity pituitary formation of TSH.
D. Hyperthyroidism due to diffuse hyperplasia of thyrotropes of the
anterior pituitary Guyton.Textbook of Medical Physiology 11th ed.Chapter 76, pages
E. Iodine deficiency 938, 940

TOPNOTCH MEDICAL BOARD PREP MED PHYSIOLOGY GANONG SUPPLEMENT HANDOUT BY THE TOPNOTCH TEAM Page 26 of 46
For inquiries visit www.topnotchboardprep.com.ph or https://web.facebook.com/topnotchmedicalboardprep/
TOPNOTCH MEDICAL BOARD PREP PHYSIOLOGY GANONG SUPPLEMENT HANDOUT BY THE TOPNOTCH TEAM
For inquiries visit www.topnotchboardprep.com.ph or https://www.facebook.com/topnotchmedicalboardprep/

6. Hypothyroidism due to disease of the thyroid gland is associated with ANSWER: A


increased plasma levels of:
A. Cholesterol • Thyroid hormones lower circulating cholesterol levels.
B. Albumin Therefore in hypothyroidism, levels of cholesterol is
C. RT3 expected to increase.
D. Iodide
Ganong. Review of Medical Physiology 23rd ed. Chap20 , page 313.
E. TBG
ANSWER: D
7. A young woman has puffy skin and a hoarse voice. Her plasma TSH
concentration is low but increases markedly when she is given TRH. • Synthesis and secretion of thyroid hormones is
She probably has: stimulated by thyroid-stimulating hormone (TSH) from
A. hyperthyroidism due to a thyroid tumor. the pituitary, which in turn is released in response to
B. hypothyroidism due to a primary abnormality in the thyroid gland. thyrotropin-releasing hormone (TRH) from the
C. hypothyroidism due to a primary abnormality in the pituitary hypothalamus.
gland. • Problems in the hypothalamus would result in low levels
D. hypothyroidism due to a primary abnormality in the hypothalamus. of TRH and therefore exogenous administration of TRH
E. hyperthyroidism due to a primary abnormality in the hypothalamus. would increase the level of TSH.
Ganong. Review of Medical Physiology 23rd ed. Chap20 , page 308..
ANSWER: A
8. The enzyme primarily responsible for the conversion of T4 to T3 in the
periphery is: • D1 deiodinase (liver, kidneys, thyroid, and pituitary)
A. D1 thyroid deiodinase primarily responsible for monitoring the formation of T3
B. D2 thyroid deiodinase from T4 in the periphery.
C. D3 thyroid deiodinase • D2 deiodinase (brain, pituitary, and brown fat) produces
D. Thyroid peroxidase a supply of T3 to neurons.
E. None of the above • D3 deiodinase (brain and in reproductive tissues).
Ganong. Review of Medical Physiology 23rd ed. Chap20 , page 306 ..
ANSWER:
• Cyclic Adenosine Monophosphate mediates the
9. Which of the following would be least affected by injections of TSH? stimulatory effect of TSH:
A. Thyroidal uptake of iodine 1. Increased proteolysis of the thyroglobulin
B. Synthesis of thyroglobulin 2. Increased activity of the iodide pump
C. Cyclic adenosine monophosphate (cAMP) in thyroid cells 3. Increased iodination of tyrosine
D. Cyclic guanosine monophosphate (cGMP) in thyroid cells 4. Increased size and increased secretory activity of the
E. Size of the thyroid thyroid cells
5. Increased number of thyroid cells
Guyton.Textbook of Medical Physiology 11th ed. Chap 76 , page 938..
10. Thyroid hormone receptors bind to DNA in which of the ff forms? ANSWER: C
A. A heterodimer with the prolactin receptor • Thyroid receptors bind to DNA as monomers,
B. A heterodimer with the growth hormone receptor homodimers, and heterodimers with other nuclear
C. A heterodimer with the retinoid X receptor receptors, particularly the retinoid X receptor.
D. A heterodimer with the insulin receptor
E. A heterodimer with the progesterone receptor Ganong. Review of Medical Physiology 23rd ed. Chap 20, page 308

(CHAPTER 20) THE ADRENAL MEDULLA & ADRENAL CORTEX


1. Which of the following is produced only by large amounts of ANSWER:
glucocorticoids? • “Glucocorticoids inhibit the inflammatory response to
A. Normal responsiveness of fat depots to norepinephrine tissue injury these effects require high levels of
B. Maintenance of normal vascular reactivity circulating glucocorticoids and cannot be produced by
C. Increased excretion of a water load administering steroids without producing the other
D. Inhibition of the inflammatory response manifestations of glucocorticoid excess.”
E. Inhibition of ACTH secretion Ganong. Review of Medical Physiology 23rd ed. Chapter 22 , page 351
ANSWER: B
2. Which of the following are incorrectly paired?
• Gluconeogenesis : Cortisol
A. Gluconeogenesis : Cortisol
B. Free fatty acid mobilization : Dehydroepiandrosterone • Puberty: Dehydroepiandrosterone
C. Muscle glycogenolysis : Epinephrine • Muscle glycogenolysis : Epinephrine
D. Kaliuresis : Aldosterone • Kaliuresis : Aldosterone
E. Hepatic glycogenesis : Insulin • Hepatic glycogenesis : Insulin
Ganong. Review of Medical Physiology 23rd ed. Chapter 22 , page 348
ANSWER: E
3. Which of the following hormones has the shortest plasma half-life?
• Norepinephrine :2 minutes
A. Corticosterone
B. Renin • Renin: 15 minutes
C. Dehydroepiandrosterone • Aldosterone: 20 minutes
D. Aldosterone • Corticosterone: 60-90 minutes
E. Norepinephrine • Dehydroepiandrosterone: 7-22 hours
Ganong. Review of Medical Physiology 23rd ed. Chap77, page 947.
4. Mole for mole, which of the following has the greatest effect on Na+ ANSWER: D
excretion?
A. Progesterone • “Changes in Na+ excretion occur too rapidly to be due
B. Cortisol solely to changes in aldosterone secretion. Na+ excretion
C. Vasopressin is decreased within a few minutes, and this rapid change
D. Aldosterone in Na+ excretion occurs in adrenalectomized subjects.”
Ganong. Review of Medical Physiology 23rd ed. Chap 39, page 670.
E. Dehydroepiandrosterone

TOPNOTCH MEDICAL BOARD PREP MED PHYSIOLOGY GANONG SUPPLEMENT HANDOUT BY THE TOPNOTCH TEAM Page 27 of 46
For inquiries visit www.topnotchboardprep.com.ph or https://web.facebook.com/topnotchmedicalboardprep/
TOPNOTCH MEDICAL BOARD PREP PHYSIOLOGY GANONG SUPPLEMENT HANDOUT BY THE TOPNOTCH TEAM
For inquiries visit www.topnotchboardprep.com.ph or https://www.facebook.com/topnotchmedicalboardprep/
ANSWER: 20
5. Mole for mole, which of the following has the greatest effect on
plasma osmolality? • The injection of extremely minute quantities of ADH—as
A. Progesterone small as 2 nanograms—can cause decreased excretion
B. Cortisol of water by the kidneys (antidiuresis) but allows
C. Vasopressin continuing excretion of electrolytes, thus decreasing
D. Aldosterone the plasma osmolarity back toward normal.
E. Dehydroepiandrosterone Guyton.Textbook of Medical Physiology 11th ed. Chapter 58, page
733. Chapter 75 , page 928.
ANSWER: D
6. The secretion of which of the following would be least affected by a • In cases when there is decrease in extracellular fluid
decrease in extracellular fluid volume? volume (e.i. hemorrhage) aside from activating the renin
A. CRH –angiotensin-aldosterone system and vasopressin, the
B. Arginine vasopressin physiologic stress also increases release of CRH which
C. Dehydroepiandrosterone stimulates ACTH which in turn will also affect the output
D. Estrogens of aldosterone as well as that of glucocorticoids and
E. Aldosterone dehydroepiandrosterone
Ganong. Review of Medical Physiology 23rd ed. Chap22 , page 356..
ANSWER: D
7. A young man presents with a blood pressure of 175/110 mm Hg. He is • The patient had glucocorticoid-remediable
found to have a high circulating aldosterone but a low circulating aldosteronism (GRA) which is an autosomal dominant
cortisol. Glucocorticoid treatment lowers his circulating aldosterone disorder in which the increase in aldosterone secretion
and lowers his blood pressure to 140/85 mm Hg. He probably has an produced by ACTH is no longer transient.
abnormal: • The hypersecretion of aldosterone and the
A. 17α-hydroxylase. accompanying hypertension are remedied when ACTH
B. 21β-hydroxylase. secretion is suppressed by administering
C. 3β-hydroxysteroid dehydrogenase. glucocorticoids.
D. aldosterone synthase. • The genes encoding aldosterone synthase and 11β-
E. cholesterol desmolase. hydroxylase are 95% identical and are close together on
chromosome 8.
Ganong. Review of Medical Physiology 23rd ed. Chapter 22, page 356
ANSWER: A
8. A 32-year-old woman presents with a blood pressure of 155/96 mm
Hg. In response to questioning, she admits that she loves licorice and • Patient had apparent mineralocorticoid excess (AME)
eats some at least three times a week. She probably has a low level of secondaryto prolonged ingestion of licorice which
A. type 2 11β-hydroxysteroid dehydrogenase activity. contains glycyrrhetinic acid, that inhibits 11β-
B. ACTH. hydroxysteroid dehydrogenase type 2.
C. 11β-hydroxylase activity. • If 11β-hydroxysteroid dehydrogenase type 2 is
D. glucuronyl transferase. inhibited or absent, cortisol has marked
E. norepinephrine. mineralocorticoid effects.
Ganong. Review of Medical Physiology 23rd ed. Chapter 22 , page 355
ANSWER:
• It increases transport of ENaCs from the cytoplasm to the
cell membrane.
9. In its action in cells, aldosterone • Aldosterone binds to a cytoplasmic receptor, and the
A. increases transport of ENaCs from the cytoplasm to the cell receptor-hormone complex moves to the nucleus.
membrane. • Evidence is accumulating that aldosterone also binds to
B. does not act on the cell membrane. the cell membrane and increases the activity of
C. binds to a receptor excluded from the nucleus. membrane Na+–K+ exchangers.
D. may activate a heat shock protein. • Among the genes activated by aldosterone is the gene for
E. also binds to glucocorticoid receptors. serum- and glucocorticoid regulated kinase (sgk) , a
serine-threonine protein kinase.
• Aldosterone doesn’t bind to glucocorticoid receptors.
Ganong. Review of Medical Physiology 23rd ed. Chapter 22 , page 355

(CHAPTER 21) HORMONAL CONTROL OF CALCIUM, & PHOSPHATE METABOLISM & THE PHYSIOLOGY OF BONE
1. A patient with parathyroid deficiency 10 days after inadvertent ANSWER: C
damage to the parathyroid glands during thyroid surgery would
probably have:
A. low plasma phosphate and Ca2+ levels and tetany. • After parathyroidectomy:
B. low plasma phosphate and Ca2+ levels and tetanus. o Decline in the plasma calcium level.
C. a low plasma Ca2+ level, increased muscular excitability, and o Neuromuscular hyperexcitability appear.
spasm of the muscles of the upper extremity (Trousseau sign). o Hypocalcemic tetany.(chvostek’s and trousseau’s).
D. high plasma phosphate and Ca2+ levels and bone o Plasma phosphate levels usually rise.
demineralization.
Ganong. Review of Medical Physiology 23rd ed. Chapter 23 , page 368
E. increased muscular excitability, a high plasma Ca2+ level, and
bone demineralization.
2. In an experiment, a rat is infused with a small volume of a calcium ANSWER: E
chloride solution, or sodium chloride as a control. Compared to the
control condition, which of the following would result from the • When the plasma Ca2+ level is high, little 1,25-
calcium load? dihydroxycholecalciferol is produced and the kidneys
A. Bone demineralization produce the relatively inactive metabolite 24,25-
B. Increased formation of 1,25-dihydroxycholecalciferol dihydroxycholecalciferol instead.
C. Decreased secretion of calcitonin
D. Decreased blood coagulability Ganong. Review of Medical Physiology 23rd ed. Chapter 23 , page 366
E. Increased formation of 24,25-dihydroxycholecalciferol

TOPNOTCH MEDICAL BOARD PREP MED PHYSIOLOGY GANONG SUPPLEMENT HANDOUT BY THE TOPNOTCH TEAM Page 28 of 46
For inquiries visit www.topnotchboardprep.com.ph or https://web.facebook.com/topnotchmedicalboardprep/
TOPNOTCH MEDICAL BOARD PREP PHYSIOLOGY GANONG SUPPLEMENT HANDOUT BY THE TOPNOTCH TEAM
For inquiries visit www.topnotchboardprep.com.ph or https://www.facebook.com/topnotchmedicalboardprep/
ANSWER: D
• Kidneys: converts 25-Hydroxycholecalciferol into
1,25-Dihydroxycholecalciferol (active form) using
3. Which of the following is not involved in regulating plasma Ca2+ levels? 1α-Hydroxylase and 24,25-Dihydroxycholecalciferol
A. Kidneys (inactive form) using 24-Hydroxylase.
B. Skin • Skin: sunlight activates 7-Dehydrocholesterol into
C. Liver Vitamin D3 (cholecalciferol)
D. Lungs • Liver: converts Vitamin D3 (cholecalciferol) to 25-
E. Intestine Hydroxycholecalciferol with the use of 25-Hydroxylase
• Intestine: increases absorption of calcium in response to
increased vitamin D level
Ganong. Review of Medical Physiology 23rd ed. Chapter 23, page 366
4. 1,25-Dihydroxycholecalciferol affects intestinal Ca2+ absorption ANSWER: A
through a mechanism that: • 1,25 dihydroxycholecalciferol stimulates the expression of
A. includes alterations in the activity of genes. a number of gene products involved in calcium transport
B. activates adenylyl cyclase. (Ca2+–ATPase) and handling via its receptor
C. decreases cell turnover. (calbindin) which acts as a transcriptional regulator in its
D. changes gastric acid secretion. ligand-bound form.
E. involves degradation of apical calcium channels. Ganong. Review of Medical Physiology 23rd ed. Chapter 23 , page 365

5. Which of the following would you expect to find in a patient whose ANSWER: C
diet has been low in calcium for 2 months? • Increased formation of 1,25 dihydroxycholecalciferol
A. Increased formation of 24,25-dihydroxycholecalciferol • Increased amounts of calcium-binding protein in
B. Decreased amounts of calcium-binding protein in intestinal intestinal epithelial cells
epithelial cells • Increased parathyroid hormone secretion
C. Increased parathyroid hormone secretion • A low plasma calcitonin concentration
D. A high plasma calcitonin concentration • Decreased plasma phosphate
E. Increased plasma phosphate Ganong. Review of Medical Physiology 23rd ed. Chapter 23 , page 368
6. A mouse is engineered to lack a transcription factor necessary for the ANSWER: D
normal development of osteoclasts. Compared to normal littermate • Osteoclast forms acid proteases using two types of
mice, which of the following would be reduced in the knock-out substances: (1) proteolytic enzymes, released from the
animals? lysosomes of the osteoclasts, and (2) several acids,
A. Phosphate deposition in trabecular bone including citric acid and lactic acid, released from the
B. Hydroxyapatite levels in bone mitochondria and secretory vesicles.
C. Osteoblast proliferation • The enzymes digest or dissolve the organic matrix of the
D. Secretion of acid proteases bone, and the acids cause solution of the bone salts.
E. Bone collagen Ganong. Review of Medical Physiology 23rd ed. Chapter 79 , page 982
7. The skeleton of a normal male college student would be expected to ANSWER: E
display which of the following features, relative to that of his 7-year-
old brother? • The patient age group (college) approximates the time of
A. Merging of cortical bone and trabecular bone. epiphyseal closure in men. Linear bone growth can occur
B. Differentiation of osteoclasts and osteoblasts. as long as the epiphyses are separated from the shaft of
C. An extended amount of proliferating cartilage that contributes to the bone, but such growth ceases after the epiphyses
bone elongation. unite with the shaft (epiphysial closure).
D. A meeting of the lacunae with the trabecular bone.
Ganong. Review of Medical Physiology 23rd ed. Chapter 23, page 371
E. Epiphyses that are united with the bone shaft.
8. At epiphysial closure ANSWER: E
A. cortical bone and trabecular bone merge • Linear bone growth can occur as long as the epiphyses are
B. osteoclasts and osteoblasts undergo differentiation separated from the shaft of the bone, but such growth
C. there is an extended amount of prolifering cartilage that ceases after the epiphyses unite with the shaft
contributes to bone elongation (epiphysial closure).
D. lacunae meet the trabecular bone
E. ephyses unit with the shaft to end normal linear bone growth Ganong. Review of Medical Physiology 23rd ed. Chapter 23, page 371

(CHAPTER 22) REPRODUCTIVE DEVELOPMENT & FUNCTION OF THE FEMALE REPRODUCTIVE SYSTEM
ANSWER:
1. If a young woman has high plasma levels of T3, cortisol, and renin • The cytotrophoblast of the human chorion contains
activity but her blood pressure is only slightly elevated and she has no prorenin
symptoms or signs of thyrotoxicosis or Cushing syndrome, the most • The rate of adrenocortical secretion of the
likely explanation is that glucocorticoids is moderately increased throughout
A. she has been treated with TSH and ACTH. pregnancy
B. she has been treated with T3 and cortisol. • The increased thyroxine (as well as T3) production is
C. she is in the third trimester of pregnancy. caused at least partly by a thyrotropic effect of human
D. she has an adrenocortical tumor. chorionic gonadotropin.
E. she has been subjected to chronic stress. Ganong. Review of Medical Physiology 23rd ed. Chapter 25, page 425
Guyton.Textbook of Medical Physiology 11th ed.Chap82 , page 1034.

2. In humans, fertilization usually occurs in the: ANSWER: D


A. vagina.
B. cervix • In humans, fertilization of the ovum by the sperm usually
C. uterine cavity. occurs in the ampulla of the uterine tube.
D. uterine tubes.
Ganong. Review of Medical Physiology 23rd ed. Chapter 25, page 423
E. abdominal cavity.

TOPNOTCH MEDICAL BOARD PREP MED PHYSIOLOGY GANONG SUPPLEMENT HANDOUT BY THE TOPNOTCH TEAM Page 29 of 46
For inquiries visit www.topnotchboardprep.com.ph or https://web.facebook.com/topnotchmedicalboardprep/
TOPNOTCH MEDICAL BOARD PREP PHYSIOLOGY GANONG SUPPLEMENT HANDOUT BY THE TOPNOTCH TEAM
For inquiries visit www.topnotchboardprep.com.ph or https://www.facebook.com/topnotchmedicalboardprep/
ANSWER: C
• Relaxin is a polypeptide hormone that is produced in the
3. Which of the following is not a steroid? corpus luteum, uterus, placenta, and mammary glands
A. 17α-hydroxyprogesterone in women and in the prostate gland in men.
B. Estrone • During pregnancy, it relaxes the pubic symphysis and
C. Relaxin other pelvic joints and softens and dilates the uterine
D. Pregnenolone cervix.
E. Etiocholanolone • In men, it is found in semen, where it may help maintain
sperm motility and aid in sperm penetration of the ovum
Ganong. Review of Medical Physiology 23rd ed. Chapter 25 , page 420
ANSWER: A
• fetal ACTH  DHEAS  estrogen
4. Which of the following probably triggers the onset of labor? • This makes the uterus :
A. ACTH in the fetus o more excitable (increases the number of gap junctions
B. ACTH in the mother between myometrial cells)
C. Prostaglandins o production of more prostaglandins, which in turn
D. Oxytocin cause uterine contractions
E. Placental renin • In early labor, the oxytocin concentration in maternal
plasma is not elevated from the prelabor value.
Ganong. Review of Medical Physiology 23rd ed. Chapter 25, page 425
ANSWER: E
5. Full development and function of the seminiferous tubules require: • Factors that affects spermatogenesis in the seminiferous
A. somatostatin tubules:
B. LH o FSH acts on the Sertoli cells to facilitate the last stages
C. Oxytocin of spermatid maturation.
D. FSH o Androgen affects maturation from spermatids to
E. androgens and FSH spermatozoa depends on androgen acting on the
Sertoli cells.
Ganong. Review of Medical Physiology 23rd ed. Chapter 25 , page 404
ANSWER: A
• fetal ACTH  DHEAS  estrogen
6. Which of the following probably triggers the onset of labor? • This makes the uterus :
A. ACTH in the fetus o more excitable (increases the number of gap junctions
B. ACTH in the mother between myometrial cells)
C. Prostaglandins o production of more prostaglandins, which in turn
D. Oxytocin cause uterine contractions
E. Placental renin • In early labor, the oxytocin concentration in maternal
plasma is not elevated from the prelabor value.
Ganong. Review of Medical Physiology 23rd ed. Chapter 25, page 425

(CHAPTER 23) FUNCTION OF THE MALE REPRODUCTIVE SYSTEM


ANSWER: E
1. Full development and function of the seminiferous tubules require • Factors that affects spermatogenesis in the seminiferous
A. somatostatin. tubules:
B. LH. o FSH acts on the Sertoli cells to facilitate the last stages
C. oxytocin. of spermatid maturation.
D. FSH. o Androgen affects maturation from spermatids to
E. androgens and FSH. spermatozoa depends on androgen acting on the
Sertoli cells.
Ganong. Review of Medical Physiology 23rd ed. Chapter 25 , page 404
2. In human males, testosterone is produced mainly by the: ANSWER: A
A. Leydig cells.
B. Sertoli cells. • Testosterone is synthesized from cholesterol in the
C. seminiferous tubules. Leydig cells and is also formed from androstenedione
D. epididymis. secreted by the adrenal cortex.
Ganong. Review of Medical Physiology 23rd ed. Chapter 25, page 406
E. vas deferens.
3. Nitric oxide synthase contributes to erection by:’ ANSWER: C
A. raising cAMP levels that relax smooth muscles and increase blood
flow.
B. blocking phosphodiesterases to increase cGMP levels that release • Nitric Oxide Synthase  nitric oxide production
smooth muscle and increase blood flow. activates guanylyl cyclase  increased production of
C. activating soluble guanylyl cyclases to increase cGMP levels that cyclic GMP  potent vasodilator.
relax smooth muscle and increase blood flow.
Ganong. Review of Medical Physiology 23rd ed. Chap25 , page 405.
D. raising intracellular Ca2+ concentrations that relax smooth
muscles and increase blood flow.
ANSWER: B
• Testosterone is not produced by LH, it is produced by
4. Testosterone is produced pregnenolone precursor in the Leydig cells, it is also not
A. in the testes after reduction of dihydrotestosterone. due to reduction of dihydrotestosterone:
B. in Leydig cells from cholesterol and pregnenolone precursors. • LH stimulates  Leydig cells  converts cholesterol to
C. by LH in Leydig cells. pregnenolone DHEAS Testosterone.
D. as a precursor for several membrane lipids. • Dihydrotestosterone (DHT) is formed by the reduction
of testosterone with 5α-reductase. DHT amplifies the
action of testosterone in target tissues.
Ganong. Review of Medical Physiology 23rd ed. Chap25 , page 406.

TOPNOTCH MEDICAL BOARD PREP MED PHYSIOLOGY GANONG SUPPLEMENT HANDOUT BY THE TOPNOTCH TEAM Page 30 of 46
For inquiries visit www.topnotchboardprep.com.ph or https://web.facebook.com/topnotchmedicalboardprep/
TOPNOTCH MEDICAL BOARD PREP PHYSIOLOGY GANONG SUPPLEMENT HANDOUT BY THE TOPNOTCH TEAM
For inquiries visit www.topnotchboardprep.com.ph or https://www.facebook.com/topnotchmedicalboardprep/
(CHAPTER 24) ENDOCRINE FUNCTIONS OF THE PANCREAS & REGULATION OF CARBOHYDRATE METABOLISM
ANSWER:
• Pancreatic endocrine and exocrine cells structure:
1. Which of the following are incorrectly paired? o A cells: glucagon
A. B cells: insulin o B cells: insulin
B. D cells: somatostatin o D cells: somatostatin
C. A cells: glucagons o F cells: pancreatic polypeptide
D. Pancreatic exocrine cells: chymotrypsinogen o Pancreatic exocrine cells: chymotrypsinogen
E. F cells: gastrin • Gastrin is a hormone that is released by G cells in the antrum
of the stomach both in response to a specific neurotransmitter
released from enteric nerve endings.
Ganong. Review of Medical Physiology 23rd ed. Chap21, pg316. Chap26, pg432
ANSWER: D
• Insulin increases protein synthesis
2. Which of the following are incorrectly paired? • Progesterone does not have anabolic effects.
A. Epinephrine: increased glycogenolysis in skeletal muscle • Hypoglycemia triggers increased secretion of at least four
B. Insulin: increased protein synthesis counter-regulatory hormones:
C. Glucagon: increased gluconeogenesis o Epinephrine: glycogenolysis in skeletal muscle
D. Progesterone: increased plasma glucose level o Glucagon: gluconeogenesis
E. Growth hormone: increased plasma glucose level o Cortisol: plasma glucose level
o Growth hormone: plasma glucose level
• **The end result is increased blood glucose level
Ganong. Review of Medical Physiology 23rd ed. Chapter 21, page . 325
ANSWER: D
3. Which of the following would be least likely to be seen 14 days • B cells are responsible pof the production of insulin.
after a rat is injected with a drug that kills all of its pancreatic B o Destroyed b cells  low insulin less protein syntesis 
cells? elevated amino acids in the plasma.
A. A rise in the plasma H+ concentration • Since the body cannot utilized glucose, it would release
B. A rise in the plasma glucagon concentration counterregulatory hormones :
C. A fall in the plasma HCO3 – concentration glucagon,cortisol,epinephrine,growth hormone.
D. A fall in the plasma amino acid concentration • Increased H+ and decreased HCO3- is caused by Ketoacidosis
E. A rise in plasma osmolality secondary to accumulation of acetoacetic acid.
• Hyperglycemia increased plama osmolarity.
Ganong. Review of Medical Physiology 23rd ed. Chapter 21, page .323.
4. When the plasma glucose concentration falls to low levels, a ANSWER: C
number of different hormones help combat the hypoglycemia.
After intravenous administration of a large dose of insulin, the • “The keys to counter-regulation appear to be epinephrine and
return of a low blood sugar level to normal is delayed in glucagon: if the plasma concentration of either increases, the
A. adrenal medullary insufficiency. decline in the plasma glucose level is reversed; but if both fail
B. glucagon deficiency. to increase, there is little if any compensatory rise in the
C. combined adrenal medullary insufficiency and glucagon plasma glucose level. The actions of the other hormones are
deficiency. supplementary.”
D. thyrotoxicosis. Ganong. Review of Medical Physiology 23rd ed. Chapter 21 , page .326.
E. acromegaly
5. Insulin increases the entry of glucose into: ANSWER: E
A. all tissues.
B. renal tubular cells. • GLUT 4 is an insulin-stimulated glucose transporter that
C. the mucosa of the small intestine. promotes uptake of glucose to skeletal and cardiac muscle,
D. most neurons in the cerebral cortex. and adipose tissue.
Ganong. Review of Medical Physiology 23rd ed. Chapter 21, page .320.
E. skeletal muscle.
6. Glucagon increases glycogenolysis in liver cells but ACTH does ANSWER:
not because: • Membranes of liver cells contain receptors different from those
A. cortisol increases the plasma glucose level. in adrenocortical cells.
B. liver cells have an adenylyl cyclase different from that in • Glucagon acts on phospholipase C on hepatic
adrenocortical cells. cellsincrease cytoplasmic Ca2+  glycogenolysis.
C. ACTH cannot enter the nucleus of liver cells. • ACTH binds to high-affinity receptors on the plasma
D. the membranes of liver cells contain receptors different from membraneof adrenocortical cells. This activates adenylyl
those in adrenocortical cells. cyclase via Gs.
E. liver cells contain a protein that inhibits the action of ACTH. Ganong. Review of Medical Physiology 23rd ed. Chapter 21 , page . 321.
7. A meal rich in proteins containing the amino acids that stimulate ANSWER: C
insulin secretion but low in carbohydrates does not cause
hypoglycemia because
A. the meal causes a compensatory increase in T4 secretion. • The increase in glucagon secretion following a protein meal
B. cortisol in the circulation prevents glucose from entering is also valuable, since the amino acids stimulate insulin
muscle. secretion and the secreted glucagon prevents the
C. glucagon secretion is also stimulated by the meal. development of hypoglycemia while the insulin promotes
D. the amino acids in the meal are promptly converted to storage of the absorbed carbohydrates and lipids.
glucose
Ganong. Review of Medical Physiology 23rd ed. Chapter 21, page . 330
E. insulin does not bind to insulin receptors if the plasma
concentration of amino acids is elevated.

TOPNOTCH MEDICAL BOARD PREP MED PHYSIOLOGY GANONG SUPPLEMENT HANDOUT BY THE TOPNOTCH TEAM Page 31 of 46
For inquiries visit www.topnotchboardprep.com.ph or https://web.facebook.com/topnotchmedicalboardprep/
TOPNOTCH MEDICAL BOARD PREP PHYSIOLOGY GANONG SUPPLEMENT HANDOUT BY THE TOPNOTCH TEAM
For inquiries visit www.topnotchboardprep.com.ph or https://www.facebook.com/topnotchmedicalboardprep/

SECTION 4: GASTROINTESTINAL PHYSIOLOGY


(CHAPTER 25) OVERVIEW OF GASTROINTESTINAL FUNCTION & REGULATION
ANSWER: C

1. Water is absorbed in the jejunum, ileum, and colon and excreted


in the feces. Arrange these in order of the amount of water
absorbed or excreted from greatest to smallest.
A. Colon, jejunum, ileum, feces
B. Feces, colon, ileum, jejunum
C. Jejunum, ileum, colon, feces
D. Colon, ileum, jejunum, feces
E. Feces, jejunum, ileum, colon

Ganong. Review of Medical Physiology 23rd ed. Chapter 26 , page .4411


2. Following a natural disaster in Haiti, there is an outbreak of ANSWER:
cholera among displaced persons living in a tent encampment.
The affected individuals display severe diarrheal symptoms • Cholera toxin increase cAMP opens chloride channels
because of which of the following changes in intestinal transport?’ chloride ions secretion activate a sodium pump  secretion
A. Increased Na+–K+ cotransport in the small intestine of sodium ions NaCl formed in the crypts extreme
B. Increased K+ secretion into the colon osmosis of water from the blood into the intestinal lumen.
C. Reduced K+ absorption in the crypts of Lieberkü hn
D. Increased Na+ absorption in the small intestine Guyton.Textbook of Medical Physiology 11th ed.Chapter 65, page 815
E. Increased Cl− secretion into the intestinal lumen
3. A 50-year-old man comes to see his clinician complaining of ANSWER: D
severe epigastric pain, frequent heartburn, and unexplained
weight loss of 20 pounds over a 6-month period. He claims to
have obtained no relief from over-the-counter H2 antihistamine
drugs. He is referred to a gastroenterologist, and upper
endoscopy reveals erosions and ulcerations in the proximal • Zollinger–Ellison syndrome is a syndrome is seen in patients
duodenum and an increased output of gastric acid in the fasting with gastrinomas. Gastrin causes prolonged hypersecretion
state. The patient is most likely to have a tumor secreting which of acid, and severe ulcers are produced.
of the following hormones?
A. Secretin Ganong. Review of Medical Physiology 23rd ed. Chapter 26, page 433 .
B. Somatostatin
C. Motilin
D. Gastrin
E. Cholecystokinin
ANSWER: C
4. Which of the following has the highest pH?
• Gastric juice pH 1-3 MOST ACIDIC
A. Gastric juice
B. Colonic luminal contents • Colonic luminal contents pH 5-7
C. Pancreatic juice • Pancreatic juice pH 8 MOST BASIC
D. Saliva • Saliva pH 6.5-7
E. Contents of the intestinal crypts • Contents of the intestinal crypts pH 5-7
Ganong. Review of Medical Physiology 23rd ed. Chapter 26, page 438 .
ANSWER: D
5. A 60-year-old woman undergoes total pancreatectomy because
of the presence of a tumor. Which of the following outcomes • Pancreatectomy loss of pancreatic secretions
would not be expected after she recovers from the operation? steatorrhea and protein malabsorption.
A. Steatorrhea • Pancreatectomy loss of insulin secretion high
B. Hyperglycemia glucagon/insulin ratio hyperglycemia
C. Metabolic acidosis • Pancreatectomy loss of insulin secretion decreased
D. Weight gain intracellular uptake of glucose and increase protein and fat
E. Decreased absorption of amino acids catabolism weight loss.
Ganong. Review of Medical Physiology 23rd ed. Chapter 21, page 324.
6. A patient with a tumor secreting abnormal amounts of gastrin ANSWER: C
(gastrinoma) would be most likely to exhibit which of the ff?
A. decreased chief cell exocytosis • Zollinger–Ellison syndrome is a syndrome is seen in patients
B. duodenal ulceration with gastrinomas. Gastrin causes prolonged hypersecretion
C. increased gastric pH in the period between meals of acid, and severe ulcers are produced.
D. a reduced incidence of gastroesophageal reflux disease
Ganong. Review of Medical Physiology 23rd ed. Chapter 26, page 433 ..
E. protein malabsoprtion

TOPNOTCH MEDICAL BOARD PREP MED PHYSIOLOGY GANONG SUPPLEMENT HANDOUT BY THE TOPNOTCH TEAM Page 32 of 46
For inquiries visit www.topnotchboardprep.com.ph or https://web.facebook.com/topnotchmedicalboardprep/
TOPNOTCH MEDICAL BOARD PREP PHYSIOLOGY GANONG SUPPLEMENT HANDOUT BY THE TOPNOTCH TEAM
For inquiries visit www.topnotchboardprep.com.ph or https://www.facebook.com/topnotchmedicalboardprep/
(CHAPTER 26) DIGESTION, ABSORPTION, & NUTRITIONAL PRINCIPLES
1. Maximum absorption of short-chain fatty acids produced by ANSWER:
bacteria occurs in the • Short-chain fatty acids are absorbed by specific transporters
A. stomach. present in colonic epithelial cells. SCFAs make a significant
B. duodenum. contribution to the total caloric intake and exert a trophic
C. jejunum. effect on the colonic epithelial cells, combat inflammation,
D. ileum. and maintain acid–base equilibrium.
E. colon. Ganong. Review of Medical Physiology 23rd ed. Chapter 27 , page 458 .
2. A premenopausal woman who is physically active seeks advice ANSWER: D
from her primary care clinician regarding measures she can take
to ensure adequate availability of dietary calcium to ensure bone
health later in life. Which of the following dietary components • Role of vitamin D derivative: Ca2+ absorption is adjusted to
should enhance calcium uptake? body needs; absorption is increased in the presence of Ca2+
A. Protein deficiency and decreased in the presence of Ca2+ excess.
B. Oxalates
C. Iron Ganong. Review of Medical Physiology 23rd ed. Chapter 27 , page 458 .
D. Vitamin D
E. Sodium
3. A decrease in which of the following would be expected in a child ANSWER: E
exhibiting a congenital absence of enterokinase? • The protein assimilation machinery, which rests heavily on
A. Incidence of pancreatitis the proteases in pancreatic juice, is arranged such that these
B. Glucose absorption enzymes are not activated until they reach their substrates in
C. Bile acid reabsorption the small intestinal lumen. This is accomplished by the
D. Gastric pH restricted localization of an activating enzyme, enterokinase.
E. Protein assimilation Ganong. Review of Medical Physiology 23rd ed. Chapter 27 , page 466.
ANSWER: A
• Hartnup disease is a congenital defect in the transport of
neutral amino acids does not lead to nutritional deficiencies of
these amino acids because peptide transport (PepT1)
compensates.
4. In Hartnup disease (a defect in the transport of neutral amino
acids), patients do not become deficient in these amino acids due
to the activity of:
A. PepT1.
B. brush border peptidases.
C. Na+, K+ ATPase.
D. cystic fibrosis transmembrane conductance regulator (CFTR).
E. trypsin.

Ganong. Review of Medical Physiology 23rd ed. Chapter 27 , page 456..


5. A newborn baby is brought to the pediatrician suffering from ANSWER: C
severe diarrhea that worsens with meals. The symptoms diminish
when nutrients are delivered intravenously. The child most likely • “When the sodium-dependent glucose transporter (SGLT-1)
has a mutation in which of the following intestinal transporters? is congenitally defective, the resulting glucose/galactose
A. Na+, K+ ATPase malabsorption causes severe diarrhea that is often fatal if
B. NHE3 glucose and galactose are not promptly removed from the
C. SGLT-1 diet.”
D. H+, K+ ATPase Ganong. Review of Medical Physiology 23rd ed. Chapter 27 , page 453 .
E. NKCC1
6. Calcium absorption is increased by: ANSWER: D
A. hypercalcemia • Role of 1,25- dihydroxycholecalciferol : Ca2+ absorption is
B. oxalates in the diet adjusted to body needs; absorption is increased in the
C. iron overload presence of Ca2+ deficiency and decreased in the presence of
D. 1,25- dihydroxycholecalciferol Ca2+ excess.
E. increased Na+ absorption Ganong. Review of Medical Physiology 23rd ed. Chapter 27 , page 458 .

(CHAPTER 27) GASTROINTESTINAL MOTILITY


ANSWER: C
1. In infants, defecation often follows a meal. The cause of colonic • Distention of the stomach by food initiates contractions of
contractions in this situation is the rectum and, frequently, a desire to defecate. The response
A. histamine. is called the gastrocolic reflex and may be amplified by an
B. increased circulating levels of CCK. action of gastrin on the colon. Because of the response,
C. the gastrocolic reflex. defecation after meals is the rule in children.
D. increased circulating levels of somatostatin. • In adults, habit and cultural factors play a large role in
E. the enterogastric reflex. determining when defecation occurs.
Ganong. Review of Medical Physiology 23rd ed. Chapter , page ..
2. The symptoms of the dumping syndrome (discomfort after meals ANSWER: D
in patients with intestinal short circuits such as anastomosis of
the jejunum to the stomach) are caused in part by • “Dumping syndrome,” is a distressing syndrome that develops
A. increased blood pressure. in patients in whom portions of the stomach have been
B. increased secretion of glucagon. removed or the jejunum has been anastomosed. Symptoms of
C. increased secretion of CCK. weakness, dizziness, and sweating after meals is due to the
D. hypoglycemia. hypoglycemia about 2 h after meals.
Ganong. Review of Medical Physiology 23rd ed. Chapter 28 , page 474 .
E. hyperglycemia.
TOPNOTCH MEDICAL BOARD PREP MED PHYSIOLOGY GANONG SUPPLEMENT HANDOUT BY THE TOPNOTCH TEAM Page 33 of 46
For inquiries visit www.topnotchboardprep.com.ph or https://web.facebook.com/topnotchmedicalboardprep/
TOPNOTCH MEDICAL BOARD PREP PHYSIOLOGY GANONG SUPPLEMENT HANDOUT BY THE TOPNOTCH TEAM
For inquiries visit www.topnotchboardprep.com.ph or https://www.facebook.com/topnotchmedicalboardprep/

3. Gastric pressures seldom rise above the levels that breach the ANSWER:
lower esophageal sphincter, even when the stomach is filled
with a meal, due to which of the following processes? • The stomach accommodates the meal by a process of receptive
A. Peristalsis relaxation. When food enters the stomach, the fundus and
B. Gastroileal reflex upper portion of the body relax, this permits an increase in
C. Segmentation volume without a significant increase in pressure.
D. Stimulation of the vomiting center
Ganong. Review of Medical Physiology 23rd ed. Chapter 28 , page 473
E. Receptive relaxation
ANSWER: A
• Motilin released cyclically and stimulates interdigestive
myoelectric complexes from stomach and small intestine every
4. The migrating motor complex is triggered by which of the
90 minutes in a fasted person.
following?
A. Motilin • NO intestinal smooth muscle relxation.
B. NO • CCK strongly contracts the gallbladder, expelling bile into the
C. CCK small intestine where the bile in turn plays important roles in
D. Somatostatin emulsifying fatty substances.
E. Secretin • Somatostatin inhibits intestinal secretions.
• Secretin promote pancreatic secretion of bicarbonate which in
turn helps to neutralize the acid in the small intestine.
Guyton.Textbook of Medical Physiology 11th ed.Chap62 , page 776
5. A patient is referred to a gastroenterologist because of ANSWER: B
persistent difficulties with swallowing. Endoscopic examination
reveals that the lower esophageal sphincter fails to fully open
as the bolus reaches it, and a diagnosis of achalasia is made. • In achalasia, the myenteric plexus of the esophagus is deficient
During the examination, or in biopsies taken from the sphincter at the LES in this condition and the release of NO and VIP is
region, a decrease would be expected in which of the following? defective, therefore there will be no anterograde relaxation
A. Esophageal peristalsis or relaxation ahead of the stimulus.
B. Expression of neuronal NO synthase
C. Acetylcholine receptors Ganong. Review of Medical Physiology 23rd ed. Chapter 28, page 470, 473.
D. Substance P release
E. Contraction of the crural diaphragm

(CHAPTER 28) TRANSPORT & METABOLIC FUNCTIONS OF THE LIVER


1. A patient suffering from severe ulcerative colitis undergoes a ANSWER: E
total colectomy with formation of a stoma. After a full recovery
from surgery, and compared to his condition prior to surgery, • Colectomy antibiotic bowel preparation + resection of the
which of the following would be expected to be decreased? entire colon eradicate intestinal bacteria and colonic
A. Ability to absorb lipids bacteria deconjugation of bilirubin absorption of
B. Ability to clot the blood uribilinogen urobilinogen excretion in the kidneys.
C. Circulating levels of conjugated bile acids • Lipids, fat soluble vitamin(vitamin K) reabsorption is not
D. Urinary urea altered since the patient had intact intestinal mucosa
Ganong. Review of Medical Physiology 23rd ed. Chapter 29, page . 483..
E. Urinary urobilinogen
2. A surgeon is studying new methods of liver transplantation. She ANSWER: E
performs a complete hepatectomy in an experimental animal. • Estrogen levels are expected to rise since estrogen is converted to
Before the donor liver is grafted, a rise in the blood level of which glucuronide and sulfate conjugates. All these compounds, along with
of the following would be expected? other metabolites, are excreted in the urine.
A. Glucose • The other choices are expected to fall because:
o Glucose: no liver glycogen source
B. Fibrinogen
o Fibrinogen: no synthesis
C. 25-Hydroxycholecalciferol o 25-Hydroxycholecalciferol: absence of liver enzyme 24hydroxylase.
D. Conjugated bilirubin o Conjugated bilirubin: absence of liver UDP-glucoronidase
E. Estrogens Ganong. Review of Medical Physiology 23rd ed. Chapter 25 , page 412..
ANSWER: C
3. Which of the following cell types protects against sepsis
• Hepatic stellate cell: formation of fibrosis
secondary to translocation of intestinal bacteria?
A. Hepatic stellate cell • Cholangiocyte: absorptive and secretory function.
B. Cholangiocyte • Kupffer cell: prevents bacterial translocation by its phagocytic
C. Kupffer cell activity
D. Hepatocyte • Hepatocyte: synthesis of various compounds, & detoxification.
E. Gallbladder epithelial cell • Gallbladder epithelial cell: concentrates bile
Ganong. Review of Medical Physiology 23rd ed. Chapter 29 , page . 486..
4. P450s (CYPs) are highly expressed in hepatocytes. In which of ANSWER: E
the following do they not play an important role?
A. Bile acid formation • Glycogen synthesis is the function of glycogen synthase and
B. Carcinogenesis not the CYP450.
C. Steroid hormone formation
D. Detoxification of drugs Ganong. Review of Medical Physiology 23rd ed. Chapter 1 , page 21..
E. Glycogen synthesis
5. A 40-year-old woman comes to her primary care clinician ANSWER: E
complaining of severe, episodic abdominal pain that is particularly
intense after she ingests a fatty meal. An imaging procedure reveals
that her gallbladder is acutely dilated, and a diagnosis of • The most common cause of pancreatitis is drinking excess
cholelithiasis is made. A gallstone lodged in which location will also alcohol, and the second most common cause is blockage of the
increase her risk of pancreatitis? papilla of Vater by a gallstone; the two together account for
A. Left hepatic duct more than 90 per cent of all cases.
B. Right hepatic duct
C. Cystic duct Guyton.Textbook of Medical Physiology 11th ed.Chapter 66, page 821.
D. Common bile duct
E. Sphincter of Oddi
TOPNOTCH MEDICAL BOARD PREP MED PHYSIOLOGY GANONG SUPPLEMENT HANDOUT BY THE TOPNOTCH TEAM Page 34 of 46
For inquiries visit www.topnotchboardprep.com.ph or https://web.facebook.com/topnotchmedicalboardprep/
TOPNOTCH MEDICAL BOARD PREP PHYSIOLOGY GANONG SUPPLEMENT HANDOUT BY THE TOPNOTCH TEAM
For inquiries visit www.topnotchboardprep.com.ph or https://www.facebook.com/topnotchmedicalboardprep/

ANSWER: B
• The concentration of sodium ions rises with a concomitant loss
6. Compared to hepatic bile, gallbladder bile contains a reduced of chloride and bicarbonate as the bile is concentrated in the
concentration of which of the following? gallbladder.
A. Bile acids • Other substances which are highly concentrated in the
B. Chloride ions gallbladder bile:
C. Protons o Calcium ions
D. Glucose o Bile salts
E. Calcium ions o Cholesterol
o Lecithin
Ganong. Review of Medical Physiology 23rd ed., Chapter 29 page .486
Guyton.Textbook of Medical Physiology 11th ed.Chapter 64, page 587, 803.
ANSWER: E
• Colectomy antibiotic bowel preparation + resection of the
7. Removal of the entire colon would be expected to cause: entire colon eradicate intestinal bacteria and colonic
A. death bacteria decreased deconjugation of bilirubin decreased
B. megaloblastic anemia absorption of uribilinogen decreased urobilinogen excretion
C. severe malnutrition in the kidneys.
D. a decrease in the blood level of ammonia in patients with • Anemia due to decreased B12 and malnutrition because of
cirrhosis of the liver protein malabsorption is not a consequence because both are
E. decreased urinary urobilinogen reabsorb in the small intestines.
• Ammonia is handled by the liver, and levels will increase in
cirrhosis.
Ganong. Review of Medical Physiology 23rd ed. Chapter 29, page . 483

SECTION 5: CARDIOVASCULAR PHYSIOLOGY

(CHAPTER 29) ORIGIN OF THE HEARTBEAT & THE ELECTRICAL ACTIVITY OF THE HEART
ANSWER: C
1. Which part of the ECG (eg, Figure 29–5) corresponds to
• The P wave: atrial depolarization
ventricular repolarization?
A. The P wave • The QRS duration: ventricular depolarization
B. The QRS duration • The T wave: ventricular repolarization
C. The T wave • The U wave: inconstant finding, prominent in hypokalemia.
D. The U wave • The PR interval: time between atrial depolarization and
E. The PR interval conduction through AV node.
Ganong. Review of Medical Physiology 23rd ed. Chapter 30, page 492 ...
2. Which of the following normally has a slowly depolarizing ANSWER: A
“prepotential”?
A. Sinoatrial node • “Pacemaker or prepotentials are normally prominent only in the SA
B. Atrial muscle cells and AV nodes, wherein there is no sharp, rapid depolarizing spike
before the plateau ass there is in other parts of the conduction system
C. Bundle of His
and the atrial and ventricular fibers.”
D. Purkinje fibers Ganong. Review of Medical Physiology 23rd ed. Chapter 30 , page 491 .
E. Ventricular muscle cells
ANSWER: A
3. In second-degree heart block
A. the ventricular rate is lower than the atrial rate. • In second-degree heart block, not all atrial impulses are
B. the ventricular ECG complexes are distorted. conducted to the ventricles. For example, a ventricular beat
C. there is a high incidence of ventricular tachycardia. may follow every second or every third atrial beat (2:1
D. stroke volume is decreased. block, 3:1 block, etc). This would lead to ventricular rates
E. cardiac output is increased. which is lower than the atrial.
Ganong. Review of Medical Physiology 23rd ed. Chapter 30, page 497 ..
ANSWER: D
4. Currents caused by opening of which of the following channels
• Initial depolarization: Na + influx through rapidly opening
contribute to the repolarization phase of the action potential of
ventricular muscle fibers? Na +channels (the
A. Na+ channels • Rapid repolarization: inactivation of Na+ channels
B. Cl− channels • Plateau phase: Ca2+ influx through more slowly opening
C. Ca2+ channels Ca2+ channels .
D. K+ channels • Repolarization: net K+ efflux through multiple types of K+
E. HCO3− channels channels
Ganong. Review of Medical Physiology 23rd ed. Chapter 30 , page 491 ..
5. In complete heart block ANSWER: D
A. fainting may occur because the atria are unable to pump • Individuals with complete heart block, ventricles beat at a low
blood into the ventricles. rate independently of the atria, there may also be periods of
B. ventricular fibrillation is common. asystole lasting a minute or more. The resultant cerebral
C. the atrial rate is lower than the ventricular rate. ischemia causes dizziness and fainting (Stokes–Adams
D. fainting may occur because of prolonged periods during syndrome)
which the ventricles fail to contract. Ganong. Review of Medical Physiology 23rd ed. Chapter 30 , page 497 .

TOPNOTCH MEDICAL BOARD PREP MED PHYSIOLOGY GANONG SUPPLEMENT HANDOUT BY THE TOPNOTCH TEAM Page 35 of 46
For inquiries visit www.topnotchboardprep.com.ph or https://web.facebook.com/topnotchmedicalboardprep/
TOPNOTCH MEDICAL BOARD PREP PHYSIOLOGY GANONG SUPPLEMENT HANDOUT BY THE TOPNOTCH TEAM
For inquiries visit www.topnotchboardprep.com.ph or https://www.facebook.com/topnotchmedicalboardprep/
(CHAPTER 30) THE HEART AS A PUMP
1. The second heart sound is caused by ANSWER: A
A. closure of the aortic and pulmonary valves. • S2: closure of the aortic and pulmonary valves.
B. vibrations in the ventricular wall during systole. • S4: vibrations in the ventricular wall during systole.
C. ventricular filling. • S3: ventricular filling.
D. closure of the mitral and tricuspid valves. • S1: closure of the mitral and tricuspid valves.
E. retrograde flow in the vena cava. Ganong. Review of Medical Physiology 23rd ed. Chapter 31 , page 513...

2. The fourth heart sound is caused by ANSWER: A


A. closure of the aortic and pulmonary valves. • S3: ventricular filling.
B. vibrations in the ventricular wall during systole. • S2: closure of the aortic and pulmonary valves.
C. ventricular filling. • S4: vibrations in the ventricular wall during systole.
D. closure of the mitral and tricuspid valves. • S1: closure of the mitral and tricuspid valves.
E. retrograde flow in the vena cava. Ganong. Review of Medical Physiology 23rd ed. Chapter 31 , page 513.

3. The dicrotic notch on the aortic pressure curve is caused by ANSWER: C


A. closure of the mitral valve. • “The dicrotic notch, a small oscillation on the falling phase of
B. closure of the tricuspid valve. the pulse wave caused by vibrations set up when the aortic
C. closure of the aortic valve. valve snaps shut.”
D. closure of the pulmonary valve.
E. rapid filling of the left ventricle. Ganong. Review of Medical Physiology 23rd ed. Chapter 31, page 512..
ANSWER: C
4. During exercise, a man consumes 1.8 L of oxygen per minute. His
arterial O2 content is 190 mL/L, and the O2 content of his mixed
venous blood is 134 mL/L. His cardiac output is approximately
A. 3.2 L/min.
B. 16 L/min.
C. 32 L/min.
D. 54 L/min.
E. 160 mL/min.
Ganong. Review of Medical Physiology 23rd ed. Chapter 31 , page 513 .
5. The work performed by the left ventricle is substantially greater ANSWER: E
than that performed by the right ventricle, because in the left • In terms of Preload (volume): right heart > left heart
ventricle • In terms of Afterload (pressure): left heart > right heart
A. the contraction is slower. • Pressure work produces a greater increase in O2
B. the wall is thicker. consumption than Volume work.
C. the stroke volume is greater. • Afterload causes a greater increase in cardiac O2 consumption
D. the preload is greater. than does an increase in Preload.
E. the afterload is greater. Ganong. Review of Medical Physiology 23rd ed. Chapter 31 , page 519 .
6. Starling’s law of the heart ANSWER: D
A. does not operate in the failing heart.
B. does not operate during exercise. • Increase venous return cardiac muscle stretched to optimal
C. explains the increase in heart rate produced by exercise. length optimal overlap of actin and myosin cross-
D. explains the increase in cardiac output that occurs when bridges increase in force of contraction of cardiac
venous return is increased. muscle increase cardiac output.
E. explains the increase in cardiac output when the sympathetic
Guyton.Textbook of Medical Physiology 11th ed.Chapter 9 , page 112..
nerves supplying the heart are stimulated.

(CHAPTER 31) BLOOD AS A CIRCULATORY FLUID & THE DYNAMICS OF BLOOD & LYMPH FLOW
ANSWER: C

1. Which of the following has the highest total cross-sectional area


in the body?
A. Arteries
B. Arterioles
C. Capillaries
D. Venules
E. Veins

Guyton.Textbook of Medical Physiology 11th ed.Chapter 14, page 162


ANSWER: B
2. Lymph flow from the foot is • Flow in the collecting lymphatics INCREASED by :
A. increased when an individual rises from the supine to the o movements of skeletal muscle (by massaging/contraction)
standing position. as with foot massage or exercise.
B. increased by massaging the foot. o negative intrathoracic pressure during inspiration
C. increased when capillary permeability is decreased. o suction effect of high velocity flow of blood in the veins in
D. decreased when the valves of the leg veins are incompetent. which the lymphatics terminate.
E. decreased by exercise. o Increase capillary permeability
Ganong. Review of Medical Physiology 23rd ed. Chapter 31 , page 513..

TOPNOTCH MEDICAL BOARD PREP MED PHYSIOLOGY GANONG SUPPLEMENT HANDOUT BY THE TOPNOTCH TEAM Page 36 of 46
For inquiries visit www.topnotchboardprep.com.ph or https://web.facebook.com/topnotchmedicalboardprep/
TOPNOTCH MEDICAL BOARD PREP PHYSIOLOGY GANONG SUPPLEMENT HANDOUT BY THE TOPNOTCH TEAM
For inquiries visit www.topnotchboardprep.com.ph or https://www.facebook.com/topnotchmedicalboardprep/
ANSWER: D
• Fluid movement = k[(Pc – Pi) – (πc – πi)]
3. The pressure in a capillary in skeletal muscle is 35 mm Hg at the • where
arteriolar end and 14 mm Hg at the venular end. The interstitial o k = capillary filtration coefficient
pressure is 0 mm Hg. The colloid osmotic pressure is 25 mm Hg o Pc = capillary hydrostatic pressure 35-14= 11mmHg
in the capillary and 1 mm Hg in the interstitium. The net force o Pi = interstitial hydrostatic pressure 0
producing fluid movement across the capillary wall at its o πc = capillary colloid osmotic pressure 25mmHg
arteriolar end is o πi = interstitial colloid osmotic pressure 1mmHg
A. 3 mm Hg out of the capillary.
B. 3 mm Hg into the capillary.
C. 10 mm Hg out of the capillary.
D. 11 mm Hg out of the capillary.
E. 11 mm Hg into the capillary

Ganong. Review of Medical Physiology 23rd ed. Chapter 32 , page . 458


ANSWER: B
• The average velocity of fluid movement at any point in a
system of tubes in parallel is inversely proportional to the
total cross-sectional area.
• Referring to the illustration below, veins had greater velocity
4. The velocity of blood flow than venules because it has smaller cross sectional area.
A. is higher in the capillaries than the arterioles.
B. is higher in the veins than in the venules.
C. is higher in the veins than the arteries.
D. falls to zero in the descending aorta during diastole.
E. is reduced in a constricted area of a blood vessel.

Ganong. Review of Medical Physiology 23rd ed. Chapter 32, page 541.
ANSWER: E
• flow varies DIRECTLY and resistance INVERSELY with the
fourth power of the radius .
5. When the radius of the resistance vessels is increased, which of
the following is increased? • in which:
A. Systolic blood pressure
B. Diastolic blood pressure
C. Viscosity of the blood • F is the rate of blood flow,
D. Hematocrit • DP is the pressure difference
E. Capillary blood flow • r is the radius of the vessel
• l is length of the vessel
• h is viscosity of the blood.
Ganong. Review of Medical Physiology 23rd ed. Chapter , page .
ANSWER: A
6. A 30-year-old patient comes to her primary care clinician • Increased hematocrit increased viscosity Increase
complaining of headaches and vertigo. A blood test reveals a resistance increase mean blood pressure
hematocrit of 55%, and a diagnosis of polycythemia is made.
Which of the following would also be increased? • in which:
A. Mean blood pressure
B. Radius of the resistance vessels • R is resistance,
C. Radius of the capacitance vessels • DP is the pressure difference
D. Central venous pressure • r is the radius of the vessel
• l is length of the vessel
E. Capillary blood flow
• h is viscosity of the blood.
Ganong. Review of Medical Physiology 23rd ed. Chapter 32 , page 542 .
7. A pharmacologist discovers a drug that stimulates the ANSWER: A
production of VEGF receptors. He is excited because the drug • VEGF agonist responsible for vasculogenesis or development
might be of value in the treatment of of new blood supply that will help bypass the blocked
A. coronary artery disease. arteries as seen in coronary artery disease.
B. cancer. • VEGF antagonists and other angiogenesis inhibitors have now
C. emphysema. entered clinical practice as adjunctive therapies for many
D. diabetes insipidus. malignancies
E. dysmenorrhea. Ganong. Review of Medical Physiology 23rd ed. Chapter 32, page . 539
8. Why is the dilator response to injected acetylcholine changed to ANSWER: E
a constrictor response when the endothelium is damaged?
A. More Na+ is generated. • Acetylcholine act directly on vascular smooth muscle would
B. More bradykinin is generated. produce much greater constriction if their effects were not
C. The damage lowers the pH of the remaining layers of the restricted by simultaneous release of NO from the intact blood
artery. vessels.
D. The damage augments the production of endothelin by the • Damage endothelium NO is not released absence of
endothelium. relaxation of vascular smooth muscle predominance of
E. The damage interferes with the production of NO by the vasoconstriction effect of acetylcholine.
Ganong. Review of Medical Physiology 23rd ed. Chapter 33 , page 564 .
Endothelium

TOPNOTCH MEDICAL BOARD PREP MED PHYSIOLOGY GANONG SUPPLEMENT HANDOUT BY THE TOPNOTCH TEAM Page 37 of 46
For inquiries visit www.topnotchboardprep.com.ph or https://web.facebook.com/topnotchmedicalboardprep/
TOPNOTCH MEDICAL BOARD PREP PHYSIOLOGY GANONG SUPPLEMENT HANDOUT BY THE TOPNOTCH TEAM
For inquiries visit www.topnotchboardprep.com.ph or https://www.facebook.com/topnotchmedicalboardprep/
(CHAPTER 32) CARDIOVASCULAR REGULATORY MECHANISMS
1. When a pheochromocytoma (tumor of the adrenal medulla) ANSWER: B
suddenly discharges a large amount of epinephrine into the
circulation, the patient's heart rate would be expected to
A. increase because the increase in blood pressure stimulates
the carotid and aortic baroreceptors. • Pheochromocytoma large amount of epinephrine
B. increase because epinephrine has a direct chronotropic released direct chronotropic effect increase in heart
effect on the heart. rate.
C. increase because of increased tonic parasympathetic • Low blood pressure stimulates the baroreceptor to increase
discharge to the heart. the heart rate.
D. decrease because the increase in blood pressure stimulates
Ganong. Review of Medical Physiology 23rd ed. Chapter 31, page 514 ..
the carotid and aortic chemoreceptors.
E. decrease because of increased tonic parasympathetic
discharge to the heart.
2. Orthostatic hypotension due to a malfunction in the ANSWER: A
baroreceptor reflex was diagnosed in a 65-year-old man who
had been experiencing frequent episodes of syncope as he got
out of bed in the mornings. Activation of the baroreceptor reflex • Baroreceptors are very important in short-term control of
A. is primarily involved in short-term regulation of systemic arterial pressure. Activation of the reflex allows for rapid
blood pressure. adjustments in blood pressure in response to abrupt
B. leads to an increase in heart rate because of inhibition of the changes in blood volume, cardiac output, or peripheral
vagal cardiac motor neurons. resistance.
C. inhibits neurons in the CVLM.
D. excites neurons in the RVLM. Ganong. Review of Medical Physiology 23rd ed. Chapter 33, page 561 .
E. occurs only under situations in which blood pressure is
markedly elevated.
3. A 45-year-old woman had a blood pressure of 155/95 mm Hg ANSWER: D
when she was at her clinician's office for a physical. It was her • Glutamate: stimulatory
first time to see this clinician and her first physical in over 10 • GABA: inhibitory
years. The clinician suggested that she begin monitoring her • Glutamate receptors were activated in the NTS glutamate
blood pressure at home. Sympathetic nerve activity would be receptors were activated in the CVLM  GABA receptors
expected to increase were activated in the RVLM decrease sympathetic output.
A. if glutamate receptors were activated in the NTS. • Sympathetic signals are increased during sexual
B. if GABA receptors were activated in the RVLM. excitement, anger, and stress thru descending tracts to the
C. if glutamate receptors were activated in the CVLM. vasomotor area from the cerebral cortex (particularly the
D. during stress. limbic cortex).
E. when one transitions from an erect to a supine posture. Ganong. Review of Medical Physiology 23rd ed. Chapter 33 , page 557.
4. Which of the following neurotransmitters are correctly matched ANSWER: D
with an autonomic pathway?
A. GABA is released by NTS neurons projecting to the RVLM. • Glutamate: stimulatory
B. Glutamate is released by CVLM neurons projecting to the IML • GABA: inhibitory
C. GABA is released by NTS neurons projecting to the nucleus • Glutamate receptors were activated in the NTS glutamate
ambiguus. receptors were activated in the CVLM  GABA receptors
D. GABA is released by CVLM neurons projecting to the RVLM. were activated in the RVLM decrease sympathetic output.
Ganong. Review of Medical Physiology 23rd ed. Chapter 33 , page 557.
E. Glutamate is released by CVLM neurons projecting to the NTS
5. A 53-year-old woman with chronic lung disease was ANSWER: D
experiencing difficulty breathing. Her arterial PO2 and PCO2
were 50 mm Hg and 60 mm Hg, respectively. Which one of the
following statements about chemoreceptors is correct?
A. Peripheral chemoreceptors are very sensitive to small
increases in arterial PCO2. • Increase intracranial pressuredecreased medullary blood
B. Activation of arterial chemoreceptors leads to a fall in flow decreased oxygen and excess buildup of carbon
arterial pressure. dioxide and hydrogen ions  central chemoreceptors
C. Peripheral chemoreceptors are located in the NTS. become stimulated.
D. Central chemoreceptors can be activated by an increase in
Guyton.Textbook of Medical Physiology 11th ed.Chapter 18 , page 212 .
intracranial pressure that compromises blood flow in the
medulla.
E. Central chemoreceptors are activated by increases in tissue
pH
6. Why is the dilator response to injected acetylcholine change to a ANSWER: E
constrictor response when the endothelium is damaged?
A. More Na+ is generated • Acetylcholine act directly on vascular smooth muscle would
B. More bradykinin is generated produce much greater constriction if their effects were not
C. The damage lowers the pH of the remaining layers of the restricted by simultaneous release of NO from the intact blood
artery vessels.
D. The damage augments the production of endothelin by the • Damage endothelium NO is not released absence of
endothelium relaxation of vascular smooth muscle predominance of
E. The damage interferes with the production of NO by the vasoconstriction effect of acetylcholine.
Ganong. Review of Medical Physiology 23rd ed. Chapter 33 , page 564 .
endothelium

TOPNOTCH MEDICAL BOARD PREP MED PHYSIOLOGY GANONG SUPPLEMENT HANDOUT BY THE TOPNOTCH TEAM Page 38 of 46
For inquiries visit www.topnotchboardprep.com.ph or https://web.facebook.com/topnotchmedicalboardprep/
TOPNOTCH MEDICAL BOARD PREP PHYSIOLOGY GANONG SUPPLEMENT HANDOUT BY THE TOPNOTCH TEAM
For inquiries visit www.topnotchboardprep.com.ph or https://www.facebook.com/topnotchmedicalboardprep/
(CHAPTER 33) CIRCULATION THROUGH SPECIAL REGIONS
1. Blood in which of the following vessels normally has the lowest ANSWER:
Po2?
A. Maternal artery • Maternal artery: 98% oxygen saturation
B. Maternal uterine vein • Umbilical vein: 80% oxygen saturation
C. Maternal femoral vein • Umbilical artery: 60% oxygen saturation
D. Umbilical artery
Ganong. Review of Medical Physiology 23rd ed. Chapter 34, page .582..
E. Umbilical vein
ANSWER: A

2. The pressure differential between the heart and the aorta is


least in the
A. left ventricle during systole.
B. left ventricle during diastole.
C. right ventricle during systole.
D. right ventricle during diastole.
E. left atrium during systole.

Ganong. Review of Medical Physiology 23rd ed. Chapter 34 , page 579 ..


3. Injection of tissue plasminogen activator (t-PA) would probably ANSWER:
be most beneficial:
A. after at least 1 year of uncomplicated recovery following • Tissue Plasminogen Activator:
occlusion of a coronary artery. o Effective in activating plasminogen to plasmin.
B. after at least 2 months of rest and recuperation following o “Dissolve some intravascular clots (if used within the first
occlusion of a coronary artery. hour or so after thrombotic occlusion) and heart is often
C. during the second week after occlusion of a coronary artery. spared serious damage.”
D. during the second day after occlusion of a coronary artery. Guyton.Textbook of Medical Physiology 11th ed. Chapter 36, page
E. during the second hour after occlusion of a coronary artery. 466.
ANSWER: E
4. Which of the following organs has the greatest blood flow per • On a per gram weight basis, the kidneys normally consume
100 g of tissue? oxygen at twice the rate of the brain but have almost seven
A. Brain times the blood flow of the brain. Thus, the oxygen delivered to
B. Heart muscle the kidneys far exceeds their metabolic needs, and the arterial-
C. Skin venous extraction of oxygen is relatively low compared with
D. Liver that of most other tissues.
E. Kidneys Ganong. Review of Medical Physiology 23rd ed. Chapter 26, page 320.
Guyton.Textbook of Medical Physiology 11th ed.Chapter 26, page 320.
5. Which of the following does not dilate arterioles in the skin? ANSWER: E
A. Increased body temperature
B. Epinephrine • Vasopressin is even more powerful than angiotensin II as a
C. Bradykinin vasoconstrictor, thus making it one of the body’s most potent
D. Substance P vascular constrictor substances.
Guyton.Textbook of Medical Physiology 11th ed.Chapter 17 , page 202.
E. Vasopressin
6. A baby boy is brought to the hospital because of convulsions. In ANSWER: D
the course of a workup, his body temperature and plasma
glucose are found to be normal, but his cerebrospinal fluid
glucose is 12 mg/dL (normal, 65 mg/dL). A possible explanation • Infants with congenital GLUT1 55K deficiency develop low
of his condition is: CSF glucose concentrations in the presence of normal plasma
A. constitutive activation of GLUT3 in neurons. glucose, and they have seizures and delayed development.
B. SGLT-1 deficiency in astrocytes.
C. GLUT5 deficiency in cerebral capillaries. Ganong. Review of Medical Physiology 23rd ed. Chapter 34, page 573..
D. GLUT1 55K deficiency in cerebral capillaries.
E. GLUT1 45K deficiency in microglia.

TOPNOTCH MEDICAL BOARD PREP MED PHYSIOLOGY GANONG SUPPLEMENT HANDOUT BY THE TOPNOTCH TEAM Page 39 of 46
For inquiries visit www.topnotchboardprep.com.ph or https://web.facebook.com/topnotchmedicalboardprep/
TOPNOTCH MEDICAL BOARD PREP PHYSIOLOGY GANONG SUPPLEMENT HANDOUT BY THE TOPNOTCH TEAM
For inquiries visit www.topnotchboardprep.com.ph or https://www.facebook.com/topnotchmedicalboardprep/

SECTION 6: RESPIRATORY PHYSIOLOGY

(CHAPTER 34) INTRODUCTION TO PULMONARY STRUCTURE & MECHANICS


1. On the summit of Mt. Everest, where the barometric pressure is ANSWER: D
about 250 mm Hg, the partial pressure of O2 in mm Hg is about:
A. 0.1 • The composition of dry air is 20.98% O2, 0.04% CO 2, 78.06%
B. 0.5 N2, and 0.92% other inert constituents such as argon and helium.
C. 5 • The partial pressure of O2 in dry air is therefore 0.20 × 250, or
D. 50 50 mmHg at the level of Mt. Everest.
Ganong. Review of Medical Physiology 23rd ed. Chapter 35 , page 588..
E. 100
2. The forced vital capacity is ANSWER: C
A. the amount of air that normally moves into (or out of) the
lung with each respiration. • The forced vital capacity (FVC), the largest amount of air that
B. the amount of air that enters the lung but does not can be expired after a maximal inspiratory effort, is frequently
participate in gas exchange. measured clinically as an index of pulmonary function.
C. the amount of air expired after maximal expiratory effort.
D. the largest amount of gas that can be moved into and out of Ganong. Review of Medical Physiology 23rd ed. Chapter 35 , page 593 .
the lungs in 1 min.
ANSWER: A
• Tidal Volume: the amount of air that normally moves into (or
out of) the lung with each respiration.
• Dead space: the amount of air that enters the lung but does not
participate in gas exchange.
• Expiratory Reserve Volume: the amount of air expired after
3. The tidal volume is
maximal expiratory effort.
A. the amount of air that normally moves into (or out of) the
• FEV1: fraction of the vital capacity expired during the first second
lung with each respiration.
of a forced expiration.
B. the amount of air that enters the lung but does not
participate in gas exchange.
C. the amount of air expired after maximal expiratory effort.
D. the amount of gas that can be moved into and out of the lungs
in 1 min.

*Respiratory volume and capacities: see below

Ganong. Review of Medical Physiology 23rd ed. Chap35, page 593


4. Which of the following is responsible for the movement of O2 ANSWER: E
from the alveoli into the blood in the pulmonary capillaries?
A. Active transport • Gases moves thru passive diffusion from the alveoli to the blood
B. Filtration in the pulmonary capillaries or vice versa across the thin
C. Secondary active transport alveolocapillary membrane.
D. Facilitated diffusion Ganong. Review of Medical Physiology 23rd ed. Chapter 35 , page 601.
E. Passive diffusion
ANSWER: D
5. Airway resistance • Airway resistance:
A. is increased if the lungs are removed and inflated with saline. o is increased following bronchial smooth muscle contraction.
B. does not affect the work of breathing. o is decreased if the lungs are removed and inflated with
C. is increased in paraplegic patients. saline. (saline decreases surface tension)
D. is increased following bronchial smooth muscle contraction. o does affect the work of breathing; it makes up 28% of the
E. makes up 80% of the work of breathing. work of breathing.
Ganong. Review of Medical Physiology 23rd ed. Chapter 35, pages 597,598 ..
ANSWER:
6. Surfactant lining the alveoli • Surfactant:
A. helps prevent alveolar collapse. o Prevents alveolar collapse.
B. is produced in alveolar type I cells and secreted into the o produced in alveolar type II cells
alveolus. o mixture of dipalmitoylphosphatidylcholine, other lipids,
C. is increased in the lungs of heavy smokers. and proteins.
D. is a glycolipid complex. o Decreased in the lungs of chronic smokers.
Ganong. Review of Medical Physiology 23rd ed. Chapter 35, page . 597
ANSWER: A
7. Which of the following causes relaxation of bronchial smooth
muscle?
A. Vasointestinal Peptide
B. Leukotrienes
C. Acetylcholine
D. Cool Air
E. Sulfur Dioxide
Ganong. Review of Medical Physiology 23rd ed. Chapter 35, page . 594

TOPNOTCH MEDICAL BOARD PREP MED PHYSIOLOGY GANONG SUPPLEMENT HANDOUT BY THE TOPNOTCH TEAM Page 40 of 46
For inquiries visit www.topnotchboardprep.com.ph or https://web.facebook.com/topnotchmedicalboardprep/
TOPNOTCH MEDICAL BOARD PREP PHYSIOLOGY GANONG SUPPLEMENT HANDOUT BY THE TOPNOTCH TEAM
For inquiries visit www.topnotchboardprep.com.ph or https://www.facebook.com/topnotchmedicalboardprep/

(CHAPTER 35) GAS TRANSPORT & PH


ANSWER: E

1. Most of the CO2 transported in the blood is


A. dissolved in plasma.
B. in carbamino compounds formed from plasma proteins.
C. in carbamino compounds formed from hemoglobin.
D. bound to Cl−.
E. in HCO3−.

Guyton.Textbook of Medical Physiology 11th ed.Chapter 40 , page 510.


2. Which of the following has the greatest effect on the ability of ANSWER: B
blood to transport oxygen?
A. Capacity of the blood to dissolve oxygen • The primary function of hemoglobin in the body is to combine
B. Amount of hemoglobin in the blood with oxygen in the lungs and then to release this oxygen
C. pH of plasma readily in the peripheral tissue capillaries, where the gaseous
D. CO2 content of red blood cells tension of oxygen is much lower than in the lungs.
Ganong. Review of Medical Physiology 23rd ed. Chapter 32, page 425
E. Temperature of the blood
3. Which of the following is true of the system? ANSWER: D

• Reaction 1
o is catalyzed by carbonic anhydrase.
A. Reaction 2 is catalyzed by carbonic anhydrase. o helps decrease the pH of blood during hyperventilation.
B. Because of reaction 2, the pH of blood declines during o occurs primarily in plasma but can also occur in the red
hyperventilation. blood cell, at faster rate with the help of carbonic anhydrase
C. Reaction 1 occurs in the red blood cell. • The reactions move to the left when there is excess H+ in the
D. Reaction 1 occurs primarily in plasma. tissues.
E. The reactions move to the right when there is excess H+ in Guyton.Textbook of Medical Physiology 11th ed.Chapt 40, pgs 510-511.
the tissues.
4. In comparing uncompensated respiratory acidosis & uncompen- ANSWER: D
sated metabolic acidosis which one of the following is true?
A. Plasma pH change is always greater in uncompensated
respiratory acidosis compared to uncompensated metabolic
acidosis.
B. There are no compensation mechanisms for respiratory
acidosis, whereas there is respiratory compensation for
metabolic acidosis.
C. Uncompensated respiratory acidosis involves changes in
plasma [HCO3−], whereas plasma [HCO3−] is unchanged in
uncompensated metabolic acidosis.
D. Uncompensated respiratory acidosis is associated with a
change in Pco2, whereas in uncompensated metabolic
acidosis Pco2 is constant. Ganong. Review of Medical Physiology 23rd ed. Chapter 36, pages 615-616 ..

ANSWER:
5. O2 delivery to the tissues would be reduced to the greatest
extent in • oxygen concentration: a normal subject breathing
A. a normal subject breathing 100% O2 on top of Mt. Everest atmospheric O2 on top of Mt. Everest
B. a normal subject running a marathon at sea level • oxygen transport: a patient with carbon monoxide poisoning
C. a patient with carbon monoxide poisoning • oxygen utilization: a patient who has ingested cyanide
D. a patient who has ingested cyanide • oxygen delivery: a patient with moderately severe metabolic
E. a patient with moderately severe metabolic acidosis acidosis. (high H+ shifts the O2-Hgb curve to the RIGHT)
Ganong. Review of Medical Physiology 23rd ed. Chapter , page .

TOPNOTCH MEDICAL BOARD PREP MED PHYSIOLOGY GANONG SUPPLEMENT HANDOUT BY THE TOPNOTCH TEAM Page 41 of 46
For inquiries visit www.topnotchboardprep.com.ph or https://web.facebook.com/topnotchmedicalboardprep/
TOPNOTCH MEDICAL BOARD PREP PHYSIOLOGY GANONG SUPPLEMENT HANDOUT BY THE TOPNOTCH TEAM
For inquiries visit www.topnotchboardprep.com.ph or https://www.facebook.com/topnotchmedicalboardprep/
(CHAPTER 36) REGULATION OF RESPIRATION
ANSWER: D
1. The main respiratory control neurons • Main respiratory control neurons:
A. send out regular bursts of impulses to expiratory muscles o is not active during normal, quiet breathing, when
during quiet respiration. expiration is passive
B. are unaffected by stimulation of pain receptors. o are located in the medulla.
C. are located in the pons. o send out regular bursts of impulses to inspiratory muscles
D. send out regular bursts of impulses to inspiratory muscles during quiet respiration.
during quiet respiration. o are affected by impulses from the pain receptors and
E. are unaffected by impulses from the cerebral cortex. cerebral cortex.
Guyton.Textbook of Medical Physiology 11th ed.Chapter 41 , pages 514-515
ANSWER: B
2. Intravenous lactic acid increases ventilation. The receptors • Carotid bodies:
responsible for this effect are located in the o Peripheral chemoreceptors located at the bifurcation of the
A. medulla oblongata. common carotid arteries.
B. carotid bodies. o Stimulated in cases of:
C. lung parenchyma. ▪ Decreases in arterial PO2 (<60 mm Hg)
D. aortic baroreceptors. ▪ Increases in arterial PCO2
E. trachea and large bronchi. ▪ Increases in arterial [H+]
Ganong. Review of Medical Physiology 23rd ed. Chapter 37 , page 628.
ANSWER: B

3. Spontaneous respiration ceases after


A. transection of the brainstem above the pons.
B. transection of the brainstem at the caudal end of the medulla.
C. bilateral vagotomy.
D. bilateral vagotomy combined with transection of the
brainstem at the superior border of the pons.
E. transection of the spinal cord at the level of the first thoracic
segment.
Respiratory neurons in the brain stem. Dorsal view of brain stem;
cerebellum intact. The effects of various lesions and brain stem transactions
are shown; the spirometer tracings at the right indicate the depth and rate of
breathing. If a lesion is introduced at D, breathing ceases. The effects of
higher transections, with and without vagus nerves transections, are shown.
Ganong. Review of Medical Physiology 23rd ed. Chapter 37 , page 626.
4. The following physiologic events that occur in vivo are listed in ANSWER: D
random order:
(1) decreased CSF pH;
(2) increased arterial Pco2;
(3) increased CSF Pco2;
(4) stimulation of medullary chemoreceptors; • Increased alveolar Pco2 Increased arterial Pco2 Increased
(5) increased alveolar Pco2. CSF Pco2 Decreased CSF pHStimulation of medullary
What is the usual sequence in which they occur when they affect chemoreceptors.
respiration?
A. 1, 2, 3, 4, 5 Ganong. Review of Medical Physiology 23rd ed. Chapter 37 , pages 629-630 .
B. 4, 1, 3, 2, 5
C. 3, 4, 5, 1, 2
D. 5, 2, 3, 1, 4
E. 5, 3, 2, 4, 1
5. The following events that occur in the carotid bodies when they ANSWER: E
are exposed to hypoxia are listed in random order:
(1) depolarization of type I glomus cells;
(2) excitation of afferent nerve endings;
(3) reduced conductance of hypoxia-sensitive K+ channels in
type I glomus cells; • Reduced conductance of hypoxia-sensitive K+ channels in type I
(4) Ca2+ entry into type I glomus cells; glomus cells Decreased K+ efflux Depolarization of type I
(5) decreased K+ efflux. glomus cells Ca2+ entry into type I glomus cells Excitation of
What is the usual sequence in which they occur on exposure to afferent nerve endings.
hypoxia?
A. 1, 3, 4, 5, 2 Ganong. Review of Medical Physiology 23rd ed. Chapter 37 , pages 628-629 .
B. 1, 4, 2, 5, 3
C. 3, 4, 5, 1, 2
D. 3, 1, 4, 5, 2
E. 3, 5, 1, 4, 2
6. Injection of a drug that stimulates the carotid bodies would be ANSWER: B
expected to cause
A. a decrease in the pH of arterial blood. • Stimulation of carotid bodies Increased ventilation Decreased
B. a decrease in the Pco2 of arterial blood. alveolar Pco2 Decreased arterial Pco2 Increased in pH of
C. an increase in the HCO3− concentration of arterial blood. arterial blood.
D. an increase in urinary Na+ excretion.
Ganong. Review of Medical Physiology 23rd ed. Chapter 37 , pages 628-629 ..
E. an increase in plasma Cl–.

TOPNOTCH MEDICAL BOARD PREP MED PHYSIOLOGY GANONG SUPPLEMENT HANDOUT BY THE TOPNOTCH TEAM Page 42 of 46
For inquiries visit www.topnotchboardprep.com.ph or https://web.facebook.com/topnotchmedicalboardprep/
TOPNOTCH MEDICAL BOARD PREP PHYSIOLOGY GANONG SUPPLEMENT HANDOUT BY THE TOPNOTCH TEAM
For inquiries visit www.topnotchboardprep.com.ph or https://www.facebook.com/topnotchmedicalboardprep/
7. Variations in which of the following components of blood or CSF ANSWER:
do not affect respiration?
A. Arterial HCO3− concentration • The rhythmic discharges from the brain that produce affects
B. Arterial H+ concentration respiration are alterations in arterial PO2, PCO2, H+, and HCO3-
C. Arterial Na+ concentration concentration, and this chemical control of breathing is
D. CSF CO2 concentration supplemented by a number of non-chemical influences.
Ganong. Review of Medical Physiology 23rd ed. Chapter 37, page 625 ..
E. CSF H+ concentration
8. Stimulation of the central (proximal) end of a cut vagus nerve ANSWER: E
would be expected to • “ Stretching of the lungs during inspiration initiates impulses in
A. increase heart rate afferent pulmonary vagal fibers. These impulses inhibit
B. stimulate inspiration inspiratory discharge. This is why the depth of inspiration is
C. inhibit coughing increased after vagotomy and apneusis develops if the vagi are cut
D. raise blood pressure after damage to the pneumotaxic center.”
E. cause apnea Ganong. Review of Medical Physiology 23rd ed. Chapter 37, page 627.

SECTION 7: RENAL PHYSIOLOGY


(CHAPTER 37) RENAL FUNCTION AND MICTURITION
ANSWER: A
• Proximal tubule:
• Reabsorbs two-thirds, or
67%, of the filtered Na+
and H2O, more than any
other part of the
1. In the presence of vasopressin, the greatest fraction of filtered nephron.
water is absorbed in the
A. proximal tubule.
B. loop of Henle.
C. distal tubule.
D. cortical collecting duct.
E. medullary collecting duct

Costanzo. BRS Physiology 5th


ed. Chapter 5 , page 156
ANSWER: A
• Proximal tubule:
• Reabsorbs two-thirds, or
67%, of the filtered Na+
and H2O, more than any
other part of the nephron.
2. In the absence of vasopressin, the greatest fraction of filtered
water is absorbed in the
A. proximal tubule.
B. loop of Henle.
C. distal tubule.
D. cortical collecting duct.
E. medullary collecting duct

Costanzo. BRS Physiology 5th ed.


Chapter 5 , page 156

ANSWER: A
3. If the clearance of a substance which is freely filtered is less than • Inulin
that of inulin, o freely filtered
A. there is net reabsorption of the substance in the tubules. o neither reabsorbed nor secreted in the tubules
B. there is net secretion of the substance in the tubules. o nontoxic,
C. the substance is neither secreted nor reabsorbed in the o not metabolized by the body.
tubules. • If the clearance of a substance which is freely filtered is less
D. the substance becomes bound to protein in the tubules. than that of inulin = (+) tubular REABSORPTION of substance.
E. the substance is secreted in the proximal tubule to a greater • If the clearance of a substance which is freely filtered is more
degree than in the distal tubule. than that of inulin = (+) tubular SECRETION of subtance.
Ganong. Review of Medical Physiology 23rd ed. Chapter 37 , page 626.
ANSWER: A
• Proximal tubule:
4. Glucose reabsorption occurs in the • reabsorbs about 65% of the
A. proximal tubule. filtered sodium, chloride,
B. loop of Henle. bicarbonate, and potassium
C. distal tubule. and essentially all the
D. cortical collecting duct. filtered glucose and amino
E. medullary collecting duct. acids.

Ganong. Review of Medical Physiology 23rd ed. Chapter 37 , pages 629-630 .

TOPNOTCH MEDICAL BOARD PREP MED PHYSIOLOGY GANONG SUPPLEMENT HANDOUT BY THE TOPNOTCH TEAM Page 43 of 46
For inquiries visit www.topnotchboardprep.com.ph or https://web.facebook.com/topnotchmedicalboardprep/
TOPNOTCH MEDICAL BOARD PREP PHYSIOLOGY GANONG SUPPLEMENT HANDOUT BY THE TOPNOTCH TEAM
For inquiries visit www.topnotchboardprep.com.ph or https://www.facebook.com/topnotchmedicalboardprep/

ANSWER: E
• The mineralocorticoids act primarily in the cortical collecting
ducts to increase the number of active epithelial sodium channels
(ENaCs) in this part of the nephron

5. On which of the following does aldosterone exert its greatest


effect?
A. Glomerulus
B. Proximal tubule
C. Thin portion of the loop of Henle
D. Thick portion of the loop of Henle
E. Cortical collecting duct

Ganong. Review of Medical Physiology 23rd ed. Chapter 38, page 658.
ANSWER: C

6. What is the clearance of a substance when its concentration in


the plasma is 10 mg/dL, its concentration in the urine is 100
mg/dL, and urine flow is 2 mL/min?
A. 2 mL/min
B. 10 mL/min
C. 20 mL/min
D. 200 mL/min
E. Clearance cannot be determined from the information given

Ganong. Review of Medical Physiology 23rd ed. Chapter 38 , page 657 .


7. As urine flow increases during osmotic diuresis ANSWER: D
A. the osmolality of urine falls below that of plasma.
B. the osmolality of urine increases because of the increased
amounts of nonreabsorbable solute in the urine. • “In osmotic diuresis, As the load of excreted solute is increased, the
C. the osmolality of urine approaches that of plasma because concentration of the urine approaches that of plasma in spite of
maximal vasopressin secretion, because an increasingly large fraction of the
plasma leaks into the tubules.
excreted urine is isotonic proximal tubular fluid.”
D. the osmolality of urine approaches that of plasma because an
increasingly large fraction of the excreted urine is isotonic Ganong. Review of Medical Physiology 23rd ed. Chapter 38 , page 657 .
proximal tubular fluid.
E. the action of vasopressin on the renal tubules is inhibited.

(CHAPTER 38) REGULATION OF EXTRACELLULAR FLUID COMPOSITON AND VOLUME


ANSWER: E
• ECF volume expansion release of ANP and BNP natriuresis
1. Dehydration increases the plasma concentration of all the and diuresis.
following hormones except • ECF volume contraction (i.e.:dehydration)  activation of
A. vasopressin. Renin-Angiotensin-Aldosterone System Sodium
B. angiotensin II. reabsorption and vasoconstriction restoration of ECF volume
C. aldosterone. and maintenance of blood pressure.
D. norepinephrine. • ECF volume contraction (i.e.:dehydration)  vasopressin
E. atrial natriuretic peptide. secretion Water reabsorption  restoration of ECF volume.
• ECF volume contraction (i.e.:dehydration)  release of
norepinephrine vasoconstrictionmaintenance of BP
Ganong. Review of Medical Physiology 23rd ed. Chapter 39 , page 669-670 .
2. In a patient who has become dehydrated, body water should be ANSWER: C
replaced by intravenous infusion of • In dehydration, restoration of fluid volume is essential. Hypotonic
A. distilled water. solution promotes entry of fluids into the cells.
B. 0.9% sodium chloride solution. • 5% glucose solution is isotonic when initially infused
C. 5% glucose solution. intravenously, but glucose is metabolized, so the net effect is that
D. hyperoncotic albumin. of infusing a hypotonic solution.
E. 10% glucose solution. Ganong. Review of Medical Physiology 23rd ed. Chapter 1 , page 6.
ANSWER: D
• Renin is secreted from the juxtaglomerular cells that surround
the renal afferent arterioles as they enter the glomeruli.

3. Renin is secreted by
A. cells in the macula densa.
B. cells in the proximal tubules.
C. cells in the distal tubules.
D. granular cells in the juxtaglomerular apparatus.
E. cells in the peritubular capillary bed.

Ganong. Review of Medical Physiology 23rd ed. Chapter 22, page 357.
TOPNOTCH MEDICAL BOARD PREP MED PHYSIOLOGY GANONG SUPPLEMENT HANDOUT BY THE TOPNOTCH TEAM Page 44 of 46
For inquiries visit www.topnotchboardprep.com.ph or https://web.facebook.com/topnotchmedicalboardprep/
TOPNOTCH MEDICAL BOARD PREP PHYSIOLOGY GANONG SUPPLEMENT HANDOUT BY THE TOPNOTCH TEAM
For inquiries visit www.topnotchboardprep.com.ph or https://www.facebook.com/topnotchmedicalboardprep/
ANSWER: E
4. Erythropoietin is secreted by • Erythropoietin is produced by:
A. cells in the macula densa. o interstitial cells in the peritubular capillary bed of the kidneys
B. cells in the proximal tubules. o perivenous hepatocytes in the liver.
C. cells in the distal tubules. o in the brain, where it exerts a protective effect against
D. granular cells in the juxtaglomerular apparatus. excitotoxic damage triggered by hypoxia
E. cells in the peritubular capillary bed. o in the uterus and oviducts, where it is induced by estrogen and
appears to mediate estrogen-dependent angiogenesis.
Ganong. Review of Medical Physiology 23rd ed. Chapter 39 , page 677..
5. When a woman who has been on a low-sodium diet for 8 days ANSWER: D
is given an intravenous injection of captopril, a drug that
inhibits angiotensin-converting enzyme, which of the following
would be expected? • Angiotensin II inhibitors like captopril lower blood pressure
A. BP to rise because cardiac output would fall principally by decreasing peripheral vascular resistance.
B. BP to rise because peripheral resistance would fall Cardiac output and heart rate are not significantly changed.
C. BP to fall because cardiac output would fall
D. BP to fall because peripheral resistance would fall Katzung. Basic and Clinical Pharmacology 13th ed.page 254.
E. Plasma renin activity to fall because circulating angiotensin
I level would rise
ANSWER: C
• Administration of a drug that blocks β-adrenergic receptors
6. Which of the following would not be expected to increase renin inhibits the stimulation of renin production by catecholamines
secretion? (mediated by β1 receptors).
A. Administration of a drug that blocks angiotensin- • Administration of a drug that blocks angiotensin-converting
converting enzyme enzyme and AT1 receptors renal arterial pressure renin
B. Administration of a drug that blocks AT1 receptors secretion.
C. Administration of a drug that blocks β-adrenergic receptors • Constriction of the aorta between the celiac artery and the renal
D. Constriction of the aorta between the celiac artery and the arteries renal arterial pressure renin secretion.
renal arteries • Administration of a drug that reduces ECF volume renal nerve
E. Administration of a drug that reduces ECF volume discharge and decreased renal arterial pressure renin secretion
Ganong. Review of Medical Physiology 23rd ed. Chapter 22, page 358.
Katzung. Basic and Clinical Pharmacology 13th ed.page 245.
ANSWER:
7. Which of the following is least likely to contribute to the • Effects of ACE inhibitors:
beneficial effects of angiotensin-converting enzyme inhibitors o Vasodilation decreased peripheral resistance reduced
in the treatment of heart failure? afterloaddecreased cardiac growth
A. Vasodilation o Decreased plasma aldosterone  reduced long-term
B. Decreased cardiac growth remodelling.
C. Decreased cardiac afterload o Increased plasma renin activity increased angiotensin and
D. Increased plasma renin activity aldosterone secretion vasoconstriction and salt retention
E. Decreased plasma aldosterone increased afterload, preload, and cardiac remodelling
increased severity of heart failure.
Katzung. Basic and Clinical Pharmacology 13th ed.page 298.

(CHAPTER 39) ACIDIFICATION OF THE URINE AND BICARBONATE EXCRETION


ANSWER: C
1. Which of the following is the principal buffer in interstitial fluid?
A. Hemoglobin • Inside the red blood cells: dissociation of the imidazole groups of
B. Other proteins the histidine residues in hemoglobin
C. Carbonic acid • Principal buffer in interstitial fluid: Carbonic acid (via carbonic
D. H2PO4 acid-bicarbonate system)
E. Compounds containing histidine • Principal buffer in kidneys: H2PO4
Ganong. Review of Medical Physiology 23rd ed. Chapter 36, page 614, 615.
2. Increasing alveolar ventilation increases the blood pH because ANSWER: D
A. it activates neural mechanisms that remove acid from the
blood. • In response to metabolic acidosis ventilation is increased
B. it makes hemoglobin a stronger acid. decrease in PCO2 (eg, from 40 mm Hg to 20 mm Hg) increase
C. it increases the Po2 of the blood. in pH toward normal.
D. it decreases the Pco2 in the alveoli.
E. the increased muscle work of increased breathing generates Ganong. Review of Medical Physiology 23rd ed. Chapter 36 , page 616.
more CO2.
ANSWER: D
3. In uncompensated metabolic alkalosis
A. the plasma pH, the plasma HCO3– concentration, and the • Identify the pH:
o High= alkalosis
arterial Pco2 are all low. o Low= acidosis
B. the plasma pH is high and the plasma HCO3 – concentration • Determine if metabolic or respiratory:
and arterial Pco2 are low. o If the pH goes the same direction with HCO3= METABOLIC
C. the plasma pH and the plasma HCO3– concentration are low o If the pH goes the opposite direction with pCO2= RESPIRATORY
and the arterial Pco2 is normal. • Determine if there is compensation:
D. the plasma pH and the plasma HCO3– concentration are high o For metabolic= respiratory compensation (there must be a
and the arterial Pco2 is normal. changed in pCO2)
E. the plasma pH is low, the plasma HCO3– concentration is o For respiratory= metabolic compensation (there must be a
changed in HCO3)
high, and the arterial Pco2 is normal.
Ganong. Review of Medical Physiology 23rd ed. Chapter 36, page 615, 616 .

TOPNOTCH MEDICAL BOARD PREP MED PHYSIOLOGY GANONG SUPPLEMENT HANDOUT BY THE TOPNOTCH TEAM Page 45 of 46
For inquiries visit www.topnotchboardprep.com.ph or https://web.facebook.com/topnotchmedicalboardprep/
TOPNOTCH MEDICAL BOARD PREP PHYSIOLOGY GANONG SUPPLEMENT HANDOUT BY THE TOPNOTCH TEAM
For inquiries visit www.topnotchboardprep.com.ph or https://www.facebook.com/topnotchmedicalboardprep/
ANSWER: B

4. In a patient with a plasma pH of 7.10, the [HCO3–]/[H2CO3] ratio


in plasma is
A. 20.
B. 10.
C. 2.
D. 1.
E. 0.1.

Ganong. Review of Medical Physiology 23rd ed. Chapter 36, page 615 .

TOPNOTCH MEDICAL BOARD PREP MED PHYSIOLOGY GANONG SUPPLEMENT HANDOUT BY THE TOPNOTCH TEAM Page 46 of 46
For inquiries visit www.topnotchboardprep.com.ph or https://web.facebook.com/topnotchmedicalboardprep/

Anda mungkin juga menyukai